Sunteți pe pagina 1din 324

Exercise 1.

1 (Solutions)
mathcity.org Textbook of Algebra and Trigonometry for Class XI
Merging man and maths Available online @ http://www.mathcity.org, Version: 1.0.0
FSc-I – Ex 1.1 2
FSc-I – Ex 1.1 3
FSc-I – Ex 1.1 4
FSc-I – Ex 1.1 5
FSc-I – Ex 1.1 6
FSc-I – Ex 1.1 7
FSc-I – Ex 1.1 8
FSc-I – Ex 1.1 9

************************

If you have any question:


ASK AT
http://forum.mathcity.org

************************

*******************************
http://www.MathCity.org
Exercise 1.2 (Solutions)
MathCity.org Textbook of Algebra and
and Trigonometry for Class XI
Merging man and maths Available online at http://www.MathCity.org , Version: 1.0.0
FSc-I- Ex 1.2 - 2 www.MathCity.org
www.MathCity.org FSc-I- Ex 1.2 - 3
FSc-I- Ex 1.2 - 4 www.MathCity.org
www.MathCity.org FSc-I- Ex 1.2 - 5

******************************************
Available online at http://www.MathCity.org
Exercise 1.3 (Solutions)
MathCity.org Textbook of Algebra and
and Trigonometry for Class XI
Merging man and maths Available online at http://www.MathCity.org , Version: 1.0.0
FSc-I- Ex 1.3 - 2 www.MathCity.org
www.MathCity.org FSc-I- Ex 1.3 - 3
FSc-I- Ex 1.3 - 4 www.MathCity.org
www.MathCity.org FSc-I- Ex 1.3 - 5

******************************************
Available online at http://www.MathCity.org

If you have any question ask us at


http://forum.mathcity.org
Exercise 2.2 (Solutions)
MathCity.org Textbook of Algebra and
and Trigonometry for Class XI
Merging man and maths Available online at http://www.MathCity.org , Version: 1.0.0
FSc-I- Ex 2.2 - 2 www.MathCity.org
www.MathCity.org FSc-I- Ex 2.2 - 3
FSc-I- Ex 2.2 - 4 www.MathCity.org

To download notes of other


exercises visit:
http://www.MathCity.org

MCQs, model papers and old


papers are also available.
MathCity.org Exercise 2.3 (Solutions)
Textbook of Algebra and
and Trigonometry for Class XI
Merging man and maths Available online at http://www.MathCity.org , Version: 1.0.0
FSc-I- Ex 2.3 - 2 www.MathCity.org
www.MathCity.org FSc-I- Ex 2.3 - 3
FSc-I- Ex 2.3 - 4 www.MathCity.org
www.MathCity.org FSc-I- Ex 2.3 - 5

To download notes of other


exercises visit:
http://www.MathCity.org

MCQs, model papers and old


papers are also available.
ExePGise 2.4 (Solutions)
mathcity.or T EX TBO OK OF A LG EB RA AND T RI GO NOM ETRY FO R C LAS S
Ava i l a b l e on l i ne @ h tt p :/ / www. ma t h ci t y. or g, Ve r s i on : 1 . 0 . 0
XI

g
M e r gi n g ma n a n d mat h s

Question # 1(i) ~ p→q


Conditional:
Converse: q →~ p
Inverse: p →~ q
Contrapositive: ~ q →p
Question # 1(ii) q →p
Conditional:
Converse: p→q
Inverse: ~ q→ ~ p
Contrapositive: ~ p → ~q
Question # 1(iii) ~ p → ~q
Conditional:
Converse: ~ q→ ~ p
Inverse: p → q
Contrapositive: q→p
Question # 1(iv) Do yourself as above

Question # 2 (i)
Statement: (p → ~ p) v (p → q)
p q -p p→ ~p p → q ( p → ~ p ) v ( p → q)
T T F F T T
T F F F F F
F T T T T T
F F T T T T

Question # 2 (ii)
Statement: (p A ~ p) ® q
- A
p q p PA ~ P (
p ~ p) ® q
T T F F T
T F F F T
F T T F T
F F T F T

Question # 2 (iii)
Statement: ~(p ® q)«(p A ~ q)
-
p 0 q p®q ~(p ® q)

«(
)
)q
>

>
p

p
i

i
T T F T F F T
T F T F T T T
F T F T F F T
F F T T F F T
FSc I - 2.4 - 2 & Tautology:
The statement which is true for all possible values of the variables in it is called
tautology.
& Contingency:
The statement which is true or false depending upon the truth values of the variables
involved in it is called a contingency.
& Absurdity or Contradiction:
The statement which is false for all the possible values of the variables involved in it is
called an absurdity or contradiction.
Question # 3 (i)
Statement: (p A q) ® p
P q p q
A
pAq®p
T T T T
T F F T
F T F T
F F F T
The last column of the above table shows that the statement is true for all values ofp and
q thus given statement is tautology.
Question # 3 (ii)
Statement: p ® (p v q)
p q pvq p ®(p v q )
T T T T
T F T T
F T T T
F F F T
The last column of the above table shows that the statement is true for all values of p
and q thus given statement is tautology
Question # 3 (iii)
Statement: ~(p ® q) ® p
p 0 p®q ~ (p ® q) ~ (p ® q)®
T T T F p T
T F F T T
F T T F T
F F T F T
The last column of the above table shows that the statement is true for all values ofp and
q thus given statement is tautology.
Question # 3 (iv)
Statement: ~ q A (p ® q) ® ~ p
-
p q -p q p®q ~ q a (p ® q ) ~ q a (p ® q ) ® ~ p
T T F F T F T
T F F T F F T
F T T F T F T
F F T T T T T
The last column of the above table shows that the statement is true for all values ofp and
q thus given statement is tautology.
FSc I - 2.4 - 3
Question # 4 (i)
Statement: ~(p ® q) ® p
p -p PA ~ P

T F F
F T F

The last column of the above table shows that the statement is false for all values ofp
and q thus given statement is absurdity.
Question # 4 (ii)
Statement: p ® (q ® p)
p q q®p p ® (q ® p )
T T T T
T F T T
F T F T
F F T T

The last column of the above table shows that the statement is true for all values ofp
and q thus given statement is tautology.

Question # 4 (iii)
Statement: q v (~ q v p)
P q ~q ~qvp q v (~ q v p )
T T F T T
T F T T T
F T F F T
F F T T T

The last column of the above table shows that the statement is true for all values ofp
and q thus given statement is tautology.

Question # 5
Consider the truth table
p q -p -q p A q p v ( ~ pA ~ q ) v ( p A q ) p v ( ~ pA ~ q )
>
p

T T F F T F T T
T F F T F F T T
F T T F F F F F
F F T T F T T T
The last two column of the above table are identical this shows that the statement p v
(~ p A ~ q) v (p A q) and p v (~ p A ~ q) are equal
i.e. p v (~ pA ~ q) v (p A q) = p v (~ pA ~ q)

Made by: Atiq ur Rehman (atiq@mathcity.org), http://www.mathcity.org

Error Analyst

Wa i t i n g for s ome on e
mathcity.org Exercise 2.4 (Solutions)
Textbook of Algebra and Trigonometry for Class XI
Merging man and maths Available online @ http://www.mathcity.org, Version: 1.0.0

Question # 1(i)
Conditional: ~ p®q
Converse: q ®~ p
Inverse: p ®~ q
Contrapositive: ~q®p

Question # 1(ii)
Conditional: q® p
Converse: p ®q
Inverse: ~ q ®~ p
Contrapositive: ~ p ®~ q

Question # 1(iii)
Conditional: ~ p ®~ q
Converse: ~ q ®~ p
Inverse: p ®q
Contrapositive: q®p

Question # 1(iv)
Do yourself as above
Question # 2 (i)
Statement: ( p ® ~ p ) Ú ( p ® q )
p q ~p p ®~ p p ®q ( p ®~ p) Ú ( p ® q)
T T F F T T
T F F F F F
F T T T T T
F F T T T T

Question # 2 (ii)
Statement: ( p Ù ~ p ) ® q
p q ~p p Ù~ p ( p Ù ~ p) ® q
T T F F T
T F F F T
F T T F T
F F T F T

Question # 2 (iii)
Statement: ~ ( p ® q ) « ( p Ù ~ q )
p Q ~q p ®q ~ ( p ® q) pÙ ~ q ( p Ù ~ q) « ~ ( p ® q)
T T F T F F T
T F T F T T T
F T F T F F T
F F T T F F T
FSc I – 2.4 - 2
Ò Tautology:
The statement which is true for all possible values of the variables in it is called
tautology.
Ò Contingency:
The statement which is true or false depending upon the truth values of the
variables involved in it is called a contingency.
Ò Absurdity or Contradiction:
The statement which is false for all the possible values of the variables involved
in it is called an absurdity or contradiction.
Question # 3 (i)
Statement: ( p Ù q ) ® p
P q pÙq pÙq®p
T T T T
T F F T
F T F T
F F F T
The last column of the above table shows that the statement is true for all values
of p and q thus given statement is tautology.

Question # 3 (ii)
Statement: p ® ( p Ú q )
p q pÚq p ® ( p Ú q)
T T T T
T F T T
F T T T
F F F T
The last column of the above table shows that the statement is true for all values
of p and q thus given statement is tautology
Question # 3 (iii)
Statement: ~ ( p ® q ) ® p
p Q p ®q ~ ( p ®q) ~ ( p ®q) ® p
T T T F T
T F F T T
F T T F T
F F T F T
The last column of the above table shows that the statement is true for all values
of p and q thus given statement is tautology.
Question # 3 (iv)
Statement: ~ q Ù ( p ® q ) ® ~ p
p q ~p ~q p ®q ~ q Ù ( p ® q) ~ q Ù ( p ® q) ® ~ p
T T F F T F T
T F F T F F T
F T T F T F T
F F T T T T T
The last column of the above table shows that the statement is true for all values
of p and q thus given statement is tautology.
FSc I – 2.4 - 3
Question # 4 (i)
Statement: ~ ( p ® q ) ® p
p ~p pÙ ~ p
T F F
F T F
The last column of the above table shows that the statement is false for all values
of p and q thus given statement is absurdity.

Question # 4 (ii)
Statement: p ® (q ® p )
p q q® p p ® (q ® p)
T T T T
T F T T
F T F T
F F T T
The last column of the above table shows that the statement is true for all values
of p and q thus given statement is tautology.

Question # 4 (iii)
Statement: q Ú (~ q Ú p )
P q ~q ~qÚ p q Ú (~ q Ú p )
T T F T T
T F T T T
F T F F T
F F T T T
The last column of the above table shows that the statement is true for all values
of p and q thus given statement is tautology.
Question # 5
Consider the truth table
p q ~p ~q pÙq ~ pÙ ~ q p Ú (~ p Ù ~ q ) Ú ( p Ù q ) p Ú (~ p Ù ~ q )
T T F F T F T T
T F F T F F T T
F T T F F F F F
F F T T F T T T
The last two column of the above table are identical this shows that the statement
p Ú (~ p Ù ~ q ) Ú ( p Ù q ) and p Ú (~ p Ù ~ q ) are equal
i.e. p Ú (~ p Ù ~ q ) Ú ( p Ù q ) = p Ú (~ p Ù ~ q )

Made by: Atiq ur Rehman (atiq@mathcity.org), http://www.mathcity.org

Error Analyst

Waiting for someone

THE END
mathcity.org Exercise 2.5 (Solutions)
Textbook of Algebra and Trigonometry for Class XI
Page 57

Merging man and maths Available online @ http://www.mathcity.org, Version: 1.4.0

J Question # 1
The corresponding formula of logic is
~ ( p ∧ q) = ~ p ∨ ~ q

p q ~p ~q p ∧q ~ ( p ∧ q) ~ p∨ ~ q
T T F F T F F
T F F T F T T
F T T F F T T
F F T T F T T

The last two columns of the above table shows that ~ ( p ∧ q ) = ~ p ∨ ~ q


and hence ( A ∩ B)′ = A′ ∪ B′ .

H Question # 2
The corresponding formula of logic is
( p ∨ q) ∨ r = p ∨ (q ∨ r )
p q r p∨q q∨r ( p ∨ q) ∨ r p ∨ (q ∨ r )
T T T T T T T
T T F T T T T
T F T T T T T
T F F T F T T
F T T T T T T
F T F T T T T
F F T F T T T
F F F F F F F
The last two columns of the above table shows that ( p ∨ q ) ∨ r = p ∨ ( q ∨ r )
and hence ( A ∪ B) ∪ C = A ∪ ( B ∪ C ) .

D Question # 3
The corresponding formula of logic is
( p ∧ q) ∧ r = p ∧ (q ∧ r )
p q r p∧q q∧r ( p ∧ q) ∧ r p ∧ (q ∧ r )
T T T T T T T
T T F T F F F
T F T F F F F
T F F F F F F
F T T F T F F
F T F F F F F
F F T F F F F
F F F F F F F
The last two columns of the above table shows that ( p ∧ q ) ∧ r = p ∧ ( q ∧ r )
and hence ( A ∩ B) ∩ C = A ∩ ( B ∩ C ) .
2.5 - 2

B Question # 4
A ∪ ( B ∩ C ) = ( A ∪ B) ∩ ( A ∪ C )
The corresponding formula of logic is
p ∨ (q ∧ r ) = ( p ∨ q) ∧ ( p ∨ r )

p q r q∧r p∨q p∨r p ∨ (q ∧ r ) ( p ∨ q) ∧ ( p ∨ r )


T T T T T T T T
T T F F T T T T
T F T F T T T T
T F F F T T T T
F T T T T T T T
F T F F T F F F
F F T F F T F F
F F F F F F F F

The last two columns of the above table shows that p ∨ (q ∧ r ) = ( p ∨ q ) ∧ ( p ∨ r )


and hence A ∪ ( B ∩ C ) = ( A ∪ B) ∩ ( A ∪ C ) .

Made by: Atiq ur Rehman (atiq@mathcity.org), http://www.mathcity.org

These notes are available online at


http://www.mathcity.org/fsc
Submit error/mistake at
http://www.mathcity.org/error

Error Analyst

Waiting for someone.


mathcity.org Exercise 2.8 (Solutions)
Textbook of Algebra and Trigonometry for Class XI
Merging man and maths Available online @ http://www.mathcity.org, Version: 1.0.0

Ø Question # 1
i) From the given table we have
0 + 0 = 0 and 0 + 1 = 1 ⊕ 0 1
This show that 0 is the identity element. 0 1 1
ii) Since 1 + 1 = 0 (identity element) so the inverse 1 1 0
of 1 is 1.
iii) It is clear from table that element of the given set
satisfy closure law, associative law, identity law and inverse law
thus given set is group under ⊕ .
Also it satisfies commutative law so it is an abelian group.

Ø Question # 2
Suppose G = {0,1, 2,3}
⊕ 0 1 2 3
i) The given table show that each element of the
0 0 1 2 3
table is a member of G thus closure law holds.
ii) ⊕ is associative in G . 1 1 2 3 0
iii) Table show that 0 is identity element w.r.t. ⊕ . 2 2 3 0 1
iv) Since 0 + 0 = 0, 1 + 3 = 0, 2 + 2 = 0, 3 + 1 = 0 3 3 0 1 2
⇒ 0−1 = 0 , 1−1 = 3 , 2 −1 = 2 , 3−1 = 1
v) As the table is symmetric w.r.t. to the principal diagonal. Hence commutative law
holds.

Ø Question # 3
(i) As 0∈ ¤ , multiplicative inverse of 0 in not in set ¤ . Therefore the set of rational
number is not a group w.r.t to “ ⋅ ”.
(ii) a- Closure property holds in ¤ under + because sum of two rational number is
also rational.
b- Associative property holds in ¤ under addition.
c- 0∈ ¤ is an identity element.
d- If a ∈ ¤ then additive inverse − a ∈¤ such that a + (− a ) = (− a ) + a = 0 .
Therefore the set of rational number is group under addition.
(iii) a- Since for a , b ∈ ¤+ , ab ∈ ¤+ thus closure law holds.
b- For a , b, c ∈¤ , a (bc) = (ab)c thus associative law holds.
c- Since 1∈ ¤ + such that for a ∈ ¤ + , a × 1 = 1 × a = a . Hence 1 is the identity element.
1 1 1 1
d- For a ∈ ¤ + , ∈ ¤ + such that a × = × a = 1. Thus inverse of a is .
a a a a
Hence ¤ is group under addition.
+

(iv) Since ¢ = {0, ±1, ± 2, ± 3,............}


a- Since sum of integers is an integer therefore for a , b ∈ ¢ , a + b ∈ ¢ .
b- Since a + ( b + c ) = ( a + b ) + c thus associative law holds in ¢ .
c- Since 0∈ ¢ such that for a ∈ ¢ , a + 0 = 0 + a = ¢ . Thus 0 an identity element.
d- For a ∈ ¢ , −a ∈¢ such that a + (− a ) = (− a ) + a = 0 . Thus inverse of a is − a .
(v) Since ¢ = {0, ±1, ± 2, ± 3,............}
1
For any a ∈ ¢ the multiplicative inverse of a is ∉ ¢ . Hence ¢ is not a group under
a
multiplication.
FSc-I / 2.8 - 2

Ø Question # 4
As E + E = E, E + O = O, O + O = E
⊕ E O
Thus the table represents the sums of the elements of set { E , O} .
E E O
The identity element of the set is E because
E + E = E + E = E & E +O =O+ E = E . O O E
i) From the table each element belong to the set { E , O} .
Hence closure law is satisfied.
ii) ⊕ is associative in { E , O}
iii) E is the identity element of w.r.t to ⊕
iv) As O + O = E and E + E = E , thus inverse of O is O and inverse of E is E.
v) As the table is symmetric about the principle diagonal therefore ⊕ is commutative.
Hence { E , O} is abelian group under ⊕ .

Ø Question # 5
Suppose G = {1,ω ,ω 2 } ⊗ 1 ω ω2
i) A table show that all the entries belong to G. 1 1 ω ω2
ii) Associative law holds in G w.r.t. multiplication. ω ω ω2 1
e.g. 1× (ω × ω 2 ) = 1× 1 = 1 ω2 ω2 1 ω
(1 × ω ) × ω 2 = ω × ω 2 = 1
iii) Since 1 × 1 = 1 , 1× ω = ω × 1 = ω , 1 × ω 2 = ω 2 × 1 = ω 2
Thus 1 is an identity element in G.
iv) Since 1 × 1 = 1 × 1 = 1 , ω × ω 2 = ω 2 × ω = 1 , ω 2 × ω = ω × ω 2 = 1
therefore inverse of 1 is 1, inverse of ω is ω 2 , inverse of ω 2 is ω .
v) As table is symmetric about principle diagonal therefore commutative law holds in G.
Hence G is an abelian group under multiplication.
Ø Question # 6
Given that G is a group under the operation ∗ and a , b ∈ G such that
a∗x =b
As a ∈ G and G is group so a −1 ∈ G such that
a −1 ∗ ( a ∗ x ) = a −1 ∗ b
⇒ ( a −1 ∗ a ) ∗ x = a −1 ∗ b as associative law hold in G.
⇒ e ∗ x = a −1 ∗ b by inverse law.
⇒ x = a −1 ∗ b by identity law.
And for
x∗a =b
⇒ ( x ∗ a ) ∗ a −1 = b ∗ a −1 For a ∈ G , a −1 ∈ G
⇒ x ∗ ( a ∗ a −1 ) = b ∗ a −1 as associative law hold in G.
⇒ x ∗ e = b ∗ a −1 by inverse law.
⇒ x = b ∗ a −1 by identity law.

Ø Question # 7
{
Consider G = a + 3 b | a, b ∈ ¤ }
i) Let a + 3b , c + 3d ∈G , where a, b, c & d are rational.
(a + ) ( )
3b + c + 3d = ( a + c ) + 3 ( b + d ) = a′ + 3b′ ∈ G
where a′ = a + c and b′ = b + d are rational as sum of rational is rational.
Thus closure law holds in G under addition.
FSc-I / 2.8 - 3

ii) For a + 3b , c + 3d , e + 3 f ∈G
( ) (
(a + 3b) + ( c + 3d ) + (e + 3 f ) = (a + 3b) + ( c + e) + 3(d + f ) )
= ( a + ( c + e) ) + 3 ( b + ( d + f ) )
= ( ( a + c ) + e ) + 3 ( (b + d ) + f )
As associative law hold in ¤
( )
= (a + c) + 3(b + d ) + (e + 3 f )

= ( (a + )
3b) + (c + 3d ) + (e + 3 f )
Thus associative law hold in G under addition.
iii) 0 + 3 ⋅ 0 ∈ G as 0 is a rational such that for any a + 3b ∈G
(a + 3b) + (0 + 3 ⋅ 0) = (a + 0) + 3(b + 0) = a + 3b
And (0 + 3 ⋅ 0) + (a + 3b) = (0 + a) + 3(0 + b) = a + 3b
Thus 0 + 3 ⋅ 0 is an identity element in G.
iv) For a + 3b ∈G where a & b are rational there exit rational – a & −b such that
( )
(a + 3b) + (− a ) + 3(−b) = ( a + (− a ) ) + 3 ( b + (−b) ) = 0 + 3 ⋅ 0

& ( ( −a ) + )
3(−b) + (a + 3b) = ( (− a ) + a ) + 3 ( (−b) + b ) = 0 + 3 ⋅ 0
Thus inverse of a + 3b is (−a) + 3(−b) exists in G.
v) For a + 3b , c + 3d ∈G
(a + ) ( )
3b + c + 3d = (a + c ) + 3(b + d )
= (c + a) + 3(d + b) As commutative law hold in ¤ .
= (c + d 3) + (a + 3b)
Thus Commutative law holds in G under addition.
And hence G is an abelian group under addition.
Ø Question 8
Let A, B ∈ P( S ) where A & B are subsets of S.
As intersection of two subsets of S is subset of S.
Therefore A ∗ B = A ∩ B ∈ P( S ) . Thus closure law holds in P (S ) .
For A, B, C ∈ P( S )
A ∗ ( B ∗ C ) = A ∩ ( B ∩ C ) = ( A ∩ B) ∩ C = ( A ∗ B ) ∗ C
Thus associative law holds and P (S ) .
And hence ( P( S ), ∗) is a semi-group.
For A ∈ P( S ) where A is a subset of S we have S ∈ P( S ) such that
A∩ S = S ∩ A = A .
Thus S is an identity element in P (S ) . And hence ( P( S ), ∗) is a monoid.
Ø Question 9
Let x1 and x2 be the required elements.
∗ a b c
By associative law
a c a b
(a ∗ a) ∗ a = a ∗ (a ∗ a)
⇒ c∗a = a∗c b a b c
⇒ x1 = b c x1 x2 a
Now again by associative law
( a ∗ a ) ∗ b = a ∗ ( a ∗ b)
⇒ c ∗ b = a ∗ a ⇒ x2 = c
FSc-I / 2.8 - 4
Question 10
Let G be the all non-singular 2 × 2 matrices over the real field.
i) Let A, B ∈ G then A2×2 × B2×2 = C2×2 ∈ G
Thus closure law holds in G under multiplication.
ii) Associative law in matrices of same order under multiplication holds.
therefore for A, B, C ∈ G
A × ( B × C ) = ( A × B) × C
1 0
iii) I 2×2 =   is a non-singular matrix such that
0 1
A2×2 × I 2×2 = I 2×2 × A2×2 = A2×2
Thus I 2×2 is an identity element in G.
iv) Since inverse of non-singular square matrix exists,
therefore for A ∈ G there exist A−1 ∈ G such that AA−1 = A−1 A = I .
v) As we know for any two matrices A, B ∈ G , AB ≠ BA in general.
Therefore commutative law does not holds in G under multiplication.
Hence the set of all 2 × 2 non-singular matrices over a real field is a non-abelian group
under multiplication.

Made by: Atiq ur Rehman (mathcity@gmail.com),


http://www.mathcity.org

The End
…………………………………………………………………………………………………………………………………………………

Quotations
b The tragedy of life is not that it ends so soon, but that we wait so long to begin it.
W. M. Lewis

b It is neither good nor bad, but thinking makes it so.


William Shakespeare

b Why fear death? It is the most beautiful adventure in life.


Charles Frohman

b An unfortunate thing about this world is that the good habits are much easier to give
up than the bad ones.
W. Somerset Maugham

b The truth is that life is delicious, horrible, charming, frightful, sweet, bitter, and that it
is everything.
Anatole France

b There is big difference between getting ready to act and starting to act. Many are
forever ready to act.
Anonymous
…………………………………………………………………………………………………………………………………………………
MathCity.org Exercise 3.1 (Solutions)
Textbook of Algebra and
and Trigonometry for Class XI
Merging man and maths Available online at http://www.MathCity.org , Version: 1.0.0

To download notes of other exercises visit:


http://www.MathCity.org

MCQs, model papers and old papers are also available.

Facebook page: www.facebook.com/MathCity.org

Google+ : https://plus.google.com/b/113196409348253197516/

If you wish to share your notes, please visit

http://www.mathcity.org/participate
FSc-I- Ex 3.1 - 2 www.MathCity.org
www.MathCity.org FSc-I- Ex 3.1 - 3
FSc-I- Ex 3.1 - 4 www.MathCity.org
www.MathCity.org FSc-I- Ex 3.1 - 5
FSc-I- Ex 3.1 - 6 www.MathCity.org
www.MathCity.org FSc-I- Ex 3.1 - 7
FSc-I- Ex 3.1 - 8 www.MathCity.org
www.MathCity.org FSc-I- Ex 3.1 - 9
FSc-I- Ex 3.1 - 10 www.MathCity.org

To download notes of other exercises visit:


http://www.MathCity.org

MCQs, model papers and old papers are also available.

Facebook page: www.facebook.com/MathCity.org

Google+ : https://plus.google.com/b/113196409348253197516/

If you wish to share your notes, please visit

http://www.mathcity.org/participate
Exercise 3.3 (Solutions)
MathCity.org Textbook of Algebra and
and Trigonometry for Class XI
Merging man and maths Author: Prof. Tahir Nazir (University of Sargodha) , Version: 1.0.0
FSc-II- Ex 3.3 - 2 www.MathCity.org
www.MathCity.org FSc-II- Ex 3.3 - 3
FSc-II- Ex 3.3 - 4 www.MathCity.org
www.MathCity.org FSc-II- Ex 3.3 - 5
FSc-II- Ex 3.3 - 6 www.MathCity.org
www.MathCity.org FSc-II- Ex 3.3 - 7
FSc-II- Ex 3.3 - 8 www.MathCity.org
www.MathCity.org FSc-II- Ex 3.3 - 9
FSc-II- Ex 3.3 - 10 www.MathCity.org

----------------------------------------

******************************************
Available online at http://www.MathCity.org
Exercise 3.4 (Solutions)
MathCity.org Textbook of Algebra and Trigonometry for Class XI
Merging man and maths Author: Prof. Tahir Nazir (University of Sargodha) , Version: 1.0.0
FSc-II- Ex 3.3 - 2 www.MathCity.org
www.MathCity.org FSc-II- Ex 3.3 - 3
FSc-II- Ex 3.3 - 4 www.MathCity.org
www.MathCity.org FSc-II- Ex 3.3 - 5

******************************************
Available online at http://www.MathCity.org
FSc-I- Ex 4.1 - 2 www.MathCity.org

Available at http://www.MathCity.org/FSc
www.MathCity.org FSc-I- Ex 4.1 - 3

Available at http://www.MathCity.org/FSc
FSc-I- Ex 4.1 - 4 www.MathCity.org

Available at http://www.MathCity.org/FSc
www.MathCity.org FSc-I- Ex 4.1 - 5

Available at http://www.MathCity.org/FSc
FSc-I- Ex 4.1 - 6 www.MathCity.org

Provide by: Momin Ali


Fazaia Degree College, Peshawar.

To download notes of other exercises visit:


http://www.MathCity.org

MCQs, model papers and old papers are also available.


If you have any question, ask it at
http://forum.mathcity.org

Available at http://www.MathCity.org/FSc
FSc-I- Ex 4.2 - 2 www.MathCity.org

Available at http://www.MathCity.org/FSc
www.MathCity.org FSc-I- Ex 4.2 - 3

Available at http://www.MathCity.org/FSc
FSc-I- Ex 4.2 - 4 www.MathCity.org

Available at http://www.MathCity.org/FSc
www.MathCity.org FSc-I- Ex 4.2 - 5

Available at http://www.MathCity.org/FSc
FSc-I- Ex 4.2 - 6 www.MathCity.org

Available at http://www.MathCity.org/FSc
www.MathCity.org FSc-I- Ex 4.2 - 7

Available at http://www.MathCity.org/FSc
FSc-I- Ex 4.2 - 8 www.MathCity.org

Available at http://www.MathCity.org/FSc
www.MathCity.org FSc-I- Ex 4.2 - 9

Available at http://www.MathCity.org/FSc
FSc-I- Ex 4.2 - 10 www.MathCity.org

Available at http://www.MathCity.org/FSc
www.MathCity.org FSc-I- Ex 4.2 - 11

Available at http://www.MathCity.org/FSc
FSc-I- Ex 4.2 - 12 www.MathCity.org

Available at http://www.MathCity.org/FSc
www.MathCity.org FSc-I- Ex 4.2 - 13

Provide by: Momin Ali


Fazaia Degree College, Peshawar.

Available at http://www.MathCity.org/FSc
MathCity.org Exercise 4.3 (Solutions)
Textbook of Algebra and Trigonometry for Class XI
Merging man and maths Available online at http://www.MathCity.org , Version: 1.0.0
FSc-I- Ex 4.3 - 2 www.MathCity.org

Available at http://www.MathCity.org/FSc
www.MathCity.org FSc-I- Ex 4.3 - 3

Available at http://www.MathCity.org/FSc
FSc-I- Ex 4.3 - 4 www.MathCity.org

Available at http://www.MathCity.org/FSc
www.MathCity.org FSc-I- Ex 4.3 - 5

Available at http://www.MathCity.org/FSc
FSc-I- Ex 4.3 - 6 www.MathCity.org

Available at http://www.MathCity.org/FSc
www.MathCity.org FSc-I- Ex 4.3 - 7

Available at http://www.MathCity.org/FSc
FSc-I- Ex 4.3 - 8 www.MathCity.org

Available at http://www.MathCity.org/FSc
www.MathCity.org FSc-I- Ex 4.3 - 9

Available at http://www.MathCity.org/FSc
FSc-I- Ex 4.3 - 10 www.MathCity.org

Provide by: Momin Ali


Fazaia Degree College, Peshawar.

To download notes of other exercises visit:


http://www.MathCity.org

MCQs, model papers and old papers are also available.


If you have any question, ask it at
http://forum.mathcity.org

Available at http://www.MathCity.org/FSc
Exercise 4.4 (Solutions)
mathcity.org Textbook of Algebra and Trigonometry for Class XI
Merging man and maths Available online @ http://www.mathcity.org, Version: 1.0.0

{ Question # 1
(i) Let x be a cube root of 8 then
1
x  (8)3  x3  8 Review:
1  3
 x3 8  0  (x )3  (2)3  0 
2
 (x  2)(x 2  2x  4)  0 1  3
2 
 x 2  0 or x 2  2x  4  0 2

2  (2)2  4(1)(4)
 x 2 or x
2(1)
2  4  16 2  12
 
2 2
2  2 3  1  3 
  2  
2  2 
 1  3   1  3 
 x  2   or x  2  
 2   2 
 x  2 or x  2 2
Hence cube root of 8 are 2,2 and 2 2 .
(ii) Hint
Considering x as a cube root of 8 and Solving as above you will get the following
values of x
2  2 3 2  2 3
x  2 , x  , x
2 2
 1  3   1  3 
 x  2   , x  2  
 2   2 
 x  2 2 , x  2
Hence cube root of 8 are 2,  2 and 2 2 .

(iii) Do yourself as (iv) below.


(iv) Let x be a cube root of 27 then
1
x  (27)3  x 3  27
 x 3  27  0  (x )3  (3)3  0
 (x  3)(x 2  3x  9)  0
 x 3 0 or x 2  3x  9  0
3  (3)2  4(1)(9)
 x  3 or x
2(1)
3  9  36 3  27 3  3 3
  
2 2 2
3  3 3 3  3 3
 x or x 
2 2
FSC-I - 4.4 - 2
 1  3   1  3 
 x  3   or x  3  
 2   2 
 x  3 2 or x   3
Hence cube root of 27 are 3,  3 and 3 2 .

(v) Let x be a cube root of 64 then


1
x  (64)3  x 3  64
 x 3  64  0  (x )3  (4)3  0
 (x  4)(x 2  4x  16)  0
 x 4  0 or x 2  4x  16  0
4  (4)2  4(1)(16)
 x 4 or x
2(1)
4  16  64 4  48
 
2 2
4  4 3
 Q 48  16  3
2
4  4 3 4  4 3
 x or x 
2 2
 1  3  1  3 
 x  4   or x  4  
 2   2 
 x  4 or x  4 2
Hence cube root of 64 are 4, 4 and 4 2 .

{ Question # 2
1     2   1    w 2  2w 2 
8 8
(i)
 0  2w 2 
8
Q 1    2  0
 (2)8 (w 2 )8  256 16
 256 15    256  3   
5

 256 1    256  Q 3  1


5
Answer

(ii)  28  w 29  1   27     27   2  1
  3       3    2  1
9 9

 1    1   2  1 Q 3  1
9 9

   2  1
 0 Answer Q 1    2  0

(iii) 1     2 1     2 
 1     2  2 2 1     2  2  Q 1    2  0
 0  2 2 0  2   2 2 2 
 4  3  4(1) = 4 Answer Q 3  1

Made By: Atiq ur Rehman (mathcity@gmail.com), http://www.mathcity.org


FSC-I - 4.4 - 3
1  3 7 1  3 7 1  3
(iv)      Q 
 2   2  2
  7  ( 2 )7 1  3
2 
  7  14 2
  6    12   2   3      3    2
3 4

 13    14   2
   2  1 Answer Q 1    2  0

1  3   1  3 
5 5
(v)
 1  3 5  1  3 5 1  3
 2    2   Q 
 2   2  2
1  3
 2   5  2   2 
5
2 
2
 32  5  32 10  32  3   2  32  9  1

Q  9   3   (1)3  1
3
 32(1)   2  32(1)  
 32(   2 )
 32(1)  32 Q 1    2  0

{ Question # 3
(i) R.H.S= x  y x  y x   2y 
= x  y  x x   2y   y x   2y 
 
= x  y  x   xy  xy   y 
2 2 3 2

= x  y  x 2   2    xy  (1)y 2  Q 3  1
 
= x  y  x  1 xy  y 
2 2
Q 1    2  0
= x  y  x 2  xy  y 2      2  1
=x 3  y 3  L.H.S
(ii) R.H.S= x  y  z x  y   2z x   2y  z 
= x  y  z [x 2   2xy  xz  xy   3y 2   2yz
 2xz   4yz   3z 2 ]
= x  y  z [x 2  ( 2  )xy  (   2 )xz  ( 2   4 )yz
 3y 2   3z 2 ]
= x  y  z [x 2  (1)xy  (1)xz  ( 2  )yz (1)y 2  (1)z 2 ]
Q  4   &    2  1
= x  y  z [x 2  y 2  z 2  xy  (1)yz  xz ]
= x  y  z [x 2  y 2  z 2  xy  yz  xz ]
 x 3  y 3  z 3  3xyz  L.H.S
(iii) L.H.S= (1  )(1   2 )(1   4 )(1   8 )............2n factors
 [(1  )(1   2 )][(1   4 )(1   8 )]............ n factors
 [(1  )(1   2 )][(1   3  )(1   6   2 )]............ n factors
 [(1  )(1   2 )][(1   3  )(1  ( 3 )2   2 )]............ n factors
FSC-I - 4.4 - 4
 [(1  )(1   2 )][(1  1  )(1  (1)2   2 )]............ n factors
 [(1  )(1   2 )][(1  )(1   2 )]............ n factors
n n
 (1  )(1   2 )  1     2   3 
  0  1n Q 1    2  0 ,  3  1
 1n  1  R.H.S
{ Question # 4 (i)
Let x 2  x  1  0 ...........(i )
Since  is root of (i ) therefore
 2    1  0...........(ii )
To prove  2 is root of (i )
 2 
2
Consider   2  1   4  2 2  1   2
  2  1   2   2  1    2  1   
2

 0   2  1    from (i )

 2 
2
   2  1  0 ...........(iii )
  2 is the root of the equation (i ) .
Now subtracting (ii ) from (iii )
 2 
2
 2 1  0
2    1  0
  
 4
 0
   3  1  0
 3  1  0 as 0

 3  1

{ Question # 5
Let x be a cube root of 1 then
1
x  (1)3  x 3  1
 x3 1  0  (x )3  (1)3  0
 (x  1)(x 2  x  1)  0
 x 1  0 or x2  x  1  0
(1)  (1)2  4(1)(1)
 x  1 or x
2(1)
1  1 4 1  3
 
2 2
1  3 1  3
 x or x 
2 2
1  3i 1  3i
 x or x 
2 2
1  3i 1  3i
Hence complex cube root of 1 are and .
2 2
FSC-I - 4.4 - 5

{ Question # 6
Since 2 and 2 2 are roots of required equation, therefore
x  2  x  2 2   0
 x 2  2x  2 2x  4 3  0
 x 2  2x (   2 )  4(1)  0 Q 3  1
 x 2  2x (1)  4  0 Q 1    2  0
 x 2  2x  4  0
is the required equation.
{ Question # 7
(i) Let x be a fourth root of 16 then
1
x (16)4  x 4  16

x 2 
2
 x 4  16  0   (4)2  0
 x 2  4x 2  4  0
 x2  4  0 or x2  4  0
 x 2  4 or x2  4
 x   4 or x  4
 x  2i or x  2
Hence the four fourth root of 16 are 2,  2, 2i ,  2i .

(ii) Do yourself as above. Hint: 81  (9)2


(iii) Let x be a fourth root of 625 then
1
x (625)4  x 4  625

x 2 
2
 x 4  625  0   (25)2  0
 x 2  25x 2  25  0
 x 2  25  0 or x 2  25  0
 x 2  25 or x 2  25
 x   25 or x   25
 x  5i or x  5
Hence the four fourth root of 625 are 5,  5, 5i ,  5i .

Question # 8
(i) 2x 4  32  0
 2(x 4  16)  0  x 4  16  0
Now do you as in Question # 7 (i)
(ii) 3y 5  243y  0
 3y y 4  81  0
 3y  0 or y 4  81  0
FSC-I - 4.4 - 6

y 2   (9)2  0
2
 y0 or
 y 2  9y 2  9  0
 y2  9  0 or y2  9  0
 y 2  9 or y2  9
 y   9 or y 9
 y  3i or y  3


Hence S.Set  0, 3,  3i 
(iii) x3  x2  x  1  0
 x 2 x  1  1(x  1)  0
 (x  1)(x 2  1)  0
 x 1  0 or x2 1  0
 x  1 or x 2  1  x  i
Hence S.Set  1, i 
(iv) 5x 5  5x  0
 5x (x 4  1)  0
 5x  0 or x4 1  0

x 2   (1)2  0
2
 x 0 or
 x 2  1x 2  1  0
 x2  1  0 or x2 1  0
 x 2  1 or x2  1
 x  i or x  1


Hence S.Set  0, 1, i 

Error Analyst

Jahanzeb shahid mr.zebi@yahoo.com


MathCity.org Exercise 4.5 (Solutions)
Textbook of Algebra and Trigonometry for Class XI
Merging man and maths Available online at http://www.MathCity.org , Version: 1.0.0
FSc-I- Ex 4.5 - 2 www.MathCity.org
www.MathCity.org FSc-I- Ex 4.5 - 3

To download notes of other exercises visit:


http://www.MathCity.org

MCQs, model papers and old papers are also available.


If you have any question, ask it at
http://forum.mathcity.org
MathCity.org Exercise 4.6 (Solutions)
Textbook of Algebra and Trigonometry for Class XI
Merging man and maths Available online at http://www.MathCity.org , Version: 1.0.0
FSc-I- Ex 4.6 - 2 www.MathCity.org
www.MathCity.org FSc-I- Ex 4.6 - 3
FSc-I- Ex 4.6 - 4 www.MathCity.org

To download notes of other exercises visit:


http://www.MathCity.org

MCQs, model papers and old papers are also


available.
If you have any question, ask it at
http://forum.mathcity.org
mathcity.org Exercise 4.7 (Solutions)
Textbook of Algebra and Trigonometry for Class XI
Merging man and maths Available online @ http://www.mathcity.org, Version: 1.0.0

Nature of Roots (Page 165)  1


The roots of the quadratic equation Here a = 1 , b = −2  m +  , c = 3
 m 
ax 2 + bx + c = 0 Disc. = b − 4ac
2
are 2
  1 
−b ± b 2 − 4ac =  −2  m +   − 4(1)(3)
x =   m 
2a
(Where we take a , b & c as rational)  1 
The nature of the roots of an equation depends = 4  m 2 + 2 + 2  − 12
 m 
on the value of the expression b 2 − 4ac called  1 
discriminant. = 4  m2 + 2 + 2 − 3 
 m 
Case I: If b 2 − 4ac = 0
 1 
b b = 4  m 2 + 2 − 1
Then roots of the equation are − and − .  m 
2a 2a
So the roots are real (rational) and repeated  1 
= 4  m 2 + 2 − 2 + 1
equal.  m 
Case II: If b 2 − 4ac < 0  2

1
Then the roots are complex/imaginary and = 4   m −  + 1 > 0
 m 
distinct/unequal.  
Case III: If b 2 − 4ac > 0 Hence roots are real.
Then the roots are real and distinct/unequal. Question # 2(ii)
However, if b 2 − 4ac is a perfect square then (b − a ) x2 + ( c − a ) x + ( a − b) = 0
b 2 − 4ac will be rational and so the roots are Here A = b − c , B = c − a , C = a − b
rational and unequal. And if b 2 − 4ac is not a Disc. = b 2 − 4ac
perfect square then b 2 − 4ac will be irrational = (c − a)
2
− 4 ( b − c )( a − b )
and so the roots are irrational and unequal.
(
= c 2 + a 2 − 2ca − 4 ab − b 2 − ac + bc )
Question # 1(i)
= c + a − 2ac − 4ab + 4b + 4ac − 4bc
2 2 2
4 x2 + 6 x + 1 = 0
Here a = 4 , b = 6 , c = 1 = (a 2
)
+ c 2 + 2ac − 4ab − 4bc + 4b 2
Disc. = b 2 − 4ac = ( a + c ) − 4b ( a + c ) + ( 2b )
2 2

= ( 6 ) − 4(4)(1) = 36 − 16
2
= ( a + c − 2b )2 > 0
= 20 > 0 Hence roots are real.
Discriminant is not perfect square therefore the
roots are irrational (real) and unequal. Question # 3
(i) ( p + q ) x 2 − px − qb 2 − 4ac = 0
(ii) x2 − 5x + 6 = 0
a = 1 , b = −5 , x = 6 Here a = p + q , b = − p , c = − q
Disc. = b 2 − 4ac Disc. = b 2 − 4ac
= (− p)
2
= (−5)2 − 4(1)(6) − 4( p + q )(− q)
= 25 − 24 = 1 > 0 = p 2 + 4 pq + 4q 2
= ( p + 2q )
Disc. is perfect square therefore roots are 2
rational (real) and unequal.
∴ the roots are rational.
(iii) Do yourself as (i)
(iv) 25 x 2 − 30 x + 9 = 0 (ii) px 2 − ( p − q) x − q = 0
Do yourself
a = 25 , b = −30 , c = 9
Disc. = b 2 − 4ac Question # 4
= (−30)2 − 4(25)(9) (i) ( m + 1) x 2 + 2 ( m + 3) x + m + 8 = 0
= 900 − 900 = 0 a = m + 1 , b = 2 ( m + 3) , c = m + 8
∴ roots are rational (real) and equal.
Disc. = b 2 − 4ac
Question # 2(i) = ( 2 ( m + 3) ) − 4 ( m + 1)( m + 8 )
2

 1
x2 − 2  m +  x + 3 = 0
 m ( ) (
= 4 m 2 + 6m + 9 − 4 m 2 + 8m + m + 8 )
FSc-II / Ex- 4.7 - 2

(
= 4 m 2 + 6m + 9 − m 2 − 8m − m − 8 ) (
⇒ b 2 x 2 + a 2 m2 x 2 + c 2 + 2mcx − a 2b 2 = 0 )
= 4 ( −3m + 1) ⇒ b 2 x 2 + a 2 m 2 x 2 + 2a 2 mcx + a 2 c 2 − a 2b 2 = 0
For equal roots, we have
Disc. = 0
⇒ (b 2
)
+ a 2 m 2 x 2 + 2a 2 mcx + a 2 c 2 − b 2 = 0 ( )
⇒ 4 ( −3m + 1) = 0 Here A = b 2 + a 2 m 2 , B = 2a 2 mc ,
⇒ − 3m + 1 = 0 C = a 2 (c 2 − b 2 )
1 Disc. = B 2 − 4 AC
⇒ 3m = 1 ⇒ m =
3 = (2a 2 mc )2 − 4(b 2 + a 2 m 2 ) ⋅ a 2 (c 2 − b2 )
(ii) & (iii)
Do yourself
(
= 4a 4 m 2c 2 − 4a 2 c 2b2 − b 4 + a 2c 2 m2 − a 2b 2 m 2 )
Question # 5
(
= 4a 2 a 2 m 2 c 2 − c 2 b 2 + b 4 − a 2 c 2 m 2 + a 2b 2 m 2 )
x 2 + ( mx + c ) = a 2
2 = 4a 2 ( −b c 2 2
+ b 4 + a 2 b 2 m2 )
⇒ x 2 + m 2 x 2 + 2mcx + c 2 − a 2 = 0 For equal roots we must have
Disc. = 0
( )
⇒ x 2 1 + m 2 + 2mcx + c 2 − a 2 = 0
Here A = 1 + m , B = 2mc , C = c − a
2 2 2 (
⇒ 4 a 2 b 2 −c 2 + b 2 + a 2 m 2 = 0 )
So Disc. = B 2 − 4 AC ⇒ − c2 + b2 + a2 m2 = 0 Q a ≠ 0, b ≠ 0

(
= ( 2mc ) − 4 1 + m 2 c 2 − a 2
2
)( ) ⇒ c2 = a2 m2 + b2

= 4m 2 c 2 − 4 (c 2
− a 2 + m2c2 − m2 a2 ) Question # 8

(
= 4 m2 c 2 − c 2 + a 2 − m 2 c 2 + m 2 a 2 ) (a 2
) (
− ba x 2 + 2 b2 − ac x + c 2 − ab = 0 )
= 4 ( −c 2
+ a 2 + m2 a 2 ) A = a 2 − bc , B = 2 b 2 − ac , C = c 2 − ab ( )
For equal roots, we have Disc. = B − 4 AC 2

Disc. = 0
( ) ( )( )
2
=  2 b2 − ac  − 4 a 2 − bc c 2 − ab
−c + a + m a = 0
2 2 2 2
 
⇒ c 2 = a 2 + m2 a2 (
= 4 b 4 + a 2 c 2 − 2ab 2 c )
⇒ c = a 1+ m
2 2
( 2
) (
− 4 a c − a 3b + bc3 − ab 2c
2 2
)
as required.
(
= 4 b 4 + a 2 c 2 − 2ab 2 c
Question # 6 − a 2 c 2 + a3b + bc 3 − ab 2 c )
( mx + c ) = 4ax
2

⇒ m x + 2mcx + c − 4ax = 0
2 2 2
(
= 4 a3b + b 4 + bc 3 − 3ab 2c )
⇒ m 2 x 2 + 2 ( mc − 2a ) x + c 2 = 0 (
= 4b a 3 + b3 + c 3 − 3abc )
A = m 2 , B = 2 ( mc − 2a ) , C = c 2 For equal roots, we must have
B 2 − 4 AC = 0
Disc. = B 2 − 4 AC
=  2 ( mc − 2a )  − 4m c
2 2 2 (
⇒ 4b a 3 + b3 + c 3 − 3abc = 0 )
⇒ 4b = 0 or a3 + b3 + c3 − 3abc = 0
(
= 4 m 2 c 2 + 4a 2 − 4amc − m 2c 2 ) ⇒ b = 0 or a3 + b3 + c3 = 3abc
= 4 ( 4a 2
− 4amc )
For equal roots, we must have For updates and news visit
Disc. = 0 http://www.mathcity.org

(
⇒ 4 4a 2 − 4amc = 0 )
⇒ 16a ( a − mc ) = 0 Error Analyst
⇒ a − mc = 0 ⇒ a = mc Waiting for Someone
a a
⇒ = c or c =
m m

Question # 7
x 2 ( mx + c )
2
+ = 1
a2 b2
⇒ b 2 x 2 + a 2 ( mx + c ) = a 2b 2
2
MathCity.org Exercise 4.8 (Solutions)
Textbook of Algebra and Trigonometry for Class XI
Merging man and maths Available online at http://www.MathCity.org , Version: 1.0.0
FSc-I- Ex 4.8 - 2 www.MathCity.org

Available at http://www.MathCity.org/FSc
www.MathCity.org FSc-I- Ex 4.8 - 3

Available at http://www.MathCity.org/FSc
FSc-I- Ex 4.8 - 4 www.MathCity.org

Provide by: Momin Ali


Fazaia Degree College, Peshawar.

Available at http://www.MathCity.org/FSc
MathCity.org Exercise 4.9 (Solutions)
Textbook of Algebra and Trigonometry for Class XI
Merging man and maths Available online at http://www.MathCity.org , Version: 1.0.0
FSc-I- Ex 4.9 - 2 www.MathCity.org

Available at http://www.MathCity.org/FSc
www.MathCity.org FSc-I- Ex 4.9 - 3

Available at http://www.MathCity.org/FSc
FSc-I- Ex 4.9 - 4 www.MathCity.org

Available at http://www.MathCity.org/FSc
www.MathCity.org FSc-I- Ex 4.9 - 5

Available at http://www.MathCity.org/FSc
FSc-I- Ex 4.9 - 6 www.MathCity.org

Provide by: Momin Ali


Fazaia Degree College, Peshawar.

Available at http://www.MathCity.org/FSc
MathCity.org Exercise 4.10 (Solutions)
Textbook of Algebra and Trigonometry for Class XI
Merging man and maths Available online at http://www.MathCity.org , Version: 1.0.0
FSc-I- Ex 4.10 - 2 www.MathCity.org

Available at http://www.MathCity.org/FSc
www.MathCity.org FSc-I- Ex 4.10 - 3

Available at http://www.MathCity.org/FSc
FSc-I- Ex 4.10 - 4 www.MathCity.org

Available at http://www.MathCity.org/FSc
www.MathCity.org FSc-I- Ex 4.10 - 5

Available at http://www.MathCity.org/FSc
FSc-I- Ex 4.10 - 6 www.MathCity.org

Available at http://www.MathCity.org/FSc
www.MathCity.org FSc-I- Ex 4.10 - 7

Provide by: Momin Ali


Fazaia Degree College, Peshawar.

Available at http://www.MathCity.org/FSc
mathcity.org Exercise 5.1 (Solutions)
Textbook of Algebra and Trigonometry for Class XI
Merging man and maths Available online @ http://www.mathcity.org, Version: 1.0.0

1 1
Question # 1 =
x −1 ( x − 1)( x + 1)
2

Resolving it into partial fraction


1 A B
= +
( x − 1)( x + 1) x − 1 x + 1
Multiplying both sides by ( x − 1)( x + 1) we get
1 = A ( x + 1) + B ( x − 1) ................ (i )
Put x − 1 = 0 ⇒ x = 1 in equation (i)
1
1 = A (1 + 1) + B ( 0 ) ⇒ 1 = 2 A + 0 ⇒ A=
2
Now put x + 1 = 0 ⇒ x = −1 in equation (i)
1
1 = A ( 0 ) + B ( −1 − 1) ⇒ 1 = 0 − 2B ⇒ B=−
2
Hence
1 A B
= +
( x − 1)( x + 1) x − 1 x + 1
1 1
= − Answer
2 ( x − 1) 2 ( x + 1)

x 2 ( x 2 + 1) x4 + x2
Question # 2 = x2 + 2
( x + 1)( x − 1) ( x 2 − 1)
x2 − 1 x4 + x2
= x2 + 2 +
2 x4 − x 2
(x 2
− 1)
− +
2x2
= x2 + 2 +
2 2 x2 − 2
( x + 1)( x − 1)
− +
2
2 A B
Now consider = +
( x + 1)( x − 1) x + 1 x − 1
Multiplying both sides by ( x + 1)( x − 1)
2 = A ( x − 1) + B ( x + 1) ................ (i )
Put x + 1 = 0 ⇒ x = −1 in equation (i)
2 = A ( −1 − 1) + B ( 0 ) ⇒ 2 = − 2A + 0 ⇒ A = −1
Now put x − 1 = 0 ⇒ x = 1 in equation (i)
2 = A ( 0 ) + B (1 + 1) ⇒ 2 = 0 + 2B ⇒ B =1
2 −1 1
So = +
( x + 1)( x − 1) x +1 x −1
Hence
(
x2 x2 + 1 ) = x2 + 2 +
−1
+
1
( x + 1)( x − 1) ( x + 1) ( x − 1)
1 1
= x2 + 2 − + Answer
( x + 1) ( x − 1)
FSc I / Ex 5.1-2

2x + 1
Question # 3
( x − 1)( x + 2)( x + 3)
Resolving it into partial fraction
2x + 1 A B C
= + +
( x − 1)( x + 2)( x + 3) x −1 x + 2 x + 3
Multiplying both side by ( x − 1)( x + 2)( x + 3)
2 x + 1 = A ( x + 2)( x + 3) + B ( x − 1) ( x + 3) + C ( x − 1)( x + 2) ............ (i )
Put x − 1 = 0 ⇒ x = 1 in equation (i)
2(1) + 1 = A (1 + 2)(1 + 3) + B(0) + C (0)
3 1
3 = A (3)(4) + 0 + 0 ⇒ 3 = 12 A ⇒ = A ⇒ A=
12 4
Now put x + 2 = 0 ⇒ x = −2 in equation (i)
2(−2) + 1 = A (0) + B(−2 − 1)(−2 + 3) + C (0)
−4 + 1 = 0 + B (−3)(1) + 0 ⇒ − 3 = − 3B ⇒ B =1
Now put x + 3 = 0 ⇒ x = −3 in equation (i)
2(−3) + 1 = A (0) + B (0) + C (−3 − 1)(−3 + 2)
5
−6 + 1 = 0 + 0 + C (−4)(−1) ⇒ − 5 = 4C ⇒ C=−
4
So
2x + 1
1
1 −5
= 4 + + 4
( x − 1)( x + 2)( x + 3) x −1 x + 2 x + 3
1 1 5
= + − Answer
4( x − 1) x + 2 4( x + 3)

3x 2 − 4 x − 5
Question # 4
( x − 2)( x 2 + 7 x + 10) Q x 2 + 7 x + 10 = x 2 + 5 x + 2 x + 10
3x 2 − 4 x − 5 = x( x + 5) + 2( x + 5)
= = ( x + 5)( x + 2)
( x − 2)( x + 5)( x + 2)
Now resolving into partial fraction.
3x2 − 4 x − 5 A B C
= + +
( x − 2)( x + 5)( x + 2) x − 2 x + 5 x + 2
 Do yourself . You will get 
 
 A = − 1 , B = 30 , C = − 5 
 28 7 4 
1
Question # 5
( x − 1)(2 x − 1)(3 x − 1)
Resolving it into partial fraction.
1 A B C
= + +
( x − 1)(2 x − 1)(3 x − 1) x − 1 2 x − 1 3x − 1
Multiplying both side by ( x − 1)(2 x − 1)(3 x − 1) .
1 = A (2 x − 1)(3 x − 1) + B ( x − 1)(3x − 1) + C (2 x − 1)(3 x − 1) ............. (i )
Put x − 1 = 0 ⇒ x = 1 in equation (i)
1 = A (2(1) − 1)(3(1) − 1) + B (0) + C (0) ⇒ 1 = A (1)(2) + 0 + 0
1
⇒ 1 = 2A ⇒ A=
2
FSc I / Ex 5.1-3

1
Put 2 x − 1 = 0 ⇒ 2 x = 1 ⇒ x = in equation (i)
2
1  1   11
1 = A(0) + B  − 1  3   − 1  + C (0) ⇒ 1 = 0 + B  −    + 0
2   2   22
1
⇒1= − B ⇒ B = −4
4
1
Put 3 x − 1 = 0 ⇒ 3 x = 1 ⇒ x = in equation (i)
3
1   1   2  1
1 = A(0) + B (0) + C  − 1   2   − 1 ⇒ 1 = 0 + 0 + C−  − 
3   3   3  3
2 9
⇒1= C ⇒ C=
9 2
1 9
1 2 −4 2
Hence = + +
( x − 1)(2 x − 1)(3 x − 1) x − 1 2 x − 1 3x − 1
1 4 9
= − + Answer
2( x − 1) 2 x − 1 2(3 x − 1)

x
Question # 6
( x − a )( x − b)( x − c)
Resolving it into partial fraction.
x A B C
= + +
( x − a )( x − b)( x − c ) x − a x − b x − c
Multiplying both sides by ( x − a )( x − b)( x − c ) .
x = A ( x − b)( x − c) + B ( x − a )( x − c) + C ( x − a )( x − b) ............ (i )
Put x − a = 0 ⇒ x = a in equation (i)
a = A (a − b)(a − c) + B (0) + C (0)
a
⇒ a = A (a − b)(a − c) + 0 + 0 ⇒ A=
(a − b)(a − c)
Now put x − b = 0 ⇒ x = b in equation (i)
a = A (0) + B (b − a )(b − c ) + C (0)
b
⇒ a = 0 + B (b − a )(b − c ) + 0 ⇒ B=
(b − a )(b − c)
Now put x − c = 0 ⇒ x = c in equation (i)
c = A (0) + B (0) + C (c − a )(c − b)
c
⇒ c = 0 + 0 + C (c − a )(c − b) ⇒ B=
(c − a )(c − b )
So
a b c
x (a − b)(a − c ) (b − a )(b − c) (c − a )(c − b)
= + +
( x − a )( x − b )( x − c) x−a x−b x−c

a b c
= + +
(a − b)(a − c)( x − a ) (b − a )(b − c )( x − b) (c − a )(c − b)( x − c)
Answer

Made By Atiq ur Rehman ( atiq@mathcity.org ) URL: http://www.mathcity.org


FSc I / Ex 5.1-4

6 x3 + 5 x2 − 7 3x + 4
Question # 7
2x2 − x − 1 2 x − x − 1 6 x3 + 5x2 − 7
2

7x − 3 6 x3 − 3 x 2 − 3 x
= 3x + 4 + − + +
2x2 − x − 1
8 x + 3x − 7
2
7x − 3 7x − 3
= 3x + 4 + 2 = 3x + 4 + 8x2 − 4 x − 4
2 x − 2x + x − 1 2 x ( x − 1) + 1( x − 1) − + +

7x − 3
7x − 3
= 3x + 4 +
( x − 1)(2 x + 1)
Now Consider
7x − 3 A B
= +
( x − 1)(2 x + 1) x − 1 2 x + 1
 Find value of A & B yourself 
 13 
You will get A = 3 and B = 3 
4

4 13
7x − 3 3 3 = 4 13
so = + +
( x − 1)(2 x + 1) x − 1 2x + 1 3( x − 1) 3(2 x + 1)
Hence
6 x3 + 5 x 2 − 7 4 13
= 3 x + 4 + + Answer
2x2 − x − 1 3( x − 1) 3(2 x + 1)

2 x3 + x 2 − 5 x + 3
Question # 8 1
2 x3 + x2 − 3x
−2 x + 3 2 x 3
+ x 2
− 3 x 2 x 3
+ x 2
− 5x + 3
=1 + 3 2 x + x − 3x
3 2
2 x + x 2 − 3x − − +
−2 x + 3 −2 x + 3 − 2x + 3
=1 + = 1 +
x(2 x 2 + x − 3) x(2 x 2 + 3 x − 2 x − 3)
−2 x + 3 −2 x + 3
=1 + =1 +
x ( x(2 x + 3) − 1(2 x + 3) ) x(2 x + 3)( x − 1)
Now consider
3 − 2x A B C
= + +
x(2 x + 3)( x − 1) x 2 x + 3 x − 1
⇒ 3 − 2 x = A(2 x + 3)( x − 1) + Bx ( x − 1) + C x (2 x + 3) ............... (i )
Put x = 0 in equation (i)
3 − 2(0) = A ( 2(0) + 3)( (0) − 1) + B (0) + C (0) ⇒ 3 − 0 = A ( 0 + 3)( −1) + 0 + 0
⇒ 3 = − 3A ⇒ A = −1
3
Now put 2 x + 3 = 0 ⇒ 2 x = −3 ⇒ x = − in equation (i)
2
 3  3 3   3 5
3 − 2  −  = A(0) + B  −   − − 1  + C (0) ⇒ 3 + 3 =0+ B −   − + 0
 2  2 2   2 2
15  4 8
⇒ 6 = B ⇒ B = ( 6)  ⇒ B =
4  15  5
Now put x − 1 = 0 ⇒ x = 1 in equation (i)

3 − 2(1) = A ( 0 ) + B ( 0 ) + C (1)( 2(1) + 3) ⇒ 1 = 0 + 0 + 5C


1
⇒ C=
5
FSc I / Ex 5.1-5

8 1
3 − 2x −1 5 1 8 1
So = + + 5 =− + +
x(2 x + 3)( x − 1) x 2 x + 3 x − 1 x 5(2 x + 3) 5( x − 1)
2 x3 + x 2 − 5 x + 3 1 8 1
Hence =1− + + Answer
2 x + x − 3x
3 2
x 5(2 x + 3) 5( x − 1)

( x − 1)( x − 3)( x − 5)
Question # 9
( x − 2)( x − 4)( x − 6)
( x − 1)( x 2 − 3 x − 5 x + 15)
=
( x − 2)( x 2 − 4 x − 6 x + 24) 1
( x − 1)( x − 8 x + 15)
2
x 3 − 12 x 2 + 44 x − 48 x 3 − 9 x 2 + 23 x − 15
=
( x − 2)( x 2 − 10 x + 24)
x3 −12 x 2 + 44 x − 48
x 3 − 8 x 2 + 15 x − x 2 + 8 x − 15 − + − +
= 3 x − 21x + 33
2
x3 − 10 x 2 + 24 x − 2 x 2 + 20 x − 48
x 3 − 9 x 2 + 23 x − 15
= 3
x − 12 x 2 + 44 x − 48
3 x 2 − 21x + 33 3 x 2 − 21x + 33
= 1+ 3 = 1 +
x − 12 x 2 + 44 x − 48 ( x − 2)( x − 4)( x − 6)
Now Suppose
3 x 2 − 21x + 33 A B C
= + +
( x − 2)( x − 4)( x − 6) x − 2 x − 4 x − 6
 Find value of A, B and C yourself 
 
 You will get A = 3 8 , B = 3 4 , C = 15 8 
3 3 15
3 x 2 − 21x + 33
So = 8 + 4 + 8
( x − 2)( x − 4)( x − 6) x − 2 x − 4 x − 6
3 3 15
= + +
8( x − 2) 4( x − 4) 8( x − 6)
Hence
( x − 1)( x − 3)( x − 5) 3 3 15
= 1+ + + Answer
( x − 2)( x − 4)( x − 6) 8( x − 2) 4( x − 4) 8( x − 6)

1
Question # 10
(1 − ax)(1 − bx )(1 − cx)
Resolving it into partial fraction.
1 A B C
= + +
(1 − ax)(1 − bx)(1 − cx ) 1 − ax 1 − bx 1 − cx
Multiplying both sides by (1 − ax)(1 − bx)(1 − cx) .
1= A (1 − bx )(1 − cx) + B (1 − ax)(1 − cx) + C (1 − ax)(1 − bx) ............ (i )
1
Put 1 − ax = 0 ⇒ ax = 1 ⇒ x = in equation (i).
a
 1  1  b  c 
1= A  1 − b ⋅ 1 − c ⋅  + B (0) + C (0) ⇒ 1= A  1 − 1 −  + 0 + 0
 a  a  a  a 

⇒ 1= A 
 a − b  a − c 
⇒ =
( a − b )( a − c ) ⇒ A = a2
  1 A
 a  a  a2 ( a − b )( a − c )
FSc I / Ex 5.1-6

 Find value of B & C yourself as A. 


 2 2 
You will get B = b
,C=
c 
 (b − a ) (b − c ) (c − a )(c − b) 
2 2 2
a b c
1 (a − b ) ( a − c ) (b − a ) (b − c ) (c − a )(c − b )
Hence = + +
(1 − ax)(1 − bx)(1 − cx ) 1 − ax 1 − bx 1 − cx
a2 b2 c2
= + +
(a − b)(a − c)(1 − ax) (b − a )(b − c )(1 − bx) (c − a )(c − b)(1 − cx)
Answer

x2 + a 2
Question # 11
( x 2 + b 2 )( x 2 + c 2 )( x 2 + d 2 )
Put y = x 2 in above.
y + a2
( y + b 2 )( y + c 2 )( y + d 2 )
Now consider
y + a2 A B C
= + +
( y + b )( y + c )( y + d ) y + b
2 2 2 2
y+c 2
y + d2
⇒ y + a 2 = A ( y + c 2 )( y + d 2 ) + B ( y + b2 )( y + d 2 ) + C ( y + b2 )( y + c 2 ) ........... (i )
Put y + b2 = 0 ⇒ y = −b 2 in equation (i)
−b 2 + a 2 = A (−b2 + c 2 )(−b2 + d 2 ) + B (0) + C (0)
a 2 − b2
⇒ a − b = A (c − b )(d − b ) + 0 + 0
2 2 2 2 2 2
⇒ A= 2
(c − b 2 )(d 2 − b 2 )
Now put y + c 2 = 0 ⇒ y = −c 2 in equation (i)
−c 2 + a 2 = A (0) + B (−c 2 + b 2 )(−b 2 + d 2 ) + C (0)
a 2 − c2
⇒ a 2 − c 2 = 0 + B (b2 − c 2 )(d 2 − c 2 ) + 0 ⇒ B=
(b 2 − c 2 )(d 2 − c 2 )
Now put y + d 2 = 0 ⇒ y = −d 2 in equation (i)
−d 2 + a 2 = A (0) + B (0) + C (−d 2 + b 2 )(−d 2 + c 2 )
a2 − d 2
⇒ a − d = 0 + 0 + C (b − d )(c − d )
2 2 2 2 2 2
⇒ C= 2
(b − d 2 )(c 2 − d 2 )
Hence
a −b a −c a −d
2 2 2 2 2 2

y + a2 (c − b ) (d − b )
2 2 2 2
(b − c ) ( d − c )
2 2 2 2
(b − d ) ( c − d )
2 2 2 2

= + +
( y + b 2 )( y + c 2 )( y + d 2 ) y + b2 y + c2 y + d2
a 2 − b2 a 2 − c2 a2 − d 2
= 2 + +
(c − b 2 )(d 2 − b 2 )( y + b 2 ) (b 2 − c 2 )(d 2 − c 2 )( y + c 2 ) (b 2 − d 2 )(c 2 − d 2 )( y + d 2 )
Since y = x 2
a 2 − b2 a 2 − c2 a2 − d 2
= 2 + +
(c − b 2 )(d 2 − b 2 )( x 2 + b 2 ) (b 2 − c 2 )(d 2 − c 2 )( x 2 + c 2 ) (b 2 − d 2 )(c 2 − d 2 )( x 2 + d 2 )
Answer

Made By Atiq ur Rehman ( atiq@mathcity.org ) URL: http://www.mathcity.org


FSC-I / Ex 5.3 - 1

mathcity.org Exercise 5.3 (Solutions)


Textbook of Algebra and Trigonometry for Class XI
Merging man and maths Available online @ http://www.mathcity.org, Version: 1.0.0

9x − 7
Question # 1
( x 2 + 1)( x + 3)
Resolving it into partial fraction.
9x − 7 Ax + B C
= 2 +
( x + 1)( x + 3) ( x + 1) ( x + 3)
2

Multiplying both sides by ( x 2 + 1)( x + 3) .


9 x − 7 = ( Ax + B)( x + 3) + C ( x2 + 1) ............ (i )
Put x + 3 = 0 ⇒ x = −3 in equation (i).
9(−3) − 7 = ( A(−3) + B )( 0 ) + C ( (−3) 2 + 1) ⇒ − 27 − 7 = 0 + C ( 9 + 1)
34 17
⇒ − 34 = 10C ⇒ C =− ⇒ C =−
10 5
Now equation (i) can be written as
9 x − 7 = A ( x 2 + 3x ) + B ( x + 3) + C ( x 2 + 1)
Comparing the coefficients of x 2 , x and x 0 .
0 = A + C …………….…... (ii)
9 = 3A + B ……………..… (iii)
−7 = + 3B + C …………...…. (iv)
Putting value of C in equation (ii)
17 17
0= A− ⇒ A =
5 5
Now putting value of A in equation (iii)
 17  51 51 6
9 = 3  + B ⇒ 9= +B ⇒ 9 − = B ⇒ B=−
 5 5 5 5
Hence
17 6 17
x− −
9x − 7
= 52 5 + 5
( x + 1)( x + 3)
2
x +1 ( x + 3)
17 x − 6 17
17 x − 6 17
= 5 − 5 = − Answer
x2 + 1 ( x + 3) 5( x 2 + 1) 5( x + 3)

1
Question # 2
( x + 1)( x + 1)
2

Now Consider
1 Ax + B C
= 2 +
( x + 1)( x + 1)
2
x +1 x +1
Multiplying both sides by ( x 2 + 1)( x + 1) .
1 = ( Ax + B)( x + 1) + C ( x 2 + 1)................ (i )
Put x + 1 = 0 ⇒ x = −1 in equation (i)

1 = 0 + C ( (−1) 2 + 1)
1
⇒ 1 = 2C ⇒ C =
2
Now eq. (i) can be written as
FSC-I / Ex 5.3 - 2

1 = A( x 2 + x) + B ( x + 1) + C ( x 2 + 1)
Comparing the coefficients of x 2 , x and x 0 .
0 = A + C …………….…. (ii)
0 = A + B ………………. (iii)
1 = A + C ………………. (iv)
Putting value of C in equation (ii)
1 1
0= A+ ⇒ A = −
2 2
Putting value of A in equation (iii)
1 1
0=− +B ⇒ B=
2 2
1 1 1 −x + 1 1
− x+
1
Hence = 22 2 + 2 = 2 + 2
( x + 1)( x + 1)
2
x +1 x +1 x +1 x +1 2

−x +1 1 1− x 1
= + = + Answer
2( x 2 + 1) 2( x + 1) 2( x 2 + 1) 2( x + 1)

3x + 7
Question # 3
( x + 4)( x + 3)
2

Resolving it into partial fraction.


3x + 7 Ax + B C
= +
( x 2 + 4)( x + 3) x 2 + 4 x+3
 Now do yourself , you will get 
 
 A = 2 13 , B = 13 and C = − 2 13
33


x 2 + 15
Question # 4
( x 2 + 2 x + 5)( x − 1)
Resolving it into partial fraction.
x 2 + 15 Ax + B C
= 2 +
( x + 2 x + 5)( x − 1) x + 2 x + 5 x − 1
2

⇒ x 2 + 15 = ( Ax + B)( x − 1) + C ( x 2 + 2 x + 5).............. (i )
Put x − 1 = 0 ⇒ x = 1 in equation (i)
(1)2 + 15 = ( A(1) + B ) (0) + C ( (1)2 + 2(1) + 5 ) ⇒ 1 + 15 = 0 + C (1 + 2 + 5)
16
⇒ 16 = 8C ⇒ =C ⇒ C = 2
8
Now equation (i) can be written as
x 2 + 15 = A ( x 2 − x ) + B ( x − 1) + C ( x 2 + 2 x + 5)
Comparing the coefficients of x 2 , x and x 0 .
1 = A + C …………..………. (ii)
0 = − A + B + 2C ………….. (iii)
15 = − B + 5C ………….….. (iv)
Putting value of C in equation (ii).
1 = A + 2 ⇒ 1− 2 = A ⇒ A = − 1
Putting value of A and C in equation (iii)
FSC-I / Ex 5.3 - 3

0 = − (−1) + B + 2(2) ⇒ 0 =1+ B + 4 ⇒ 0 = B + 5 ⇒ B =−5


x 2 + 15 (−1) x − 5 2
Hence = 2 +
( x + 2 x + 5)( x − 1) x + 2 x + 5 x − 1
2

−x − 5 2
= 2 + Answer
x + 2x + 5 x −1

x2
Question # 5
( x 2 + 4)( x + 2)
Resolving it into partial fraction.
x2 Ax + B C
= 2 +
( x + 4)( x + 2)
2
x +4 x+2
 Now do yourself , you will get 
 1 , B = − 1 and C = − 1 
=
 2 
A
2

x2 + 1 x2 + 1
Question # 6 = Q x 3 + 1 = ( x + 1)( x 2 − x + 1)
x3 + 1 ( x + 1)( x 2 − x + 1)
Now consider
x2 + 1 A Bx + C
= +
( x + 1)( x 2 − x + 1) x + 1 x 2 − x + 1
 Now do yourself , you will get 
 
 A = 3 , B = 3 and C = 3
2 1 1


x2 + 2x + 2
Question # 7
( x 2 + 3)( x + 1)( x − 1)
Consider
x2 + 2 x + 2 Ax + B C D
= 2 + +
( x + 3)( x + 1)( x − 1)
2
x +3 x +1 x −1
⇒ x 2 + 2 x + 2 = ( Ax + B)( x + 1)( x − 1) + C ( x 2 + 3)( x −1) + D ( x 2 + 3)( x + 1)............. (i )
Put x + 1 = 0 ⇒ x = −1 in equation (i)
(−1)2 + 2(−1) + 2 = 0 + C ( (−1)2 + 3 ) ( (−1) −1) + 0 ⇒ 1 − 2 + 2 = C ( 4 )( −2 )
1
⇒ 1 = − 8C ⇒ C=−
8
Now put x − 1 = 0 ⇒ x = 1 in equation (i)
⇒ (1)2 + 2(1) + 2 = 0 + 0 + D ( (1)2 + 3) ( (1) + 1) ⇒ 1 + 2 + 2 = D ( 4 )( 2 )
5
⇒ 5= 8D ⇒ D=
8
Equation (i) can be written as
x 2 + 2 x + 2 = ( Ax + B)( x 2 − 1) + C ( x 3 − x 2 + 3x − 3) + D ( x 3 + x 2 + 3x + 3)
⇒ x 2 + 2 x + 2 = A( x 3 − x) + B ( x 2 − 1) + C ( x3 − x 2 + 3x − 3) + D ( x 3 + x 2 + 3x + 3)
Comparing the coefficients of x 3 , x 2 , x and x 0 .
0 = A + C + D ……………...…. (ii)
1= B − C + D ……………….…. (iii)
2 = − A + 3C + 3D ……………. (iv)
FSC-I / Ex 5.3 - 4

2 = − B − 3C + 3D ………….…. (v)
Putting values of C and D in (ii)
1 5 1 1
0= A− + ⇒ 0= A+ ⇒ A=−
8 8 2 2
Putting values of C and D in (iii)
 1 5 1 5 3
1= B −−  + ⇒ 1= B + + ⇒ 1= B +
 8 8 8 8 4
3 1
⇒ 1− =B ⇒ B=
4 4
Hence
1 1 1 5
− x+ −
x + 2x + 2
2
= 22 4 + 8 + 8
( x + 3)( x + 1)( x − 1)
2
x +3 x +1 x −1
−2 x + 1 1 5

−2 x + 1 −1 5
= 4 + 8 + 8 = + +
x2 + 3 x +1 x −1 4( x 2 + 3) 8( x + 1) 8( x − 1)
1 − 2x 1 5
= − + Answer
4( x + 3)
2
8( x + 1) 8( x − 1)

1
Question # 8
( x − 1)2 ( x 2 + 2)
Resolving it into partial fraction.
1 A B Cx+D
= + + 2
( x − 1) ( x + 2) x − 1 ( x − 1)
2 2 2
x +2
⇒ 1 = A ( x − 1)( x 2 + 2) + B ( x 2 + 2) + (C x + D)( x − 1)2 ............. (i )
Put x − 1 = 0 ⇒ x = 1 in equation (i)

1 = 0 + B ( (1) 2 + 2 ) + 0
1
⇒ 1 = 3B ⇒ B=
3
Now equation (i) can be written as
1 = A ( x 3 − x 2 + 2 x − 2) + B ( x 2 + 2) + (C x + D)( x 2 − 2 x + 1)
⇒ 1 = A ( x3 − x 2 + 2 x − 2) + B ( x 2 + 2) + C ( x3 − 2 x 2 + x ) + D ( x 2 − 2 x + 1)
Comparing the coefficients of x 3 , x 2 , x and x 0 .
0 = A + C ……………………. (ii)
0 = − A + B − 2C + D …..…….. (iii)
0 = 2 A + C − 2 D …………..… (iv)
1 = −2 A + 2 B + D ……………. (v)
Multiplying eq. (iii) by 2 and adding in (iv)
0 = −2 A + 2 B − 4C + 2 D
0 = 2A + C − 2D
0= 2 B − 3C
Putting value of B in above
1 2 2 2
0 = 2   − 3C ⇒ 0 = − 3C ⇒ 3C = ⇒ C=
 3 3 3 9
Putting value of C in eq. (ii)
2 2
0= A+ ⇒ A=−
9 9
FSC-I / Ex 5.3 - 5

Putting value of A and B in eq. (v)


 2 1 4 2 4 2 1
1 = −2  −  + 2   + D ⇒ 1 = + + D ⇒ 1 − − = D ⇒ D=−
 9 3 9 3 9 3 9
Hence
1
=
−2
9 +
1
3
2 x+ − 1
+ 9 2 9 ( ) ( )
( x − 1) ( x + 2) x − 1 ( x − 1)
2 2 2
x +2
2x − 1
−2 1
−2 1 2x − 1
= 9+ 3 + 9 = + +
x − 1 ( x − 1) 2
x +2
2
9( x − 1) 3( x − 1) 2
9( x 2 + 2)

x4
Question # 9 −1
1 − x4
1 1 1 − x 4 x4
= −1 + = − 1 +
1 − x4 (1 − x 2 )(1 + x 2 ) x 4 −1
− +
1 1
= −1 +
(1 − x)(1 + x)(1 + x 2 )

Now consider
1 A B Cx + D
= + +
(1 − x)(1 + x )(1 + x ) 1 − x 1 + x 1 + x 2
2

 Now find values of A, B, C and D yourself . 


 
 You will get A = 4 , B = 4 , C = 0 and D = 2
1 1 1

So
1
1 1 (0) x + 12
= 4 + 4 +
(1 − x)(1 + x )(1 + x 2 ) 1 − x 1 + x 1 + x2
1 1 1
= + +
4(1 − x) 4(1 + x) 2(1 + x 2 )
Hence
x4 1 1 1
= −1 + + + Answer
1− x 4
4(1 − x) 4(1 + x ) 2(1 + x 2 )

x2 − 2 x + 3
Question # 10 Q x 4 + x 2 + 1 = x4 + 2 x 2 + 1 − x 2
x 4 + x2 + 1
= ( x 2 + 1) 2 − x 2
x − 2x + 3
2
= ( x 2 + 1 + x )( x 2 + 1 − x)
= 2
( x + x + 1)( x − x + 1)
2
= ( x 2 + x + 1)( x 2 − x + 1)
Now Consider
x2 − 2 x + 3 Ax + B Cx+D
= +
( x 2 + x + 1)( x 2 − x + 1) x 2 + x + 1 x 2 − x + 1
⇒ x 2 − 2 x + 3 = ( Ax + B)( x 2 − x + 1) + (Cx + D)( x 2 + x + 1).............. (i )
⇒ x 2 − 2 x + 3 = A( x 3 − x 2 + x ) + B ( x 2 − x + 1) + C ( x3 + x 2 + x) + D( x 2 + x + 1)
Comparing the coefficients of x 3 , x 2 , x and x 0 .
0 = A + C ……………………..… (ii)
1 = − A + B + C + D ……………. (iii)
−2 = A − B + C + D …………...… (iv)
FSC-I / Ex 5.3 - 6

3 = B + D …………………….... (v)
Subtracting (ii) and (iv)
0= A +C
− 2 = A− B + C + D
+ − + − −
2= B −D
⇒ 2 = B − D …………… (vi)
Adding (v) and (vi)
3= B+ D
2=B−D
5 = 2B
5
⇒ B=
2
Putting value of B in (v)
5 5 1
3= + D ⇒3− = D ⇒ D=
2 2 2
Putting value of B and D in (iii)
5 1 5 1
1= − A + + C + ⇒ 1− − =− A + C
2 2 2 2
⇒ − 2 = − A + C ……………. (vii)
Adding (ii) and (vii)
0= A+C
−2 = − A + C
−2 = 2C ⇒ C = −1
Putting value of C in equation (ii)
0 = A −1 ⇒ A=1
Hence
5 1
(1) x + (−1) x +
x − 2x + 3
2
= 2 2 + 2
( x + x + 1)( x − x + 1) x + x + 1 x − x + 1
2 2 2

2x + 5 −2 x + 1

= 2 + 2
x + x +1
2
x − x +1
2

2x + 5 −2 x + 1
= +
2( x 2 + x + 1) 2( x 2 − x + 1)
2x + 5 1 − 2x
= + Answer
2( x + x + 1)
2
2( x 2 − x + 1)

For updates and news visit


http://www.mathcity.org

Error Analyst

Waiting for Someone


mathcity.org Exercise 6.1 (Solutions)
Textbook of Algebra and Trigonometry for Class XI
Merging man and maths Available online @ http://www.mathcity.org, Version: 1.0.0

Question # 1 (i) an = 2n − 3
Put n = 1
a1 = 2(1) − 3 ⇒ a1 = 2 − 3 = − 1
Put n = 2
a2 = 2(2) − 3 ⇒ a2 = 4 − 3 = 1
Put n = 3
a3 = 2(3) − 3 ⇒ a3 = 6 − 3 = 3
Put n = 4
a4 = 2(4) − 3 ⇒ a4 = 8 − 3 = 5
Hence -1, 1, 3 , 5 are the first four term of the sequence.

(ii) an = (−1)n n 2
Put n = 1
a1 = (−1)1 (1)2 ⇒ a1 = (−1) (1) = − 1
Put n = 2
a2 = (−1)2 (2)2 ⇒ a2 = (1) (4) = 4
Put n = 3
a3 = (−1)3 (3) 2 ⇒ a3 = (−1) (9) = − 9
Put n = 4
a4 = (−1)4 (4)2 ⇒ a4 = (1) (16) = 16
Hence -1, 4, -9, 16 are the first four terms of the sequence.
(iii), (iv), (v) and (vi) Do yourself as above.
(vii) an − an −1 = n + 2 , a1 = 2
Put n = 2
a2 − a2−1 = 2 + 2 ⇒ a2 − a1 = 4 ⇒ a2 = 4 + a1 = 4 + 2 = 6 Q a1 = 2
Put n = 3
a3 − a3−1 = 3 + 2 ⇒ a3 − a2 = 5 ⇒ a3 = 5 + a2 = 5 + 6 = 11 Q a2 = 6
Put n = 4
a4 − a4−1 = 4 + 2 ⇒ a4 − a3 = 6 ⇒ a4 = 6 + a3 = 6 + 11 = 17 Q a3 =11
Hence 2, 6, 11, 17 are the first four terms of the sequence.
(viii) an = nan−1 , a1 =1
Put n = 2
a2 = (2) a2−1 ⇒ a2 = 2 a1 = 2(1) = 2 Q a1 =1
Put n = 3
a3 = (3) a3−1 ⇒ a3 = 3 a2 = 3(2) = 6 Q a2 = 2
Put n = 4
a4 = (4)a4−1 ⇒ a4 = 4 a3 = 4(6) = 24 Q a3 = 6
Hence 1, 2, 6, 24 are the first four terms of the sequence.
(ix) Same as above
1
(x) an =
a + (n − 1)d
Put n = 1
1 1 1 1
a1 = ⇒ a1 = = =
a + (1 − 1)d a + (0)d a + 0 a
Put n = 2
1 1 1
a2 = ⇒ a2 = =
a + (2 − 1)d a + (1)d a + d
Put n = 3
FSc-I / Ex 6.1 - 2

1 1 1
a3 = ⇒ a3 = =
a + (3 − 1)d a + (2)d a + 2d
Put n = 4
1 1 1
a4 = ⇒ a4 = =
a + (4 − 1)d a + (3)d a + 3d
1 1 1 1
Hence , , , are the first four terms of the sequence.
a a + d a + 2 d a + 3d

Question # 2 (i) 2,6,11,17,..............a7


We see that the successive difference of the given terms are 4, 5, 6 and conclude that
sequence of the differences is 4, 5, 6, 7, 8, 9,……….
So a5 =17 + 7 = 24 , a6 = 24 + 8 = 32 and a7 = 32 + 9 = 41
Thus the required term is a7 = 41
(ii) 1,3,12,60,................a6
We see that the successive multiplying factor are 3, 4, 5 and conclude that the sequence of
multiplying factors is 3, 4, 5, 6, 7, 8, 9………….
So a5 = 60 × 6 = 360 , a6 = 360 × 7 = 2520
Thus the required term is a6 = 2520
3 5 7
(iii) 1, , , ,............. a7
2 4 8
The successive terms in numerator are 1, 3, 5, 7, ………….. , which are the consecutive odd
numbers and next terms are 9, 11, 13.
And the successive terms in denominators are 1, 2, 4, 8, …………. with common ratio 2, so
the next terms are 16, 32, 64.
13
Thus the required term is a7 =
64
(iv) 1,1, − 3,5, − 7,9, .................a8 *Correction
We see that the common difference of odd terms is -4, so a7 = − 7 + ( −4) = −11
And the common difference of even terms is 4, so a8 = 9 + 4 =13
Thus the required term is a8 =13
(v) 1, − 3,5, − 7,9, − 11,...............a8
We see that the common difference of odd terms is 4, so a7 = 9 + 4 =13 .
And the common difference of the even terms is -4, so a8 = − 11 + ( −4) = − 15
Thus the required term is a8 = − 15

Question # 3 (i) 7,9,12,16,.................


We see that the sequence of the successive difference is 2, 3, 4, ….….. so the next two
differences are 5 and 6.
Thus the next two terms are 16 + 5 = 21 and 21 + 6 = 27.
(ii) 1,3,7,15,31,................
We see that the sequence of the successive difference is 2, 4, 8, 16, ……… so the next two
differences are 32 and 64.
Thus the next two terms of the sequence are 31 + 32 = 63 and 63 + 64 = 127.
(iii) −1,2,12,40,.............
The sequence of the above terms can be written as
−1 × 1, 1 × 2, 3 × 4, 5 × 8,................
So the next two terms are 7 × 16 = 112 and 9 × 32 = 288 .
(iv) 1, − 3,5, − 7,9, − 11,...............
We see that the common difference of odd terms is 4, so a7 = 9 + 4 =13 .
And the common difference of the even terms is -4, so a8 = − 11 + ( −4) = − 15
Thus the next two terms are 13 and -15.
Made By Atiq ur Rehman (atiq@mathcity.org ). http://www.mathcity.org
Exercise 6.4 (Solutions)
mathcity.org Textbook of Algebra and
an d Trigonometry for Class XI
Merging man and maths Available online @ http://www.mathcity.org, Version: 1.0.0
FSc-I- Ex 6.4 - 2 www.mathcity.org
www.mathcity.org FSc-I- Ex 6.4 - 3
FSc-I- Ex 6.4 - 4 www.mathcity.org
www.mathcity.org FSc-I- Ex 6.4 - 5
FSc-I- Ex 6.4 - 6 www.mathcity.org
www.mathcity.org FSc-I- Ex 6.4 - 7
FSc-I- Ex 6.4 - 8 www.mathcity.org

******************************************
Available online at http://www.mathcity.org
******************************************
mathcity.org Exercise 6.5 (Solutions)
Textbook of Algebra and Trigonometry for Class XI
Merging man and maths Available online @ http://www.mathcity.org, Version: 1.0.0

Question # 1
The sequence of the deposits is
10, 15, 20, ……………. to 9 terms
Here a1 = 10 , d = 15 − 10 = 5 , n = 9
n
Since S n = [ 2a1 + (n − 1)d ]
2
9 9 9
⇒ S9 = [ 2(10) + (9 − 1)(5)] = [ 20 + 40] = (60) = 270
2 2 2
Hence the total amount he deposits is Rs. 270.

Question # 2
The sequence of the trees from top to base row is
1, 2, 3, ……………
Let n be the total number of tress in base row then
a1 = 1 , d = 2 − 1 = 1 , n = n , S n = 378
n
Now S n = [ 2a1 + (n − 1)d ]
2
⇒ 378 = [ 2(1) + (n − 1)(1) ] ⇒ 756 = n [ 2 + n − 1]
n
2
⇒ 756 = n (n + 1) ⇒ 756 = n2 + n
⇒ n 2 + n − 756 = 0
−1± (−1)2 − 4(1)(−756)
So n=
2(1)
−1 ± 1 + 3024 −1 ± 3025 −1 ± 55
⇒ n= = =
2 2 2
−1 + 55 54 −1 − 55 −56
So n= = = 27 or n= = = − 28
2 2 2 2
Since n can never be negative therefore n = 27
Now an = a1 + (n − 1)d
⇒ a27 = (1) + (27 − 1)(1) = 1 + 26 = 27
Thus the numbers of trees in the base row are 27.

Question # 3
Let the first installment be x then the sequence of installment will be
x , x − 10, x − 20,......................
Here a1 = x , d = -10 , n = 14 and S n = 1100 + 230 = 1330
n
Now S n = [ 2a1 + (n − 1)d ]
2
⇒ 1330 = [ 2 x + (14 − 1)(−10)] ⇒ 1330 = 7 [ 2 x − 130] ⇒ 1330 = 14 x − 910
14
2
2240
⇒ 1330 + 910 = 14x ⇒ 2240 = 14x ⇒ x = = 160
14
Hence the first installment is 160.
FSc-I / Ex 6.5 - 2

Question # 4
The sequence of the strikes is
1, 2, 3, …………., 12
Here a1 = 1 , d = 2 – 1 = 2 , n = 12 , an = 12
n
Now S n = [ 2a1 + (n − 1)d ]
2
12 12 12
⇒ S n = [ 2(1) + (12 − 1)(1)] = [ 2 + 11] = [13] = 78
2 2 2
Hence clock strikes 78 hours in twelve strikes.

Question # 5
The sequence of the savings is
12, 16, 20, ……………
Total Savings = 2100
So here a1 = 12 , d = 16 – 12 = 4 , S n = 2100 , n = ?
n
Since S n = [ 2a1 + (n − 1)d ]
2
⇒ 2100 = [ 2(12) + (n − 1)(4)] ⇒ 4200 = n [ 24 + 4n − 4]
n
2
⇒ 4200 = n [ 4n + 20] ⇒ 4200 = 4n2 + 20n
⇒ 4n2 + 20n − 4200 = 0 ⇒ 4(n2 + 5n − 1050) = 0
⇒ n2 + 5n − 1050 = 0
−5 ± (5) 2 − 4(1)(−1050) −5 ± 25 + 4200
⇒ n= =
2(1) 2
−5 ± 4225 −5 ± 65
= =
2 2
−5 − 65 −70 −5 + 65 60
So n = = = −35 or n= = = 30
2 2 2 2
As n can never be negative therefore n = 30
Thus student will save Rs. 2100 in 30 weeks.

Question # 6
The sequence of the falls is
9, 27, 45, ………………
(i) a1 = 9 , d = 27 – 9 = 18 , a5 = ?
Since a5 = a1 + 4d = 9 + 4(18) = 9 + 72 = 81
Hence in fifth second the object will fall 81 meters.
(ii) Here a1 = 9 , d = 27 – 9 = 18 , n = 5, S5 = ?
n
Since S n =[ 2a1 + (n − 1)d ]
2
5 5 5
⇒ S5 = [ 2(9) + (5 − 1)(18) ] = [18 + 72 ] = (90) = 225
2 2 2
th
Thus up to 5 second the object will fall 225 meters.
FSc-I / Ex 6.5 - 3

Question # 7
Here a1 = 6000 , a11 = 12000 , n = 11
n
Now S n = [ a1 + an ]
2
11 11
⇒ S11 = [ 6000 + 12000 ] = (18000) = 99000
2 2
Hence he will receive Rs. 99000 in past eleven years.

Question # 8
Since the sum of angels of 3 sided polygon (triangle) = a1 = π
Sum of angels of 4 sided polygon (quadrilateral) = a2 = 2π
Sum of the angels of 5 sided polygon (pentagon) = a3 = 3π
So
The sum of interior angels of 16 side polygon = a14 = ?
Here a1 = π , d = a2 − a1 = 2π − π = π , n = 14
Since an = a1 + (n − 1) d
⇒ a14 = π + (14 − 1)(π ) = π + 13π = 14π
Hence sum of interior angels of 16 side polygon is 14π .

Question # 9
Let a1 denotes the prize money for the last position
Then a1 = 4000 , S n = 60000 , n = 8 , an = ?
n
Since S n = ( a1 + an )
2
8
⇒ 60000 = ( 4000 + an ) ⇒ 60000 = 4 ( 4000 + an )
2
⇒ 60000 = 16000 + 4an ⇒ 60000 − 16000 = 4an ⇒ 44000 = 4an
44000
⇒ an = = 11000
4
Hence the team at 1st place will get 11000 Rs.

Question # 10
Balls in the first layer = 8 + 7 + 6 +……… + 2 + 1
8
= [ 2(8) + (8 − 1)(−1)] = 4(16 − 7) = 36
2
Balls in the second layer = 7 + 6 + 5 + ………+ 2 + 1
7 7 7
= [ 2(7) + (7 − 1)(−1)] = [14 − 6 ] = [ 8 ] = 28
2 2 2
Balls in the third layer = 6 + 5 + 4 +3 + 2 + 1 = 21
Balls in the fourth layer = 5 + 4 + 3 + 2 +1 = 15
Balls in the fifth layer = 4 + 3 + 2 + 1 = 10
Balls in the sixth layer = 3 + 2 + 1 = 6
Balls in the seventh layer = 2 + 1 = 3
Balls in the eighth layer = 1
Hence the number of balls in pyramid
= 36 + 28 + 21 + 15 + 10 + 6 + 3 + 1 = 120

The End http://www.mathcity.org


Exercise 6.6 (Solutions)
MathCity.org Textbook of Algebra and
and Trigonometry for Class XI
Merging man and maths Available online @ http://www.mathcity.org, Version: 1.0.0
FSc-II- Ex 6.6 - 2 www.MathCity.org
www.MathCity.org FSc-II- Ex 6.6 - 3
FSc-II- Ex 6.6 - 4 www.MathCity.org

************************************************
Available online at http://www.MathCity.org
Exercise 6.7 (Solutions)
mathcity.org Textbook of Algebra and
a nd Trigonometry for Class XI
Merging man and maths Available online @ http://www.mathcity.org, Version: 1.0.0
FSc-I- Ex 6.7 - 2 www.mathcity.org

******************************************
Available online at http://www.mathcity.org
******************************************
Exercise 6.8 (Solutions)
MathCity.org Textbook of Algebra and
and Trigonometry for Class XI
Merging man and maths Available online @ http://www.mathcity.org, Version: 1.0.0
FSc-II- Ex 6.8 - 2 www.MathCity.org
www.MathCity.org FSc-II- Ex 6.8 - 3
FSc-II- Ex 6.8 - 4 www.MathCity.org

************************************************
Available online at http://www.MathCity.org
Exercise 6.10 (Solutions)
MathCity.org Textbook of Algebra and Trigonometry for Class XI
Merging man and maths Available online @ http://www.mathcity.org, Version: 1.0.0
FSc-II- Ex 6.10 - 2 www.MathCity.org
www.MathCity.org FSc-II- Ex 6.10 - 3
FSc-II- Ex 6.10 - 4 www.MathCity.org

Do your self
The End
mathcity.org Exercise 7.2 (Solutions)
Textbook of Algebra and Trigonometry for Class XI
Merging man and maths Available online @ http://www.mathcity.org, Version: 2.0.0

Question # 1 (i)
20! 20! 20 ⋅19 ⋅ 18 ⋅ 17!
20
P3 = = = = 20 ⋅19 ⋅18 = 6840
( 20 − 3)! 17! 17!
(ii)
16! 16! 16 ⋅ 15 ⋅ 14 ⋅ 13 ⋅ 12!
16
P4 = = = = 16 ⋅ 15 ⋅ 14 ⋅13 = 43680
(16 − 4 )! 12! 12!
Others do yourself

Question # 2 (i)
n! n ( n − 1)( n − 2 )!
n
P2 = 30 ⇒ = 30 ⇒ = 30
( n − 2 )! ( n − 2 )!
⇒ n ( n − 1) = 30 ⇒ n ( n − 1) = 6 ⋅ 5 ⇒ n = 6
(ii)
11 ⋅10 ⋅ 9 ⋅ 8!
11
Pn = 11 ⋅ 10 ⋅ 9 ⇒ = 11 ⋅ 10 ⋅ 9
(11 − n )!
8!
⇒ =1 ⇒ 8!= (11 − n )! ⇒ 8 = 11 − n ⇒ n = 11 − 8 ⇒ n = 3
(11 − n )!
(iii)
n
n−1 P4 9
n
P4 : P3 = 9 :1= ⇒ n P4 = 9 n−1P3
⇒ n−1
P3 1
n! (n − 1)! n(n − 1)! (n − 1)!
⇒ =9 ⇒ =9 ⇒ n=9
(n − 4)! (n − 1 − 3)! (n − 4)! (n − 4)!

Question # 3 (i)
(n − 1)! n(n − 1)! n!
R.H.S = n ⋅ n −1Pr −1 = n ⋅ = = = n Pr = L.H.S
( n − 1 − (r − 1) ) ( n − 1 − r + 1) ( n − r )
(ii)
n −1 (n − 1)! (n − 1)!
R.H.S = Pr + r ⋅ n−1Pr −1 = +r⋅
(n − 1 − r )! (n − 1 − r + 1)!
(n − 1)! (n − 1)! (n − 1)! (n − 1)!
= +r⋅ = +r⋅
(n − r − 1)! (n − r )! (n − r − 1)! (n − r )(n − r − 1)!
(n − 1)!  1  (n − 1)!  n − r + r 
= + ⋅ =
(n − r − 1)!  (n − r )  (n − r − 1)!  (n − r ) 
1 r

(n − 1)!  n  n (n − 1)!
= =
(n − r − 1)!  (n − r )  (n − r )(n − r − 1)!
n!
= = n Pr = L.H.S
(n − r )!

Question # 4
Total number of flags = n = 6
Number of signal using one flag = 6 P1 = 6
Number of signal using two flag = 6 P2 = 30
Number of signal using three flag = 6 P3 = 120
FSc-I / 7.2 - 2

Number of signal using four flag = 6 P4 = 360


Number of signal using five flag = 6 P5 = 720
Number of signal using six flag = 6 P6 = 720
Total number of signals = 6 + 30 + 120 + 360 + 720 + 720 = 1956

Question # 6 (i)
Since number of letters in PLANE = n = 5
Therefore total words form = 5 P5 = 120
(ii)
Since number of letters in OBJECT = n = 6
Therefore total words forms = 6 P6 = 720
(iii)
Since number of letters in FASTING = n = 7
Therefore total words forms = 7 P7 = 5040

Question # 7
Number of digits = n = 5
So numbers forms taken 3 digits at a time = 5 P3 = 60

Question # 8
Number greater than 23000 can be formed as
Number of numbers of the form 23 ∗∗∗= 3 P3 = 6
Number of numbers of the form 25 ∗∗∗= 3 P3 = 6
Number of numbers of the form 26 ∗∗∗= 3 P3 = 6
Number of numbers of the form 3 ∗∗∗∗= 4 P4 = 24
Number of numbers of the form 5 ∗∗∗∗= 4 P4 = 24
Number of numbers of the form 6 ∗∗ ∗∗= 4 P4 = 24
Thus the total number formed = 6 + 6 + 6 + 24 + 24 + 24 = 90
Ø Alternative (Submitted by Waqas Ahmad - FAZMIC Sargodha – 2004-06)
Permutation of 5 digits numbers = 5C5 = 120
Numbers less than 23000 are of the form 1∗ ∗ ∗ ∗
Then permutations = 4 C4 = 24
If number less than 23000 are of the form 21∗ ∗ ∗
Then permutations = 3C3 = 6
Thus number greater than 23000 formed = 120 − 24 − 6 = 90

Question # 9
Total number of digits = 5
(i) If we take 28 as a single digit then number of numbers = 4 P4 = 24
If we take 82 as a single digit then number of numbers = 4 P4 = 24
So the total numbers when 2 and 8 are next to each other = 24 + 24 = 48
(ii) Number of total permutation = 5 P5 = 120
thus number of numbers when 2 and 8 are not next to each other = 120 – 48 = 72

Question # 10
Since number of permutation of 6 digits = 6 P6 = 720
But 0 at extreme left is meaning less
so number of permutation when 0 is at extreme left = 5 P5 = 120
FSc-I / 7.2 - 3

Thus the number formed by 6 digits = 720 – 120 = 600


Now if we fix 0 at ten place then number formed = 5 P5 = 120

Question # 11
Number of digits = 5
For multiple of 5 we must have 5 at extreme right so number formes = 4 P4 = 24

Question # 12
Total numbers of books = 8
Total number of permutation = 8 P8 = 40320
Let E1 and E2 denotes two English books then
Number of permutation when E1E2 place together = 7 P7 = 5040
Number of permutation when E2 E1 place together = 7 P7 = 5040
So total permutation when E1 and E2 together = 5040 + 5040 = 10080
Required permutation when English books are not together = 40320 – 10080
= 30240

Question # 13
Let E1 , E2 , E3 be the book on English and U1 ,U 2 ,U 3 ,U 4 ,U 5 be the book on Urdu
Then the permutation when
books are arranged as E1 , E2 , E3 ,U1 ,U 2 ,U 3 ,U 4 ,U 5 = 3 P3 × 5 P5 = 6 × 120 = 720
books are arranged as U1 ,U 2 ,U 3 ,U 4 ,U 5 , E1 , E2 , E3 = 5 P5 × 3 P3 = 120 × 6 = 720
so total permutation when books of same subject are together = 720 + 720
= 1440

Question # 14
Let the five boys be B1 , B2 , B3 , B4 , B5 and the four girls are G1 , G2 , G3 , G4 then
there seats plane is B1 , G1 , B2 , G2 , B3 , G3 , B4 , G4 , B5
Then the permutations = 5 P1 × 4 P1 × 4 P1 × 3 P1 × 3 P1 × 2 P1 × 2 P1 × 1P1 × 1P1
= 5 × 4 × 4 × 3 × 3 × 2 × 2 × 1 × 1 = 2880

Made by Atiq ur Rehman (atiq@mathcity.org), http:/www./mathcity.org

The End
mathcity.org Exercise 7.3 (Solutions)
Textbook of Algebra and Trigonometry for Class XI
Merging man and maths Available online @ http://www.mathcity.org, Version: 2.0.5

Question # 1 (i) n! 6!
So number formed = =
PAKPATTAN p !⋅ q ! 2!⋅ 2!
Number of letters = n = 9 720
Number of P’s = p = 2 = = 180
(2) (2)
Number of A’s = q = 3 Now if numbers are of the form 3∗∗∗∗∗∗
Number of T’s = r = 2 Where each ∗ can be filled with 0, 2, 2, 2, 4, 4
Thus the number of worlds formed Then number of digits = n = 6
n! 9! Number of 2’s = p = 3
= =
p !⋅ q !⋅ r ! 2!⋅ 3!⋅ 2! Number of 4’s = q = 2
362880
= = 15120 Answer n! 6!
(2) (6) (2) So number formed = =
p !⋅ q ! 3!⋅ 2!
(ii) & (iii) 720
= = 60
Do yourself as above (6) (2)
(iv) ASSASSINATION Now if numbers are of the form 4∗∗∗∗∗∗
Number of letters = n = 13 Where each ∗ can be filled with 0, 2, 2, 2,3, 4
Number of A’s = p = 3 Then number of digits = n = 6
Number of S’s = q = 4 Number of 2’s = p = 3
Number of I’s = r = 2 n! 6!
So number formed = =
Number of N’s = s = 2 p! 3!
So the number of words 720
= = 120
n! 13! 6
= =
p !⋅ q !⋅ r !⋅ s ! 3!⋅ 4!⋅ 2!⋅ 2! So required numbers greater than 1000000
6227020800 = 180 + 60 + 120
= = 10810800 = 360
(6) (24) (2) (2)
Ø Alternative
Question # 2 (Submitted by Waqas Ahmad - FAZMIC Sargodha – 2004-06)
No. of digits = 7
If P is the first letter then words are of the
No. of 2’s = 3
form P ∗∗∗∗∗ ,
No. of 4’s = 2
Where five ∗ can be replace with A,N,A,M,A.
7!
So number of letters = n = 5 Permutations of 7 digits number =
Number of A’s = p = 3 3!⋅ 2!
5040
5! 120 = = 420
So required permutations = = = 20 (6) (2)
3! 6
Number less than 1,000,000 are of the form
0∗∗∗∗∗∗ ,
Question # 3 Where each ∗ can be replaced with 2, 2, 3, 4, 4.
If C be the first letter and K is the last letter No. of digits = 6
then words are of the form C ∗∗∗∗∗∗K .Where No. of 2’s = 3
each ∗ can be replaced with A,T,T,A,E,D. No. of 4’s = 2
So number of letters = n = 6 So permutations =
6!
=
720
= 60
Number of A’s = p = 2 3!⋅ 2! (6) (2)
Number of T’s = q = 2 Hence number greater than 1000000 = 420 − 60
n! 6! = 360
So required permutations = =
p !⋅ q ! 2!⋅ 2!
Question # 5
720 Total number of digits = n = 6
= = 180
(2) (2) Number of 2’s = p = 2
Number of 3’s = q = 2
Question # 4
The number greater than 1000000 are of the Number of 4’s = r = 2
following forms. So number formed by these 6 digits
If numbers are of the form 2 ∗∗∗∗∗∗ n! 6!
= =
Where each ∗ can be filled with 0, 2, 2,3, 4, 4 p! q! r ! (2!) (2!) (2!)
Then number of digits = n = 6 =
720
= 90
Number of 2’s = p = 2 (2) (2) (2)
Number of 4’s = q = 2
FSc-I / Ex 7.3 - 2
Question # 6 A A
Total members = n = 11
Members in first committee = p = 3
Members in second committee = q = 4 B D D B
Members in third committee = r = 2
Members in fourth committee = s = 2
So required number of committees C C

n! 11! Since above figures of arrangement are


= = reflections of each other
p !⋅ q !⋅ r !⋅ s ! 3!⋅ 4!⋅ 2!⋅ 2!
1
39916800 Therefore permutations = × 6 = 3
= = 69300 2
(6) (24) (2) (2)
Question # 12
Question # 7 Number of beads = 6
Number of D.C.O’s = 9 Fixing one bead, we have permutation = 5 P5
Let D1 and D2 be the two D.C.O’s insisting
= 120
to sit together so consider them one. A A
If D1D2 sit together then permutations
B F B
= 9 P9 = 362880 F

If D2 D1 sit together then permutations


= 9 P9 = 362880 C E E C
So total permutations = 362880 + 362880 D D
= 725760 Since above figures of arrangement are
reflections of each other
Question # 8 1
Fixing one officer on a particular seat Therefore permutations = × 120 = 60
2
We have permutations of remaining 11 officers
= 11P11 = 39916800
These notes are available online at
Question # 9 http://www.mathcity.org/fsc
9 males can be seated on a round table
= 8 P8 = 40320 Submit error/mistake at
http://www.mathcity.org/error
And 5 females can be seated on a round table
= 4 P4 = 24
So permutations of both = 40320 + 24
= 967680
Why 0! = 1
Question # 10 fix man
We know that
10! = 10 ⋅ 9 ⋅ 8.....3 ⋅ 2 ⋅1
= 10(10 − 1)(10 − 2).....3 ⋅ 2 ⋅ 1
And generally we write
n ! = n(n − 1)(n − 2)....3 ⋅ 2 ⋅1
Similarly we write
(n − 1)! = (n − 1)(n − 2)....3 ⋅ 2 ⋅1
So we can write
n ! = n (n − 1)!
If we fix one man round a table
n!
then their permutations = 4 P4 = 24 ⇒ = (n − 1)! ÷ing by n
n
Now if women sit between the two men n!
then their permutations = 5 P5 = 120 i.e. (n − 1)! =
n
So total permutations = 24 × 120 = 2880 Putting n = 1 in above
1!
Question # 11 (1 − 1)! =
1
Number of keys = 4 ⇒ 0! = 1 Proved
Fixing one key we have permutation = 3 P3 = 6
mathcity.org Exercise 7.4 (Solutions)
Textbook of Algebra and Trigonometry for Class XI
Merging man and maths Available online @ http://www.mathcity.org, Version: 1.0.2

Question # 1 (i)
12! 12! 12 ⋅ 11 ⋅ 10 ⋅ 9! 12 ⋅ 11 ⋅ 10 1320
12
C3 = = = = = = 220 Answer
(12 − 3)! 3! 9! 3! 9! 3! 3! 6
20! 20! 20 ⋅ 19 ⋅ 18 ⋅ 17! 20 ⋅19 ⋅ 18 6840
(ii) 20
C17 = = = = = = 1140 Answer
( 20 − 17 )!17! 3!17! 3!17! 3! 6
n! n (n − 1)(n − 2)(n − 3)(n − 4)! n (n − 1)(n − 2)(n − 3)
(iii) n
C4 = = = Answer
( n − 4 )! 4! ( n − 4 )! 4! 4!

Question # 2 (i)
Since nC5 = nC4
⇒ nCn −5 = nC4 Q nCr = nCn−r
⇒ n−5=4 ⇒ n= 4+5 ⇒ n=9
12 × 11 12 ⋅ 11 ⋅ 10! 12!
(ii) n
C10 = ⇒ nC10 = ⇒ nC10 =
2! 2!10! (12 − 10)!10!
⇒ nC10 = 12C10 ⇒ n = 12 .
(iii)
Do yourself as Q # 2 (i)

Question # 3 (i)
Cr = 35 and n Pr = 210
n

n! n!
Since nCr = 35 ⇒ = 35 ⇒ = 35 ⋅ r ! ………. (i)
( n − r )! r ! ( n − r )!
n!
Also n Pr = 210 ⇒ = 210 ………… (ii)
( n − r )!
Comparing (i) and (ii)
35 ⋅ r ! = 210
210
⇒ r!= ⇒ r ! = 6 ⇒ r ! = 3! ⇒ r = 3
35
Putting value of r in equation (ii)
n!
= 210
( n − 3)!
n (n − 1)(n − 2)(n − 3)!
⇒ = 210
( n − 3 )!
⇒ n (n − 1)(n − 2) = 210 ⇒ n (n − 1)(n − 2) = 7 ⋅ 6 ⋅ 5
⇒ n= 7

(ii) n−1
Cr −1 : nCr : n+1Cr +1 = 3: 6 :11
n−1 (n − 1)! n!
First consider Cr −1 : nCr = 3: 6 ⇒ : = 3:6
( n − 1 − r + 1)! (r − 1)! ( n − r )! r !
(n − 1)!


(n − 1)!
:
n!
= 3: 6 ⇒
( n − r )! (r − 1)! = 3
( n − r )! (r − 1)! ( n − r )! r ! n! 6
( n − r )! r !
FSc-I / Ex 7.4 - 2


(n − 1)!
×
( n − r )! r ! = 1 ⇒
(n − 1)! r ! 1
× =
( n − r )! (r − 1)! n! 2 (r − 1)! n! 2
(n − 1)! r (r − 1)! 1 r 1
⇒ × = ⇒ = ⇒ n = 2r ………. (i)
(r − 1)! n (n − 1)! 2 n 2
Now consider
n! (n + 1)!
n
Cr : n +1Cr +1 = 6 :11 ⇒ : = 6 :11
( n − r )! r ! ( n + 1 − r − 1)! (r + 1)!
n!


n!
:
(n + 1)!
= 6 :11 ⇒
( n − r )! r ! =
6
( n − r )! r ! ( n − r )! (r + 1)! (n + 1)! 11
( n − r )! (r + 1)!

n!
×
( n − r )! (r + 1)! = 6 ⇒
n ! (r + 1)! 6
× =
( n − r )! r ! (n + 1)! 11 r ! (n + 1)! 11
n ! (r + 1) r ! 6 (r + 1) 6
⇒ × = ⇒ = ⇒ 11(r + 1) = 6(n + 1)
r ! (n + 1) n! 11 (n + 1) 11
⇒ 11(r + 1) = 6(2r + 1) Q n = 2r
⇒ 11r + 11 = 12r + 6 ⇒ 11r − 12r = 6 − 11
⇒ − r = −5 ⇒ r =5
Putting value of r in equation (ii)
n = 2(5) ⇒ n = 10

Question # 4 (i)
(a) 5 sided polygon has 5 vertices,
so joining two vertices we have line segments = 5C2 = 10
Number of sides = 5
So number of diagonals = 10 – 5 = 5
(b) 5 sided polygon has 5 vertices,
so joining any three vertices we have triangles = 5C3 = 10
Question # 4 (ii)
(a) 8 sided polygon has 8 vertices
So joining any two vertices we have line segments = 8C2 = 28
Number of sides = 8
So number of diagonals = 28 – 8 = 20
(b) 8 sided polygon has 8 vertices,
so joining any three vertices we have triangles = 8C3 = 56 .
Question # 4 (iii)
Do yourself as above.

Queston # 5
Number of boys = 12
So committees formed taking 3 boys = 12C3 = 220
Number of girls = 8
So committees formed by taking 2 girls = = 8C2 = 28
Now total committees formed including 3 boys and 2 girls = 220 × 28 = 6160

Made by: Atiq ur Rehman ( atiq@mathcity.org ), URL: http://www.mathcity.org


FSc-I / Ex 7.4 - 3

Question # 6
Number of persons = 8
Since two particular persons are included in every committee so we have to find
combinations of 6 persons 3 at a time = 6C3 = 20
Hence number of committees = 20
Question # 7
The number of player = 15
So combination, taking 11 player at a time = 15C11 = 1365
Now if one particular player is in each collection
then number of combination = 14C10 = 1001

Question # 8
L.H.S = 16C11 + 16C10
16! 16! 16! 16!
= + = +
(16 − 11)! 11! (16 − 10 )! 10! 5! 11! 6! 10!
16! 16! 16!  1 1 
= + =  + 
5! 11 ⋅ 10! 6 ⋅ 5! 10! 10! 5!  11 6 
16!  6 + 11  16!  17  16!  17 
=  =  =  
10! 5!  66  10! 5!  66  10! 5!  11 ⋅ 6 
17 ⋅ 16! 17! 17!
= = = = 17C11 = R.H.S
11 ⋅ 10! 6 ⋅ 5! 11! 6! 11! (17 − 11)!
Alternative
L.H.S = 16C11 + 16C10 = 4368 + 8008 = 12276 …..….. (i)
R.H.S = 17C11 = 12376 ……… (ii)
From (i) and (ii)
L.H.S = R.H.S

Question # 9
Number of men = 8
Number of women = 10
(i) We have to form combination of 4 women out of 10 and 3 men out of 8
= 10C4 × 8C3 = 210 × 36 = 11760
(ii) At the most 4 women means that women are less than or equal to 4, which implies
the following possibilities (1W ,6M ) , ( 2W ,5M ) , ( 3W ,4M ) , ( 4W ,3M ) , ( 7 M )
= 10C1 × 8C6 + 10C2 × 8C5 + 10C3 × 8C4 + 10C4 × 8C3 + 8C7
= (10 )( 28) + ( 45 )( 56 ) + (120 )( 70 ) + ( 210 )( 56 ) + ( 8)
= 280 + 2520 + 8400 + 11760 + 8
= 22968
(iii) At least 4 women means that women are greater than or equal to 4, which implies
the following possibilities ( 4W ,3M ) , ( 5W ,2M ) , ( 6W ,1M ) , ( 7W )
= 10C4 × 8C3 + 10C5 × 8C2 + 10C6 × 8C1 + 10C7
= ( 210 )( 56 ) + ( 252 )( 28) + ( 210 )(8 ) + 120
= 11760 + 7056 + 1680 + 120
= 20616
FSc-I / Ex 7.4 - 4

Question # 10
n! n!
L.H.S = nCr + nCr −1 = +
( n − r )! r ! ( n − (r − 1) )! ( r − 1)!
n! n!
= +
( n − r )! r ! ( n − r + 1)! ( r − 1)!
n! n!
= +
( n − r )! r ( r − 1)! ( n − r + 1)( n − r )! ( r − 1)!
n! 1 1 
=  + 
( n − r )! ( r − 1)!
 r ( n − r + 1) 
n!  n − r +1+ r 
=  
( n − r )! ( r − 1)!  r ( n − r + 1) 
n!  n +1 
=  
( n − r )! ( r − 1)!
 r ( n − r + 1) 
=
( n + 1) n!
( n − r + 1)( n − r )! r ( r − 1)!
=
( n + 1)! =
( n + 1)!
( n − r + 1)! r! ( n + 1 − r )! r!
= n +1Cr = R.H.S

Made by: Atiq ur Rehman ( atiq@mathcity.org ), URL: http://www.mathcity.org

Submit error/mistake online at http://www.mathcity.org/error

‫ﻧﻮٹ‬
‫اﮔﺮ آپ ﮐﻮ ان ﻧﻮﭨﺲ ﻣﯿﮟ ﮐﻮﺋﯽ ﻏﻠﻄﯽ ﯾﺎ ﺧﺎﻣﯽ ﻧﻈﺮ آﺋﮯ ﯾﺎ آپ ﮐﮯ ﭘﺎس ان ﺳﻮاﻻت ﮐﮯاس ﺳﮯ‬
‫آﺳﺎن ﺣﻞ ﻣﻮﺟﻮد ھﻮں ﺗﻮ آپ اوﭘﺮ دﯾﺌﮯ ﮔﺌﮯ ای ﻣﯿﻞ اﯾﮉرس ﭘﺮ ھﻤﯿﮟ ﺑﮫﯿﺞ ﺳﮑﺘﮯ ھﯿﮟ۔ ﯾﺎ ﭘﮫﺮ آپ‬
‫ﺑﺬرﯾﻪ ﺧﻂ ﯾﺎﭘﮫﺮ ﺧﻮد اس ﭘﺘﺎ ﭘﺮ اﻃﻼع دے ﺳﮑﺘﮯ ھﯿﮟ۔‬
40170 ‫ ﺳﺮﮔﻮدھﺎ۔‬،‫ ﭨﯿﻞ‬49 ،‫ ﻣﮫﺮ ﮐﺎﻟﻮﻧﯽ‬،6‫ ﮔﻠﯽ ﻧﻤﺒﺮ‬،143 ‫ﻣﮑﺎن ﻧﻤﺮ‬
﴾048﴿3750143 : ‫ﻓﻮن‬

Error Analyst

Waqas Ahmad 2004-06 FAZMIC Sargodha.


mathcity.org Exercise 7.5 (Solutions)
Textbook of Algebra and Trigonometry for Class XI
Merging man and maths Available online @ http://www.mathcity.org, Version: 1.0.0

Question # 1
Total possible outcomes = n (S ) = 1
(i) Suppose A is the event that sweet is orange flavoured.
Since box only contained orange flavoured sweets
So favourable outcomes = n ( A) = 1
n( A) 1
Probability = P ( A) = = =1
n( S ) 1
(ii) Let B be the event that the sweet is lemon-flavoured.
Since box only contained orange-flavoured sweet
So favourable outcomes = n( B) = 0
n( B ) 0
Probability = P ( B ) = = =0
n( S ) 1

Question # 2
Since there are three possibilities that Pakistan wins, loses or the match tied.
Therefore possible outcomes = n (S ) = 3
(i) Let A be the event that Pakistan wins
Favourable outcomes = n ( A) = 1
n( A) 1
Required probability = P ( A) = =
n( S ) 3
(ii) Let B be the event that India does not lose.
If India does not lose then India may win or the match tied
Therefore favourable outcomes = n ( B ) = 2
n( B ) 2
Required probability = P ( B ) = =
n( S ) 3

Question # 3
Total number of balls = 5 + 3 = 8
Therefore possible outcomes = n( S ) = 8
(i) Let A be event that the ball is green
Then favourable outcomes = n( A) = 5
n( A) 5
So probability = P ( A) = =
n( S ) 8
(ii) Let B be the event that the ball is red
Then favourable outcomes = n( B ) = 3
n( B ) 3
So probability = P ( A) = =
n( S ) 8

Question # 4
When a fair coin is tossed three times, the possible outcomes are
HHH, HHT, HTH, THH, HTT, THT, TTH, TTT.
So total possible outcomes = n (S ) = 8
(i) Let A be the event that the coin shows one tail then favourable outcomes are
HHT, HTH, THH,
i.e. n( A) = 3
n( A) 3
So required probability = P ( A) = =
n( S ) 8
FSc-I / 7.5 - 2

(ii) Let B be the event that coin shows at least one head then favourable outcomes are
HHH, HHT, HTH, THH, HTT, THT, TTH.
i.e. n ( B ) = 7
n( B ) 7
So required probability = P ( B ) = =
n( S ) 8

Queston # 5
The possible outcomes are that die show 1, 2, 3, 4, 5, 6.
So possible outcomes = n( S ) = 6
(i) Let A be the event that die show 3 or 4.
Then favourable outcomes = n ( A) = 2
n( A) 2 1
So required probability = P ( A) = = =
n( S ) 6 3
(ii) Let B be the event that top of the die show dots less than 5 then
Favourable outcomes = n ( B ) = 4
n( B ) 4 2
So required probability = P ( B ) = = =
n( S ) 6 3

Question # 6
Since the box contain 5 slips
So possible outcomes = n (S ) = 5
(i) Let A be the event that the number on the slip are prime numbers 2, 3 or 5
Then favourable outcomes = n( A) = 3
n( A) 3
So required probability = P ( A) = =
n( S ) 5
(ii) Let B be the event that number on the slips are multiple of 3 then
Favourable outcomes = n ( B ) = 1
n( B ) 1
So probability = P ( B ) = =
n( S ) 5

Question # 7
When two dice are rolled, the possible outcomes are
(1, 1) (1, 2) (1, 3) (1, 4) (1, 5) (1, 6)
(2, 1) (2, 2) (2, 3) (2, 4) (2, 5) (2, 6)
(3, 1) (3, 2) (3, 3) (3, 4) (3, 5) (3, 6)
(4, 1) (4, 2) (4, 3) (4, 4) (4, 5) (4, 6)
(5, 1) (5, 2) (5, 3) (5, 4) (5, 5) (5, 6)
(6, 1) (6, 2) (6, 3) (6, 4) (6, 5) (6, 6)
This show possible outcomes = n( S ) = 36
(i) Let A be the event that the total of two scores is 5 then favourable outcome are
(1, 4), (2, 3), (3, 2), (4, 1)
i.e. favourable outcomes = n ( A) = 4
n( A) 4 1
So required probability = P ( A) = = =
n( S ) 36 9
(ii) Let B be the event that the total of two scores is 7 then favourable outcomes are
(1, 6), (2, 5), (3, 4), (4, 3), (5, 2), (6, 1)
i.e. favourable outcomes = n( B ) = 6
n( B ) 6 1
So probability = P ( B ) = = =
n( S ) 36 6
FSc-I / 7.5 - 3

(iii) Let C be the event that the total of two score is 11 then
favourable outcomes are (5, 6), (6, 5) i.e. n (C ) = 2
n( B ) 2 1
So probability = P ( B ) = = =
n( S ) 36 18

Question # 8
Total number of balls = 40 i.e. n ( S ) = 40
Black balls = 15, Green balls = 5, Yellow balls = 40 – (15+5) = 20
(i) Let A be the event that the ball is black then n( A) = 15
n( A) 15 3
So required probability = P ( A) = = =
n( S ) 40 8
(ii) Let B denotes the event that the ball is green then n( B ) = 5
n( B ) 5 1
So required probability = P ( B ) = = =
n( S ) 40 8
Let C denotes the event that the ball is not green then ball is either black or
yellow therefore favourable outcomes = n (C ) = 15 + 20 = 35
n(C ) 35 7
So required probability = P (C ) = = =
n( S ) 40 8

Question # 9
Number of students = 30
Then possible outcomes = n( S ) = 30
(i) Now if A be the event that the monitor is the boy then
Favourable outcomes = n( A) = 18
n( A) 18 3
So probability = P ( A) = = =
n( S ) 30 5
(ii) Now if B be the event that the monitor is the girl then
Favourable outcomes = n( B ) = 12
n( B) 12 2
So probability = P ( B ) = = =
n( S ) 30 5

Question # 10
When the coin is tossed four times the possible outcomes are
HHHT HHTH HTHH THHH
HHTT HTTH TTHH THHT
HTTT TTTH TTHT THTT
TTTT HHHH THTH HTHT
i.e. n (S ) = 16
(i) Let A be the event that the top shows all head then
favourable outcome is HHHH i.e. n ( A) = 1
n( A) 1
Now probability = P ( A) = =
n( S ) 16
(ii) Let B be the event that the top shows 2 head and two tails the favourable
outcomes are HHTT, HTTH, TTHH, THHT, THTH, HTHT
i.e. n( B ) = 6
n( B ) 6 3
Now probability = P ( B ) = = =
n( S ) 16 8
Made By: Atiq ur Rehman ( atiq@mathcity.org ), http://www.mathcity.org
mathcity.org Exercise 7.7 (Solutions)
Textbook of Algebra and Trigonometry for Class XI
Merging man and maths Available online @ http://www.mathcity.org, Version: 1.0.1

³ Question # 1
Sample space = {1,2,3,..........,9} then n ( S ) = 9
Since event A = {2,4,6,8} then n ( A) = 4
Also event B = {1,3,5} then n( B ) = 3
n( A) n( B ) 4 3 7
Now P ( A ∪ B ) = P ( A) + P( B) = + = + = Answer
n( S ) n ( S ) 9 9 9

³ Question # 2
Red marble = 10, White marble = 30, Black marble = 20
Total marble = 10 + 30 + 20 = 60
Therefore n ( S ) = 60
Let A be the event that the marble is red then n( A) = 10
And let B be the event that the marble is white then n ( B ) = 30
Since A and B are mutually exclusive event therefore
n( A) n( B) 10 30 40 2
Probability = P( A ∪ B) = P( A) + P( B) = + = + = =
n( S ) n( S ) 60 60 60 3

³ Question # 3
Since sample space is first fifty natural number so S = {1, 2,3,..........,50}
Then n( S ) = 50
Let A be the event that the chosen number is a multiple of 3 then
A = {3,6,9,........, 48} so n( A) = 16
If B be the event that the chosen number is multiple of 5 then
B = {5,10,15,........,50} so n( B ) = 10
Now A ∩ B = {15,30,45} so n ( A ∩ B ) = 3
Since A and B are not mutually exclusive event therefore
Probability = P ( A ∪ B ) = P ( A) + P ( B) − P ( A ∩ B )
n( A) n( B) n( A ∩ B )
= + −
n( S ) n( S ) n(S )
16 10 3 16 + 10 − 3 23
= + − = = Answer
50 50 50 50 50

³ Question # 4
Total number of cards = 52 ,
therefore possible outcomes = n( S ) = 52
Let A be the event that the card is a diamond card.
Since there are 13 diamond card in the deck therefore n( A) = 13
Now let B the event that the card is an ace card.
Since there are 4 ace cards in the deck therefore n ( B ) = 4
Since one diamond card is also an ace card therefore A and B are not mutually
exclusive event and n ( A ∩ B) = 1
Now probability = P ( A ∪ B ) = P ( A) + P ( B) − P ( A ∩ B )
n( A) n( B) n( A ∩ B )
= + −
n( S ) n( S ) n(S )
13 4 1 13 + 4 − 1 16 4
= + − = = = Answer
52 52 52 52 52 13
³ Question # 5
When die is thrown twice the possible outcomes are
(1, 1) (1, 2) (1, 3) (1, 4) (1, 5) (1, 6)
(2, 1) (2, 2) (2, 3) (2, 4) (2, 5) (2, 6)
(3, 1) (3, 2) (3, 3) (3, 4) (3, 5) (3, 6)
(4, 1) (4, 2) (4, 3) (4, 4) (4, 5) (4, 6)
(5, 1) (5, 2) (5, 3) (5, 4) (5, 5) (5, 6)
(6, 1) (6, 2) (6, 3) (6, 4) (6, 5) (6, 6)
This shows possible outcomes = n( S ) = 36
Let A be the event that the sum is 3
Then the favourable outcomes are (1, 2) and (2, 1), i.e. n ( A) = 2
Now let B the event that the sum is 11
Then the favourable outcomes are (5, 6) and (6, 5), i.e. n ( B ) = 2
Since A and B are mutually exclusive events therefore
n( A) n( B ) 2 2 4 1
Probability = P ( A ∪ B ) = P ( A) + P( B) = + = + = =
n( S ) n (S ) 36 36 36 9
³ Question # 6 Do yourself as above
³ Question # 7
When two dice are thrown the possible outcomes are
[ See the dice table of Question # 5 ]
This shows possible outcomes = n( S ) = 36
Since A be the event that the sum of dots is and odd number
Then favourable outcomes are
(1, 2), (1, 4), (1, 6), (2, 1), (2, 3), (2, 5), (3, 2), (3, 4), (3, 6),
(4, 1), (4, 3), (4, 5), (5, 2), (5, 4), (5, 6), (6, 1), (6, 3), (6, 5)
i.e. favourable outcomes = n( A) = 18
Sine B is the event that the least one die has 3 dot on it therefore
favourable outcomes are (1, 3), (2, 3), (3, 1), (3, 2), (3, 3), (3, 4),(3, 5), (3, 6),
(4, 3), (5, 3), (6, 3) i.e. favourable outcomes = n ( B ) = 11
Since A and B have common outcome (2, 3), (3, 2), (3, 4), (3, 6), (4, 3), (6, 3)
i.e n ( A ∩ B ) = 6
Now probability = P ( AUB) = P( A) + P( B) − P( A ∩ B)
n( A) n( B) n( A ∩ B )
= + −
n( S ) n( S ) n(S )
18 11 6 18 + 11 − 6 23
= + − = = Answer
36 36 36 36 36
³ Question # 8
Number of girls = 10, Number of boys = 20
Total number of students = 10 + 20 = 30
Since half of the girls and half of the boys have blue eyes
Therefore students having blue eyes = 5 + 10 = 15
Let A be event that monitor of the class is a student of blue eyes then n( A) = 15
Now Let B be the event that the monitor of the class is girl then n( B ) = 10
Since 5 girls have blue eyes therefore A and B are not mutually exclusive
Therefore n ( A ∩ B ) = 5
Now probability = P ( A ∪ B ) = P ( A) + P ( B) − P ( A ∩ B )
n( A) n( B) n( A ∩ B )
= + −
n( S ) n( S ) n(S )
15 10 5 15 + 10 − 5 20 2
= + − = = = Answer
30 30 30 30 30 3
mathcity.org Exercise 7.8 (Solutions)
Textbook of Algebra and Trigonometry for Class XI
Merging man and maths Available online @ http://www.mathcity.org, Version: 2.0.0

Question # 1
5
Since P ( A) =
7
7
And P ( B ) =
9
Then the probability that both will alive 15 year is
5 7 5
P ( A ∩ B ) = P ( A) ⋅ P( B) = ⋅ = Answer
7 9 9

Question # 2
When a die is rolled then possible outcomes are
1, 2 ,3 , 4, 5, 6
This shows that possible outcomes = n( S ) = 6
Since E1 is the event that the dots on the die are even then favourable outcomes
are 2, 4, 6
this shows n ( E1 ) = 3
n( E1 ) 3 1
so probability = P ( E1 ) = = =
n( S ) 6 2
Now since E2 is the event that the dot appear are more than four then favourable
outcomes are 5 and 6. This show n( E2 ) = 2
n ( E2 ) 2 1
So probability = P ( E2 ) = = =
n(S ) 6 3
Since E1 and E2 are not mutually exclusive
And the possible common outcome is 6 i.e. n ( E1 ∩ E2 ) = 1
n( E1 ∩ E2 ) 1
So probability P ( E1 ∩ E2 ) = = ……….. (i)
n( S ) 6
1 1 1
Now P ( E1 ) ⋅ P ( E2 ) = ⋅ = …………. (ii)
2 3 6
Form (i) and (ii)
P ( E1 ∩ E2 ) = P ( E1 ) ⋅ P ( E2 ) Proved.

Question # 3
When two coins are tossed then possible outcomes are
HH, HT, TH, TT
i.e. n( S ) = 4
Let A be the event of getting two heads then favourable outcome is HH.
so n ( A) = 1
n( A) 1
Now probability = P ( A) = = Answer
n( S ) 4

Question # 4
When the two coins are tossed then possible outcomes are
HH, HT, TH, TT
This shows n( S ) = 4
Let A be the event that head appear in the first toss then
favourable outcomes are HT, HH, i.e. n ( A) = 2
FSc-I / 7.8 - 2

Let B be the event that same face appear on the second toss then
favourable outcomes are HH, TT. i.e. n ( B ) = 2
Now probability = P ( A ∩ B ) = P ( A) ⋅ P( B)
n( A) n( B) 2 2 1 1 1
= ⋅ = ⋅ = ⋅ = Answer
n( S ) n( S ) 4 4 2 2 4

Question # 5
Since there are 52 cards in the deck therefore n( S ) = 52
Let A be the event that first card is an ace then n ( A) = 4
And let B be the event that the second card is also an ace then n ( B ) = 4
Now probability = P ( A ∩ B ) = P ( A) ⋅ P( B)
n( A) n( B) 4 4 1
= ⋅ = ⋅ = Answer
n( S ) n( S ) 52 52 169

Question # 6
Since there are 52 cards in the deck therefore n( S ) = 52
(i) Let A be the event that the first card is king then n ( A) = 4
and let B be the event that the second card is queen then n ( B ) = 4
Now probability = P ( A ∩ B ) = P ( A) ⋅ P( B)
n( A) n( B) 4 4 1
= ⋅ = ⋅ = Answer
n( S ) n( S ) 52 52 169
(ii) Let C be the event that first card is faced card.
Since there are 12 faced card in the deck therefore n (C ) = 12
and let D be the event that the second card is also faced card then n ( D ) = 12
Now probability = P ( A ∩ B ) = P ( A) ⋅ P( B)
n( A) n( B) 12 12 3 3 9
= ⋅ = ⋅ = ⋅ = Answer
n( S ) n( S ) 52 52 13 13 169

Question # 7
When the two dice are thrown the possible outcomes are
(1, 1) (1, 2) (1, 3) (1, 4) (1, 5) (1, 6)
(2, 1) (2, 2) (2, 3) (2, 4) (2, 5) (2, 6)
(3, 1) (3, 2) (3, 3) (3, 4) (3, 5) (3, 6)
(4, 1) (4, 2) (4, 3) (4, 4) (4, 5) (4, 6)
(5, 1) (5, 2) (5, 3) (5, 4) (5, 5) (5, 6)
(6, 1) (6, 2) (6, 3) (6, 4) (6, 5) (6, 6)
Which shows that n( S ) = 36
Let A be the event that the sum of dots in first throw is 7 then
favourable outcomes are (1, 6), (2, 5), (3, 4), (4, 3), (5, 2), (6, 1) i.e. n( A) = 6
Let B be the event that the sum of dots in second throw is 11 then
favourable outcomes are (5, 6), (6, 5) i.e. n ( B ) = 2
Now probability = P ( A ∩ B ) = P ( A) ⋅ P( B)
n( A) n( B) 6 2 1 1 1
= ⋅ = ⋅ = ⋅ = Answer
n( S ) n( S ) 36 36 6 18 108
FSc-I / 7.8 - 3

Question # 8
When the two dice are thrown the possible outcomes are
(1, 1) (1, 2) (1, 3) (1, 4) (1, 5) (1, 6)
(2, 1) (2, 2) (2, 3) (2, 4) (2, 5) (2, 6)
(3, 1) (3, 2) (3, 3) (3, 4) (3, 5) (3, 6)
(4, 1) (4, 2) (4, 3) (4, 4) (4, 5) (4, 6)
(5, 1) (5, 2) (5, 3) (5, 4) (5, 5) (5, 6)
(6, 1) (6, 2) (6, 3) (6, 4) (6, 5) (6, 6)
Which shows that n( S ) = 36
Let A be the event that the sum of dots in first throw is 7 then
favourable outcomes are (1, 6), (2, 5), (3, 4), (4, 3), (5, 2), (6, 1) i.e. n( A) = 6
Let B be the event that the sum of dots in second throw is also 7 then
similarly favourable outcomes = n( B ) = 6
Now probability = P ( A ∩ B ) = P ( A) ⋅ P( B)
n( A) n( B) 6 6 1 1 1
= ⋅ = ⋅ = ⋅ = Answer
n( S ) n( S ) 36 36 6 6 36

Question # 9
When the die is thrown twice then the top may shows 1, 2, 3, 4, 5, 6
This shows possible outcomes = n( S ) = 6
Let A be the event that the number of the dots is prime then
favourable outcomes are 2, 3 ,5 , i.e. n( A) = 3
Let B be the event that the number of dots in second throw is less than 5 then
favourable outcomes are 1 ,2 ,3 ,4 i.e. n ( B ) = 4
Now probability = P ( A ∩ B ) = P ( A) ⋅ P( B)
n( A) n( B) 3 4 1 2 1
= ⋅ = ⋅ = ⋅ = Answer
n( S ) n( S ) 6 6 2 3 3

Question # 10
Since number of red balls = 8
Number of white ball = 5
Number of black ball = 7
Therefore total number of balls = 8 + 5 + 7 = 20 i.e. n ( S ) = 20
Let A be the event that the first ball is red then n( A) = 8
Let B be the event that the second ball is white then n( B ) = 5
Let C be the event that the third ball is black then n (C ) = 7
Now probability = P ( A ∩ B ∩ C ) = P( A) ⋅ P ( B ) ⋅ P (C )
n( A) n( B) n(C )
= ⋅ ⋅
n( S ) n( S ) n( S )
8 5 7 2 1 7 14 7
= ⋅ ⋅ = ⋅ ⋅ = = Answer
20 20 20 5 4 20 400 200

Made by: Atiq ur Rehman (atiq@mathcity.org ), http://www.mathcity.org

THE END
mathcity.org Exercise 8.1 (Solutions)
Textbook of Algebra and Trigonometry for Class XI
Merging man and maths Available online @ http://www.mathcity.org, Version: 1.6.7

Principle of Mathematical Induction


A given statement S (n) is true for each positive integer n if two below conditions hold
Condition I: S (1) is true i.e. S (n) is true for n = 1 and
Condition II: S (k + 1) is true whenever S (k ) is true for any positive integer k,
Then S (n) is true for all positive integers
Question # 1
Suppose S (n) : 1 + 5 + 9 + .............. + (4n − 3) = n (2n − 1)
Put n = 1
S (1) : 1 = 1( 2(1) − 1) ⇒ 1 = 1
Thus condition I is satisfied
Now suppose that S (n) is true for n = k
S (k ) : 1 + 5 + 9 + .............. + (4k − 3) = k (2k − 1) …………. (i)
The statement for n = k + 1 becomes
S (k + 1) : 1 + 5 + 9 + .............. + (4(k + 1) − 3) = (k + 1)(2(k + 1) − 1)
⇒ 1 + 5 + 9 + .............. + (4 k + 1) = (k + 1)(2k + 2 − 1)
= (k + 1)(2k + 1)
= 2k 2 + 2k + k + 1
= 2k 2 + 3k + 1
Adding 4k + 1 on both sides of equation (i)
1 + 5 + 9 + .............. + (4k − 3) + (4k + 1) = k (2k − 1) + 4k + 1
⇒ 1 + 5 + 9 + .............. + (4k + 1) = 2k 2 − k + 4k + 1
= 2k 2 + 3k + 1
Thus S (k + 1) is true if S (k ) is true, so condition II is satisfied and S (n) is true
for all positive integer n.
Question # 2
Suppose S (n) : 1 + 3 + 5 + .............. + (2n − 1) = n 2
Put n = 1
S (1) : 1 = (1) ⇒ 1 = 1
2

Thus condition I is satisfied


Now suppose that S (n) is true for n = k
S (k ) : 1 + 3 + 5 + .............. + (2k − 1) = k 2 …………. (i)
The statement for n = k + 1 becomes
S (k + 1) : 1 + 3 + 5 + .............. + (2(k + 1) − 1) = (k + 1)2
⇒ 1 + 3 + 5 + .............. + (2k + 1) = (k + 1)2
Adding 2k + 1 on both sides of equation (i)
1 + 3 + 5 + .............. + (2k − 1) + (2k + 1) = k 2 + 2k + 1
⇒ 1 + 3 + 5 + .............. + (2k + 1) = (k + 1)2
Thus S (k + 1) is true if S (k ) is true, so condition II is satisfied and S (n) is true
for all positive integer n.
Question # 3
n (3n − 1)
Suppose S (n) : 1 + 4 + 7 + .............. + (3n − 2) =
2
Put n = 1
1( 3(1) − 1) 2
S (1) : 1 = ⇒ 1= ⇒ 1=1
2 2
FSc-I / Ex 8.1 - 2

Thus condition I is satisfied


Now suppose that S (n) is true for n = k
k (3k − 1)
S (k ) : 1 + 4 + 7 + .............. + (3k − 2) = …………. (i)
2
The statement for n = k + 1 becomes
(k + 1)(3(k + 1) − 1)
S (k + 1) : 1 + 4 + 7 + .............. + (3(k + 1) − 2) =
2
(k + 1)(3k + 3 − 1)
⇒ 1 + 4 + 7 + .............. + (3k + 1) =
2
(k + 1)(3k + 2)
=
2
Adding 3k + 1 on both sides of equation (i)
k (3k − 1)
1 + 4 + 7 + .............. + (3k − 2) + (3k + 1) = + 3k + 1
2
k (3k − 1) + 2(3k + 1)
⇒ 1 + 4 + 7 + .............. + (3k + 1) =
2
3k − k + 6k + 2
2
=
2
3k + 5k + 2
2
=
2
3k + 3k + 2k + 2
2
=
2
3k (k + 1) + 2(k + 1)
=
2
(k + 1)(3k + 2)
=
2
Thus S (k + 1) is true if S (k ) is true, so condition II is satisfied and S (n) is true
for all positive integer n.

Question # 4
Suppose S (n) : 1 + 2 + 4 + .............. + 2 n−1 = 2n − 1
Put n = 1
S (1) : 1 = 21 − 1 ⇒ 1 = 1
Thus condition I is satisfied
Now suppose that S (n) is true for n = k
S (k ) : 1 + 2 + 4 + .............. + 2 k −1 = 2k − 1 …………. (i)
The statement for n = k + 1 becomes
S (k + 1) : 1 + 2 + 4 + .............. + 2 k +1−1 = 2k +1 − 1
⇒ 1 + 2 + 4 + .............. + 2k = 2k +1 − 1
Adding 2k on both sides of equation (i)
1 + 2 + 4 + .............. + 2 k −1 + 2k = 2k − 1 + 2 k
⇒ 1 + 2 + 4 + .............. + 2k = 2(2k ) − 1 Q 2k + 2k = 2(2k )
= 2 k +1 − 1
Thus S (k + 1) is true if S (k ) is true, so condition II is satisfied and S (n) is true
for all positive integer n.

Made by: Atiq ur Rehman ( atiq@mathcity.org ), http://www.mathcity.org


FSc-I / Ex 8.1 - 3

Question # 5
1 1 1  1 
Suppose S (n) : 1 + + + .............. + n −1 = 2 1 − n 
2 4 2  2 
Put n = 1
 1 1
S (1) : 1 = 2 1 − 1  ⇒ 1 = 2   ⇒ 1 = 1
 2  2
Thus condition I is satisfied
Now suppose that S (n) is true for n = k
1 1 1  1 
S (k ) : 1 + + + .............. + k −1 = 2 1 − k  …………. (i)
2 4 2  2 
The statement for n = k + 1 becomes
1 1 1  1 
S (k + 1) : 1 + + + .............. + k +1−1 = 2 1 − k +1 
2 4 2  2 
1 1 1 2
⇒ 1 + + + .............. + k = 2 − k +1
2 4 2 2
2
=2− k
2 ⋅2
1
=2− k
2
1
Adding k on both sides of equation (i)
2
1 1 1 1  1  1
1 + + + .............. + k −1 + k +1−1 = 2 1 − k  + k
2 4 2 2  2  2
1 1 1 2 1
⇒ 1 + + + .............. + k +1−1 = 2 − k + k
2 4 2 2 2
1
= 2 − k ( 2 − 1)
2
1 1
= 2 − k (1) = 2 − k
2 2
Thus S (k + 1) is true if S (k ) is true, so condition II is satisfied and S (n) is true
for all positive integer n.

Question # 6 Do yourself as Question # 1


Question # 7
Suppose S (n) : 2 + 6 + 18 + .............. + 2 × 3n−1 = 3n − 1
Put n = 1
S (1) : 2 = 31 − 1 ⇒ 2 = 2
Thus condition I is satisfied
Now suppose that S (n) is true for n = k
S (k ) : 2 + 6 + 18 + .............. + 2 × 3k −1 = 3k − 1 …………. (i)
The statement for n = k + 1 becomes
S (k + 1) : 2 + 6 + 18 + .............. + 2 × 3k +1−1 = 3k +1 − 1
Adding 2 × 3k on both sides of equation (i)
2 + 6 + 18 + .............. + 2 × 3k −1 + 2 × 3k = 3k − 1 + 2 × 3k
⇒ 2 + 6 + 18 + .............. + 2 × 3k +1−1 = 3(3k ) − 1 Q 3k + 2 × 3k = 3(3k )
= 3k +1 − 1
Thus S (k + 1) is true if S (k ) is true, so condition II is satisfied and S (n) is true
for all positive integer n.
FSc-I / Ex 8.1 - 4

Question # 8
n (n + 1)(4n + 5)
Suppose S (n) : 1 × 3 + 2 × 5 + 3 × 7 + .............. + n × (2n + 1) =
6
Put n = 1
1(1 + 1)(4(1) + 5) (2)(9)
S (1) : 1 × 3 = ⇒ 3= ⇒ 3=3
6 6
Thus condition I is satisfied
Now suppose that S (n) is true for n = k
k (k + 1)(4k + 5)
S (k ) : 1 × 3 + 2 × 5 + 3 × 7 + .............. + k × (2k + 1) = ……. (i)
6
The statement for n = k + 1 becomes
( k + 1) ( k + 1 + 1) (4(k + 1) + 5)
S (k + 1) : 1 × 3 + 2 × 5 + 3 × 7 + .............. + (k + 1) × (2(k + 1) + 1) =
6
(k + 1)(k + 2)(4k + 9)
⇒ 1 × 3 + 2 × 5 + 3 × 7 + .............. + (k + 1) × (2k + 3) =
6
Adding (k + 1) × (2k + 3) on both sides of equation (i)
k ( k + 1) (4k + 5)
1 × 3 + 2 × 5 + 3 × 7 + .............. + k × (2 k + 1) + ( k + 1) × (2k + 3) = + ( k + 1) × (2k + 3)
6
 k (4k + 5) 
⇒ 1 × 3 + 2 × 5 + 3 × 7 + .............. + (k + 1) × (2k + 3) = (k + 1)  + (2k + 3) 
 6 
 k (4k + 5) + 6(2k + 3) 
= (k + 1)  
 6 
 4k + 5k + 12 k + 18 
2
= (k + 1)  
 6 
 4k 2 + 17 k + 18 
= (k + 1)  
 6 
 4k 2 + 17 k + 18 
= (k + 1)  
 6 
 4k 2 + 8k + 9k + 18 
= (k + 1)  
 6 
 4k (k + 2) + 9(k + 2) 
= (k + 1)  
 6 
 (k + 2)(4k + 9) 
= (k + 1)  
 6 
(k + 1)(k + 2)(4k + 9)
=
6
Thus S (k + 1) is true if S (k ) is true, so condition II is satisfied and S (n) is true
for all positive integer n.
Question # 9
Do yourself as Question # 8

Question # 10
Do yourself as Question # 8

Made by: Atiq ur Rehman ( atiq@mathcity.org ), http://www.mathcity.org


FSc-I / Ex 8.1 - 5

Question # 11
1 1 1 1 1
Suppose S (n) : + + + .............. + =1−
1× 2 2 × 3 3 × 4 n (n + 1) n +1
Put n = 1
1 1 1 1 1 1
S (1) : =1− ⇒ =1− ⇒ =
1× 2 1+1 2 2 2 2
Thus condition I is satisfied
Now suppose that S (n) is true for n = k
1 1 1 1 1
S (k ) : + + + .............. + =1− …………. (i)
1× 2 2 × 3 3 × 4 k (k + 1) k +1
The statement for n = k + 1 becomes
1 1 1 1
S (k + 1) : 1 + + + .............. + =1−
2 4 (k + 1)(k + 1 + 1) k +1+1
1 1 1 1
⇒ 1 + + + .............. + =1−
2 4 (k + 1)(k + 2) k+2
1
Adding on both sides of equation (i)
(k + 1)(k + 2)
1 1 1 1 1 1 1
+ + + .............. + + =1− +
1× 2 2 × 3 3 × 4 k (k + 1) (k + 1)(k + 2) k + 1 (k + 1)(k + 2)
1 1 1 1 1  1 
⇒ + + + .............. + =1− 1−
1× 2 2 × 3 3 × 4 (k + 1)(k + 2) k + 1  (k + 2) 

1  k + 2 −1
=1−  
k + 1 k + 2 
1  k +1 
=1−  
k + 1 k + 2 
1
=1−
k+2
Thus S (k + 1) is true if S (k ) is true, so condition II is satisfied and S (n) is true
for all positive integer n.

Question # 12
1 1 1 1 n
Suppose S (n) : + + + .............. + =
1× 3 3 × 5 5 × 7 (2n − 1)(2n + 1) 2 n + 1
Put n = 1
1 1 1 1
S (1) : = ⇒ =
1 × 3 2(1) + 1 3 3
Thus condition I is satisfied
Now suppose that S (n) is true for n = k
1 1 1 1 k
S (k ) : + + + .............. + = ………. (i)
1× 3 3 × 5 5 × 7 (2 k − 1)(2k + 1) 2k + 1
The statement for n = k + 1 becomes
1 1 1 1 k +1
S (k + 1) : + + + .............. + =
1× 3 3 × 5 5 × 7 ( 2(k + 1) − 1)( 2(k + 1) + 1) 2(k + 1) + 1
1 1 1 1 k +1
⇒ + + + .............. + =
1× 3 3 × 5 5 × 7 ( 2k + 1)( 2k + 3) 2k + 3
1
Adding on both sides of equation (i)
( 2k + 1)( 2k + 3)
FSc-I / Ex 8.1 - 6

1 1 1 1 1 k 1
+ + + .............. + + = +
1× 3 3 × 5 5 × 7 (2k − 1)(2k + 1) ( 2k + 1)( 2k + 3) 2k + 1 ( 2k + 1)( 2k + 3)
1 1 1 1 1  1 
⇒ + + + .............. + =  k + 
1× 3 3 × 5 5 × 7 ( 2k + 1)( 2k + 3) 2k + 1  ( 2k + 3) 
1  k (2k + 3) + 1 
=  
2k + 1  2k + 3 
1  2k 2 + 3k + 1 
=  
2k + 1  2 k + 3 
1  2k 2 + 2k + k + 1 
=  
2k + 1  2k + 3 
1  2k (k + 1) + 1(k + 1) 
=  
2k + 1  2k + 3 
1  (2k + 1)(k + 1) 
=  
2k + 1  2k + 3 
 k +1 
= 
 2k + 3 
Thus S (k + 1) is true if S (k ) is true, so condition II is satisfied and S (n) is true
for all positive integer n.
Question # 13 Do yourself as Question # 12
Question # 14
r (1 − r n )
Suppose S (n) : r + r + r + .............. + r =
2 3 n

1− r
Put n = 1

S (1) : r =
(
r 1 − r1 )
⇒ r =r
1− r
Thus condition I is satisfied
Now suppose that S (n) is true for n = k
r (1 − r k )
S (k ) : r + r + r + .............. + r =
2 3 k
…………. (i)
1− r
The statement for n = k + 1 becomes
k +1 r (1 − r k +1 )
S (k + 1) : r + r + r + .............. + r =
2 3

1− r
k +1
Adding r on both sides of equation (i)
k +1 r (1 − r k )
r + r + r + .............. + r + r =
2 3 k
+ r k +1
1− r
k +1 r (1 − r ) + r k +1 (1 − r )
k
⇒ r + r + r + .............. + r =
2 3

1− r
r − r + r k +1 − r k +2
k +1
=
1− r
k +2
r−r
=
1− r
r (1 − r k +1 )
=
1− r
Thus S (k + 1) is true if S (k ) is true, so condition II is satisfied and S (n) is true
for all positive integer n.
FSc-I / Ex 8.1 - 7

Question # 15
n
Suppose S (n) : a + (a + d ) + (a + 2 d ) + ............... + [ a + (n − 1)d ] = [ 2a + (n − 1)d ]
2
Put n = 1
1 1 1
S (1) : a = [ 2a + (1 − 1)d ] ⇒ a = [ 2a + (0)d ] ⇒ a = [ 2a ] = a
2 2 2
Thus condition I is satisfied
Now suppose that S (n) is true for n = k
k
S (k ) : a + (a + d ) + (a + 2 d ) + ............... + [ a + (k − 1)d ] = [ 2a + (k − 1)d ] …..…. (i)
2
The statement for n = k + 1 becomes
k +1
S (k + 1) : a + (a + d ) + (a + 2d ) + ............... + [ a + (k + 1 − 1)d ] = [ 2a + (k + 1 − 1)d ]
2
k +1
⇒ a + (a + d ) + (a + 2d ) + ............... + [ a + kd ] = [ 2a + kd ]
2
Adding a + kd on both sides of equation (i)
k
a + (a + d ) + (a + 2 d ) + ............... + [ a + (k − 1)d ] + [ a + kd ] = [ 2a + (k − 1)d ] + [ a + kd ]
2
k
⇒ a + (a + d ) + (a + 2d ) + ............... + [ a + kd ] = [ 2a + kd − d ] + [ a + kd ]
2
k [ 2a + kd − d ] + 2 [ a + kd ]
=
2
2ak + k d − kd + 2a + 2kd
2
=
2
2ak + k d + kd + 2a
2
=
2
2ak + 2 a + k 2 d + kd
=
2
2a (k + 1) + kd (k + 1)
=
2
(k + 1)(2a + kd )
=
2
k +1
= [ 2a + kd ]
2
Thus S (k + 1) is true if S (k ) is true, so condition II is satisfied and S (n) is true
for all positive integer n.
Question # 16
Suppose S (n) : 1 ⋅ 1 + 2 ⋅ 2 + 3 ⋅ 3 + .............. + n ⋅ n = n + 1 − 1
Put n = 1
S (1) : 1 ⋅ 1 = 1 + 1 − 1 ⇒ 1 = 2 − 1 ⇒ 1 = 2 − 1 ⇒ 1 = 1
Thus condition I is satisfied
Now suppose that S (n) is true for n = k
S (k ) : 1 ⋅ 1 + 2 ⋅ 2 + 3 ⋅ 3 + .............. + k ⋅ k = k + 1 − 1 …………. (i)
The statement for n = k + 1 becomes
S (k + 1) : 1 ⋅ 1 + 2 ⋅ 2 + 3 ⋅ 3 + .............. + (k + 1) ⋅ k + 1 = k + 1 + 1 − 1
⇒ 1 ⋅ 1 + 2 ⋅ 2 + 3 ⋅ 3 + .............. + (k + 1) ⋅ k + 1 = k + 2 − 1
Adding (k + 1) ⋅ k + 1 on both sides of equation (i)
1 ⋅ 1 + 2 ⋅ 2 + 3 ⋅ 3 + .............. + k ⋅ k + (k + 1) ⋅ k + 1 = k + 1 − 1 + (k + 1) ⋅ k + 1
FSc-I / Ex 8.1 - 8

⇒ 1 ⋅ 1 + 2 ⋅ 2 + 3 ⋅ 3 + .............. + (k + 1) ⋅ k + 1 = k + 1 + k + 1(k + 1) − 1
= k + 1 (1 + k + 1) − 1
= k + 1 ( k + 2) − 1
= ( k + 2) k + 1 − 1
= k + 2 −1
Thus S (k + 1) is true if S (k ) is true, so condition II is satisfied and S (n) is true
for all positive integer n.
Question # 17
Suppose S (n) : an = a1 + (n − 1)d
Put n = 1
S (1) : a1 = a1 + (1 − 1)d ⇒ a1 = a1 + 0d = a1
Thus condition I is satisfied
Now suppose that S (n) is true for n = k
S (k ) : ak = a1 + (k − 1)d …………. (i)
The statement for n = k + 1 becomes
S (k + 1) : ak +1 = a1 + (k + 1 − 1)d
= a1 + (k )d
Adding d on both sides of equation (i)
ak + d = a1 + (k − 1)d + d Q a2 = a1 + d
a3 = a2 + d
⇒ ak +1 = a1 + (k − 1 + 1)d
∴ ak +1 = ak + d
⇒ ak +1 = a1 + (k )d
Thus S (k + 1) is true if S (k ) is true, so condition II is satisfied and S (n) is true
for all positive integer n.
Question # 18
Suppose S (n) : an = a1r n −1
Put n = 1
S (1) : a1 = a1r 1−1 ⇒ a1 = a1r 0 = a1
Thus condition I is satisfied
Now suppose that S (n) is true for n = k
S (k ) : ak = a1r k −1 …………. (i)
The statement for n = k + 1 becomes
S (k + 1) : ak +1 = a1r k +1−1
= a1r k
Multiplying r on both sides of equation (i)
ak ⋅ r = a1r k −1 ⋅ r1 Q a2 = a1r
a3 = a2 r
⇒ ak +1 = a1r k −1+1 ∴ ak +1 = ak r
⇒ ak +1 = a1r k

Thus S (k + 1) is true if S (k ) is true, so condition II is satisfied and S (n) is true


for all positive integer n.
Question # 19
n (4n 2 − 1)
Suppose S (n) : 1 + 3 + 5 + ............... + (2n − 1) =
2 2 2 2

3
Put n = 1
1(4(1) 2 − 1) 1(4 − 1) 3
S (1) : 1 =
2
⇒ 1= ⇒ 1= =1
3 3 3
Thus condition I is satisfied
FSc-I / Ex 8.1 - 9

Now suppose that S (n) is true for n = k


k (4k 2 − 1)
S (k ) : 1 + 3 + 5 + ............... + (2k − 1) =
2 2 2 2
…..…. (i)
3
The statement for n = k + 1 becomes
(k + 1)(4(k + 1)2 − 1)
S (k + 1) : 1 + 3 + 5 + ............... + (2(k + 1) − 1) =
2 2 2 2

3
(k + 1)(4(k + 2k + 1) − 1)
2
⇒ 12 + 32 + 52 + ............... + (2k + 1)2 =
3
(k + 1)(4k + 8k + 4 − 1)
2
=
3
(k + 1)(4k + 8k + 3)
2
=
3
4k + 8k + 3k + 4k 2 + 8k + 3
3 2
=
3
4k 3 + 12k 2 + 11k + 3
=
3
Adding ( 2k + 1) on both sides of equation (i)
2

k (4k 2 − 1)
1 + 3 + 5 + ............... + (2k − 1) + ( 2k + 1) = + ( 2k + 1)
2 2 2 2 2 2

3
k (4 k − 1) + 3(2k + 1) 2
2
⇒ 12 + 32 + 52 + ............... + ( 2k + 1) =
2

3
k (4k − 1) + 3(4k 2 + 4k + 1)
2
=
3
4k − k + 12k 2 + 12k + 3
3
=
3
4k + 12k + 11k + 3
3 2
=
3
Thus S (k + 1) is true if S (k ) is true, so condition II is satisfied and S (n) is true
for all positive integer n.
Question # 20
 3  4   5  n + 2   n + 3
Suppose S (n) :   +   +   + ............... +  = 4 
     
3 3 3  3   
Put n = 1
 3
L.H.S =   = 1
 3
1 + 3   4 
R.H.S =   =   =1
 4  4
L.H.S = R.H.S
Thus condition I is satisfied
Now suppose that S (n) is true for n = k
 3  4   5  k + 2   k + 3
S (k ) :   +   +   + ............... +   =  4  …..…. (i)
     
3 3 3  3   
The statement for n = k + 1 becomes
 3  4   5  k + 1 + 2   k + 1 + 3
S (k + 1) :   +   +   + ............... +  = 4 
     
3 3 3  3   
FSc-I / Ex 8.1 - 10

 3  4   5  k + 3  k + 4 
⇒   +   +   + ............... +  = 
 3  3   3  3   4 
 k + 3
Adding   on both sides of equation (i)
 3 
 3  4   5  k + 2  k + 3  k + 3  k + 3
  +   +   + ............... +  + = + 
     
3 3 3  3   3   4   3 
 3  4   5  k + 3   k + 3 + 1
⇒   +   +   + ............... +  =   n   n   n + 1
 3  3   3  3   4  Q  + = r 
  
r r − 1   
 3  4   5  k + 3  k + 4 
⇒   +   +   + ............... +  = 
 3  3   3  3   4 
Thus S (k + 1) is true if S (k ) is true, so condition II is satisfied and S (n) is true
for all positive integer n.
Question # 21(i)
Suppose S (n) : n 2 + n
Put n = 1
S (1) : 12 + 1 = 2
S (1) is clearly divisible by 2, Thus condition I is satisfied
Now suppose that given statement is true for n = k
S (k ) : k 2 + k Q
Then there exists quotient Q such that 2 k2 + k
k 2 + k = 2Q k2 + k
The statement for n = k + 1 − −
0
S (k + 1) : ( k + 1) + k + 1
2

= k 2 + 2k + 1 + k + 1
= k 2 + k + 2k + 2
= 2Q + 2k + 2 Q k 2 + k = 2Q
= 2(Q + k + 1)
Clearly S (k + 1) is divisible by 2.
Since the truth for n = k implies the truth for n = k + 1 therefore the given statement
is true for ∀ n ∈¢ + .
Question # 21 (ii)
Suppose S (n) : 5n − 2 n
Put n = 1
S (1) : 51 − 21 = 3
S (1) is clearly divisible by 3, Thus condition I is satisfied
Now suppose that given statement is true for n = k
S ( k ) : 5k − 2 k
Then there exists quotient Q such that
5k − 2k = 3Q
The statement for n = k + 1
S (k + 1) : 5k +1 − 2 k +1
= 5 ⋅ 5k − 2 ⋅ 2k
= 5 ⋅ 5k − 5 ⋅ 2k + 5 ⋅ 2k − 2 ⋅ 2k
= 5 ( 5k − 2 k ) + 2 k ( 5 − 2 )
FSc-I / Ex 8.1 - 11

= 5(3Q) + 2k ⋅ 3 Q 5k − 2k = 3 Q
= 3(5Q + 2k )
Clearly S (k + 1) is divisible by 3.
Since the truth for n = k implies the truth for n = k + 1 therefore the given statement
is true for ∀ n ∈¢ + .
Question # 21 (iii)
Same as Question # 21 (ii)
k +1
Hint: S ( k + 1) : 5 −1
= 5 ⋅ 5k − 1 = 5 ⋅5k − 5 + 5 − 1
= 5 ( 5k − 1) + 4 = 5(4Q) − 4 Q 5k − 1 = 4 Q

Question # 21 (iv)
Suppose S (n) : 8 × 10n − 2
Put n = 1
S (1) : 8 × 101 − 2 = 80 − 2 = 78 = 6 × 13
S (1) is clearly divisible by 6, Thus condition I is satisfied
Now suppose that given statement is true for n = k
S (k ) : 8 × 10k − 2
Then there exists quotient Q such that
8 × 10k − 2 = 6 Q
The statement for n = k + 1
S (k + 1) : 8 × 10k +1 − 2
= 8 × 10 ⋅10k − 2
= 8 × 10 ⋅ 10k − 2 ⋅ 10 + 2 ⋅10 − 2 −ing & +ing 2 ⋅ 10
= 10 (8 × 10k − 2 ) + 20 − 2
= 10 ( 6Q ) + 18 Q 8 × 10k − 2 = 6 Q
= 6 (10Q + 3)
Clearly S (k + 1) is divisible by 6.
Since the truth for n = k implies the truth for n = k + 1 therefore the given statement
is true for ∀ n ∈¢ + .
Question # 21 (v)
Suppose S (n) : n 3 − n
Put n = 1
S (1) : 13 − 1 = 0
S (1) i.e. 0 is clearly divisible by 6, Thus condition I is satisfied
Now suppose that given statement is true for n = k
S (k ) : k 3 − k
Then there exists quotient Q such that
k 3 − k = 6Q
The statement for n = k + 1
S (k + 1) : (k + 1)3 − (k + 1)
= k33 + 3k22 + 3k + 1 − k − 1 Since n 2 + n is divisible by 2
= k + 3k + 3k − k
= (k 3 − k ) + 3(k 2 + k ) Therefore n 2 + n = 2 Q′
= 6 Q + 3(2 Q′) Or k 2 + k = 2 Q′
FSc-I / Ex 8.1 - 12

= 6 Q + 6 Q′
Clearly S (k + 1) is divisible by 6.
Since the truth for n = k implies the truth for n = k + 1 therefore the given statement
is true for ∀ n ∈¢ + .

Question # 22
1 1 1 1 1
Suppose S (n) : + 2 + .............. + n = 1 − n 
3 3 3 2 3 
Put n = 1
1 1 1 1 12 1 1
S (1) : = 1 − 1  ⇒ =   ⇒ =
3 2 3  3 2 3 3 3
Thus condition I is satisfied
Now suppose that S (n) is true for n = k
1 1 1 1 1
S (k ) : + 2 + .............. + k = 1 − k  …………. (i)
3 3 3 2 3 
The statement for n = k + 1 becomes
1 1 1 1 1 
S (k + 1) : + 2 + .............. + k +1 = 1 − k +1 
3 3 3 2 3 
1
Adding k on both sides of equation (i)
3
1 1 1 1 1 1 1
+ 2 + .............. + k + k +1 = 1 − k  + k +1
3 3 3 3 2 3  3
1 1 1 1 1 1
⇒ + 2 + .............. + k +1 = − +
3 3 3 2 2 ⋅ 3 3 ⋅ 3k
k

1 1  1 1
= − k − 
2 3  2 3
1 1 3−2 1 1 1
= − k  = − k 
2 3  6  2 3 6
1 1  1 
= 1 − k   
2  3  3
1 1 
= 1 − k +1 
2 3 
Thus S (k + 1) is true if S (k ) is true, so condition II is satisfied and S (n) is true
for all positive integer n.

Question # 23
n−1 (−1)n−1 ⋅ n (n + 1)
Suppose S (n) : 1 − 2 + 3 − 4 + .............. + (−1)
2 2 2 2
⋅n =2

2
Put n = 1
(−1)1−1 ⋅ 1(1 + 1) (−1)0 ⋅ 2
S (1) : 12 = ⇒ 1= ⇒ 1=1
2 2
Thus condition I is satisfied
Now suppose that S (n) is true for n = k
(−1)k −1 ⋅ k (k + 1)
S (k ) : 12 − 2 2 + 32 − 4 2 + .............. + (−1)k −1 ⋅ k 2 = ………… (i)
2
The statement for n = k + 1 becomes
k +1−1 (−1) k +1−1 ⋅ (k + 1)(k + 1 + 1)
S (k + 1) : 1 − 2 + 3 − 4 + .............. + (−1)
2 2 2 2
⋅ (k + 1) =
2

2
FSc-I / Ex 8.1 - 13

(−1)k ⋅ (k + 1)(k + 2)
⇒ 1 − 2 + 3 − 4 + .............. + (−1) ⋅ (k + 1) =
2 2 2 2 k 2

2
Adding (−1) ⋅ (k + 1) on both sides of equation (i)
k 2

k −1 (−1)k −1 ⋅ k (k + 1)
1 − 2 + 3 − 4 + .............. + (−1) ⋅ k + (−1) ⋅ (k + 1) =
2 2 2 2 2 k 2
+ (−1)k ⋅ (k + 1) 2
2
k −1
(−1) ⋅ k (k + 1) + 2(−1)k ⋅ (k + 1)2
⇒ 1 − 2 + 3 − 4 + .............. + (−1) ⋅ (k + 1) =
2 2 2 2 k 2

2
(−1) (k + 1) (−1) −1 k + 2(k + 1) 
k

=
2
(−1) (k + 1) [ − k + 2k + 2 ]
k
=
2
(−1) (k + 1) ( k + 2 )
k
=
2
Thus S (k + 1) is true if S (k ) is true, so condition II is satisfied and S (n) is true
for all positive integer n.

Question # 24
Suppose S (n) : 13 + 33 + 53 + .............. + (2n − 1)3 = n2 (2n2 − 1)
Put n = 1
S (1) : 13 = 12 ( 2(1)2 − 1) ⇒ 1 = 1(2 − 1) ⇒ 1 = 1
Thus condition I is satisfied
Now suppose that S (n) is true for n = k
S (k ) : 13 + 33 + 53 + .............. + (2k − 1)3 = k 2 (2k 2 − 1) …………. (i)
The statement for n = k + 1 becomes
S (k + 1) : 13 + 33 + 53 + .............. + (2(k + 1) − 1)3 = (k + 1)2 (2(k + 1)2 − 1)
⇒ 13 + 33 + 53 + .............. + (2k + 1)3 = (k 2 + 2k + 1)(2(k 2 + 2k + 1) − 1)
= (k 2 + 2k + 1)(2k 2 + 4k + 2 − 1)
= (k 2 + 2k + 1)(2k 2 + 4k + 1)
= 2k 4 + 4k 3 + 2k 2 + 4k 3 + 8k 2 + 4k + k 2 + 2k + 1
= 2k 4 + 8k 3 + 11k 2 + 6k + 1
Adding (2k + 1)3 on both sides of equation (i)
S (k ) : 13 + 33 + 53 + .............. + (2k − 1)3 + (2k + 1)3 = k 2 (2k 2 − 1) + (2k + 1)3
⇒ 13 + 33 + 53 + .............. + (2k + 1)3 = k 2 (2k 2 − 1) + (2k )3 + 3(2k )2 (1) + 3(2k )(1)2 + (1)3
⇒ 13 + 33 + 53 + .............. + (2k + 1)3 = 2k 4 − k 2 + 8k 3 + 12k 2 + 6k + 1
= 2k 4 + 8k 3 + 11k 2 + 6k + 1
Thus S (k + 1) is true if S (k ) is true, so condition II is satisfied and S (n) is true
for all positive integer n.

Made by: Atiq ur Rehman ( atiq@mathcity.org), http://www.mathcity.org


FSc-I / Ex 8.1 - 14

Question # 25
Suppose S (n) : x 2 n − 1
Put n = 1
S (1) : x 2(1) − 1 = x 2 − 1 = ( x − 1)( x + 1)
x + 1 is clearly factor of S (1) , Thus condition I is satisfied
Now suppose that given statement is true for n = k
S (k ) : x 2k − 1
Then there exists quotient Q such that
x 2 k − 1 = ( x + 1) Q
The statement for n = k + 1
S (k + 1) : x 2( k +1) − 1
= x 2 k +2 − 1
= x 2 k +2 − x 2 k + x 2 k − 1 +ing and –ing x 2k
= x 2 k ( x 2 − 1) + ( x 2 k − 1)
= x 2 k ( x − 1)( x + 1) + ( x + 1) Q Q x 2 k − 1 = ( x + 1) Q
= ( x + 1) ( x 2 k ( x − 1) + Q )
Clearly x + 1 is a factor of S (k + 1) .
Since the truth for n = k implies the truth for n = k + 1 therefore the given statement
is true for ∀ n ∈¢ + .

Question # 26
Suppose S (n) : x n − y n
Put n = 1
S (1) : x1 − y1 = x − y
x − y is clearly factor of S (1) , Thus condition I is satisfied
Now suppose that given statement is true for n = k
S (k ) : x k − y k
Then there exists quotient Q such that
x k − y k = ( x − y) Q
The statement for n = k + 1
S (k + 1) : x k +1 − y k +1
= x ⋅ xk − y ⋅ y k
= x ⋅ xk − x ⋅ y k + x ⋅ yk − y ⋅ yk –ing & +ing x y k
= x( x k − y k ) + y k ( x − y )
= x ( x − y )Q + y k ( x − y ) Q x k − y k = ( x − y) Q
Clearly x − y is a factor of S (k + 1) .
Since the truth for n = k implies the truth for n = k + 1 therefore the given statement
is true for ∀ n ∈¢ + .

Question # 27
Suppose S (n) : x 2 n−1 + y 2 n−1
Put n = 1
S (1) : x 2(1)−1 + y 2(1)−1 = x1 + y1 = x + y
x + y is clearly factor of S (1) , Thus condition I is satisfied
Now suppose that given statement is true for n = k
S (k ) : x 2 k −1 + y 2 k −1
Then there exists quotient Q such that
FSc-I / Ex 8.1 - 15

x 2 k −1 + y 2 k −1 = ( x + y ) Q
The statement for n = k + 1
S (k + 1) : x 2( k +1)−1 + y 2(k +1)−1
= x 2 k +2−1 + y 2 k + 2−1
= x 2 k +2−1 − x 2 k −1 y 2 + x 2 k −1 y 2 + y 2 k +2−1 +ing and –ing x 2 k −1 y 2
= x 2 k −1 ( x 2 − y 2 ) + y 2 ( x 2 k −1 + y 2 k −1 )
= x 2 k −1 ( x − y )( x + y ) + y 2 ( x + y ) Q Q x 2 k −1 + y 2 k −1 = ( x + y ) Q
= ( x + y ) ( x 2 k −1 ( x − y ) + y 2 Q )
Clearly x + y is a factor of S (k + 1) .
Since the truth for n = k implies the truth for n = k + 1 therefore the given statement
is true for ∀ n ∈¢ + .
Principle of Extended Mathematical Induction
A given statement S (n) is true for n ≥ i if the following two conditions hold
Condition I: S (i ) is true i.e. S (n) is true for n = i and
Condition II: S (k + 1) is true whenever S (k ) is true for any positive integer k,
Then S (n) is true for all positive integers
Question # 28
Suppose S (n) : 1 + 2 + 22 + .............. + 2n = 2 n+1 − 1
Put n = 0 Note: Non- negative number are
S (1) : 1 = 20+1 − 1 = ⇒ 1 = 2 − 1 ⇒ 1 = 1 0,1, 2,3,...........
Thus condition I is satisfied
Now suppose that S (n) is true for n = k
S (k ) : 1 + 2 + 22 + .............. + 2k = 2k +1 − 1 …………. (i)
The statement for n = k + 1 becomes
S (k + 1) : 1 + 2 + 2 2 + .............. + 2 k +1 = 2 k +1+1 − 1
= 2 k +2 − 1
Adding 2 k +1 on both sides of equation (i)
1 + 2 + 22 + .............. + 2k + 2k +1 = 2k +1 − 1 + 2 k +1
⇒ 1 + 2 + 4 + .............. + 2k +1 = 2(2k +1 ) − 1 Q 2k +1 + 2k +1 = 2(2k +1 )
= 2 k +1+1 − 1
= 2 k +1+1 − 1
Thus S (k + 1) is true if S (k ) is true, so condition II is satisfied and S (n) is true
for all non-negative integers n.
Question # 29
Suppose S (n) : AB n = B n A
Put n = 1
S (1) : AB1 = B1 A ⇒ AB = BA
S (1) is true as we have given AB = BA , Thus condition I is satisfied
Now suppose that given statement is true for n = k
S (k ) : AB k = B k A ……………. (i)
The statement for n = k + 1
S (k + 1) : AB k +1 = B k +1 A
Post-multiplying equation (i) by B.
( AB k ) B = ( B k A) B
⇒ A( B k B) = B k ( A B) by associative law
FSc-I / Ex 8.1 - 16

⇒ AB k +1 = B k ( BA) Q AB = BA (given)
= ( B k B) A
= B k +1 A
Thus S (k + 1) is true if S (k ) is true, so condition II is satisfied and S (n) is true
for all positive integers n.
Question # 30
Suppose S (n) : n 2 − 1
Put n = 1
S (1) : (1)2 − 1 = 0
S (1) is clearly divisible by 8, Thus condition I is satisfied
Now suppose that given statement is true for n = k where k is odd.
S (k ) : k 2 − 1
Then there exists quotient Q such that
k 2 −1 = 8 Q
As k + 2 is the next odd integer after k The statement for n = k + 1
S (k + 2) : ( k + 2 ) − 1
2

= k 2 + 4k + 4 − 1
= k 2 − 1 + 4k + 4
= 8 Q + 4(k + 1) Q k 2 + k = 2Q
Since k is odd therefore k + 1 is even so their exists integer t such that k + 1 = 2t
⇒ S (k + 2) : = 8 Q + 4(2t )
= 8Q + 8t
Clearly S (k + 2) is divisible by 8 so condition II is satisfied.
Therefore the given statement is true for odd positive integers.
Question # 31
Suppose S (n) : ln x n = n ln x
Put n = 1
S (1) : ln x1 = (1)ln x ⇒ ln x = ln x
S (1) is true so condition I is satisfied.
Now suppose that given statement is true for n = k
S (k ) : ln x k = k ln x ……………. (i)
The statement for n = k + 1
S (k + 1) : ln x k +1 = (k + 1)ln x
Now adding ln x on both sides of equation (i)
ln x k + ln x = k ln x + ln x
⇒ ln x k ⋅ x = (k + 1)ln x Q ln x + ln y = ln x y
⇒ ln x k +1 = (k + 1)ln x
Thus S (k + 1) is true if S (k ) is true, so condition II is satisfied and S (n) is true
for all n ∈¢ + .
Question # 32
Suppose S (n) : n ! > 2 n − 1 ; n≥4
Put n = 4
S (4) : 4! > 2 4 − 1 ⇒ 24 > 16 − 1 ⇒ 24 > 15
S (4) is true so condition I is satisfied.
Now suppose that given statement is true for n = k
S (k ) : k ! > 2 k − 1 ……………. (i)
FSc-I / Ex 8.1 - 17

The statement for n = k + 1


S (k + 1) : (k + 1)! > 2k +1 − 1
Multiplying both sides of equation (i) by k + 1
(k + 1)k ! > (k + 1)(2k − 1)
⇒ (k + 1)! > (k + 1 + 2 − 2)(2k − 1) Q ( k + 1) k ! = (k + 1)!
⇒ (k + 1)! > (k − 1 + 2)(2k − 1)
⇒ (k + 1)! > k ⋅ 2k − k − 2k + 1 + 2 ⋅ 2k − 2
⇒ (k + 1)! > (k ⋅ 2k − 2k − k ) + 2k +1 − 1
⇒ (k + 1)! > 2k +1 − 1 Q k ⋅ 2k − 2k − k ≥ 0 ∀ k ≥ 4
Thus S (k + 1) is true if S (k ) is true, so condition II is satisfied and S (n) is true
for all integers n ≥ 4 .
Question # 33
Suppose S (n) : n 2 > n + 3 ; n≥3
Put n = 3
S (3) : 32 > 3 + 3 ⇒ 9 > 6
S (3) is true so condition I is satisfied.
Now suppose that given statement is true for n = k
S (k ) : k 2 > k + 3 ……………. (i)
The statement for n = k + 1
S (k + 1) : (k + 1) 2 > k + 1 + 3 ⇒ (k + 1)2 > k + 4
Adding 2k + 1 on both sides of equation (i)
k 2 + 2k + 1 > k + 3 + 2 k + 1
⇒ (k + 1)2 > k + 4 + 2k
⇒ (k + 1)2 > k + 4 ignoring 2k as 2k > 0
Thus S (k + 1) is true if S (k ) is true, so condition II is satisfied and S (n) is true
for all integers n ≥ 3 .
Question # 34
Suppose S (n) : 4 n > 3n + 2n−1 ; n≥2
Put n = 2
S (2) : 4 2 > 32 + 2 2−1 ⇒ 16 > 9 + 2 ⇒ 16 > 11
S (2) is true so condition I is satisfied.
Now suppose that given statement is true for n = k
S (k ) : 4 k > 3k + 2k −1 ……………. (i)
The statement for n = k + 1
S (k + 1) : 4 k +1 > 3k +1 + 2 k +1−1
⇒ 4k +1 > 3k +1 + 2k
Multiplying both sides of equation (i) by 4.
4(4k ) > 4(3k + 2k −1 )
⇒ 4k +1 > 4 ⋅ 3k + 4 ⋅ 2k −1
⇒ 4k +1 > (3 + 1) ⋅ 3k + (2 + 2) ⋅ 2k −1
⇒ 4k +1 > 3 ⋅ 3k + 3k + 2 ⋅ 2k −1 + 2 ⋅ 2k −1
⇒ 4k +1 > 3k +1 + 2k + (3k + 2k )
⇒ 4k +1 > 3k +1 + 2k ignoring 3k + 2 k as 3k + 2k > 0
Thus S (k + 1) is true if S (k ) is true, so condition II is satisfied and S (n) is true
for all integers n ≥ 3 .
FSc-I / Ex 8.1 - 18

Question # 35
Suppose S (n) : 3n < n! ; n>6
Put n = 7
S (7) : 37 < 7! ⇒ 2187 < 5040
S (2) is true so condition I is satisfied.
Now suppose that given statement is true for n = k
S (k ) : 3k < k ! ……………. (i)
The statement for n = k + 1
S (k + 1) : 3k +1 < (k + 1)!
Multiplying both sides of equation (i) by k + 1 .
(k + 1)3k < (k + 1) k !
⇒ ( (k − 2) + 3) 3k < (k + 1)!
⇒ (k − 2)3k + 3k +1 < (k + 1)!
⇒ 3k +1 < (k + 1)! Q (k − 2)3k > 0 ∀ k > 6
Thus S (k + 1) is true if S (k ) is true, so condition II is satisfied and S (n) is true
for all integers n > 6 .

Question # 36
Suppose S (n) : n ! > n 2 ; n≥4
Put n = 4
S (4) : 4! > 42 ⇒ 24 > 16
S (4) is true so condition I is satisfied.
Now suppose that given statement is true for n = k
S (k ) : k ! > k 2 ……………. (i)
The statement for n = k + 1
S (k + 1) : (k + 1)! > (k + 1) 2
Multiplying both sides of equation (i) by k + 1 .
(k + 1) k ! > (k + 1)k 2
⇒ (k + 1)! > (k + 1)(k + 1) Q k +1 < k2 ∀ k ≥ 4
⇒ (k + 1)! > (k + 1)2
Thus S (k + 1) is true if S (k ) is true, so condition II is satisfied and S (n) is true
for all integers n ≥ 4 .
Question # 37
Suppose S (n) : 3 + 5 + 7 + .............. + (2n + 5) = (n + 2)(n + 4) ; n ≥ −1
Put n = −1
S (−1) : 3 = (−1 + 2)(−1 + 4) ⇒ 3 = (1)(3) ⇒ 3 = 3
Thus condition I is satisfied
Now suppose that S (n) is true for n = k
S (k ) : 3 + 5 + 7 + .............. + (2k + 5) = (k + 2)(k + 4) …………. (i)
The statement for n = k + 1 becomes
S (k + 1) : 3 + 5 + 7 + .............. + (2(k + 1) + 5) = ( (k + 1) + 2 )( (k + 1) + 4 )
⇒ 3 + 5 + 7 + .............. + (2k + 7) = ( k + 3)( k + 5 )
Adding (2 k + 7) on both sides of equation (i)
S (k ) : 3 + 5 + 7 + .............. + (2k + 5) + (2 k + 7) = (k + 2)(k + 4) + (2k + 7)
⇒ 3 + 5 + 7 + .............. + (2k + 7) = k 2 + 2k + 4k + 8 + 2k + 7
= k 2 + 8k + 15
= k 2 + 5k + 3k + 15
FSc-I / Ex 8.1 - 19

= k (k + 5) + 3(k + 5)
= (k + 5)(k + 3)
Thus S (k + 1) is true if S (k ) is true, so condition II is satisfied and S (n) is true
for all integers n ≥ −1 .
Question # 38
Suppose S (n) : 1 + nx ≤ (1 + x)n ; n≥2
Put n = 2
S (2) : 1 + 2 x ≤ (1 + x) 2 ⇒ 1 + 2 x ≤ 1 + 2 x + x 2
S (2) is true so condition I is satisfied.
Now suppose that given statement is true for n = k
S (k ) : 1 + kx ≤ (1 + x )k ……………. (i)
The statement for n = k + 1
S (k + 1) : 1 + (k + 1) x ≤ (1 + x) k +1
Multiplying both sides of equation (i) by 1 + x .
(1 + kx)(1 + x) ≤ (1 + x )k (1 + x )
⇒ 1 + kx + x + kx 2 ≤ (1 + x) k +1
⇒ 1 + kx + x ≤ (1 + x) k +1 Q kx 2 > 0
⇒ 1 + (k + 1) x ≤ (1 + x)k +1
Thus S (k + 1) is true if S (k ) is true, so condition II is satisfied and S (n) is true
for all integers n ≥ 2

Made by: Atiq ur Rehman ( atiq@mathcity.org ), http://www.mathcity.org

If you found any error, submit at


http://www.mathcity.org/error

Error Analyst

Waiting for someone

Tuesday, March 22, 2005 *


Updated: March 4, 2006 *
mathcity.org Exercise 8.2 (Solutions)
Textbook of Algebra and Trigonometry for Class XI
Page 273

Merging man and maths Available online @ http://www.mathcity.org, Version: 1.3.1

Ð Binomial Theorem
If a and x are two real number and n is a positive integer then
(a + x)n =   a n +   a n −1 x1 +   a n−2 x 2 + ............... + 
n n n n  n−1  n  n
a x +  n x
0 1  2  n − 1  
Proof
We will use mathematical induction to prove this so let S (n) be the given statement.
Put n = 1
S (1) : (a + x)1 =   a1 +   a1−1 x1 = (1) a + (1)(1) x ⇒ a + x = a + x
1 1
0  1
S (1) is true so condition I is satisfied.
Now suppose that S (n) is true for n = k .

S (k ) : (a + x)k =   a k +   a k −1 x1 +   a k −2 x 2 + ............... + 
k k k k  k −1  k  k
 a x +  k  x …..(i)
0 1  2  k − 1  
The statement for n = k + 1
k + 1 k +1  k + 1 k +1−1 1  k + 1 k +1− 2 2
S (k + 1) : (a + x)k +1 =  a +  1 a x + a x + ...............
 0     2 
k + 1  k +1−1  k + 1 k +1
+  a x + x
 k + 1 − 1  k + 1
k + 1 k +1  k + 1 k 1  k + 1 k −1 2
⇒ (a + x)k +1 =   a +  1  a x +  2  a x + ...............
 0     
k + 1 k  k + 1 k +1
+   a x +  k + 1 x
 k   
Multiplying both sides of equation (i) by (a + x)
 k k  k −1  k  k 
(a + x) k (a + x) =    a k +   a k −1 x1 +   a k −2 x 2 + ............... + 
k k
 a x +  k  x  ( a + x)
 0 1 2  k − 1   
 k k  k −1  k  k 
⇒ (a + x)k +1 =    a k +   a k −1 x1 +   a k −2 x 2 + ............... + 
k k
 a x +  k  x  ( a)
 0 1 2  k − 1   
 k k  k −1  k  k 
+    a k +   a k −1 x1 +   a k −2 x 2 + ............... + 
k k
 a x +  k  x  ( x)
0 1 2  k − 1   
⇒ (a + x)k +1 =   a k +1 +   a k x1 +   a k −1 x 2 + ............... + 
k k k k  2 k −1  k  k
a x +  k a x
0 1 2  k − 1  
+   a k x +   a k −1 x 2 +   a k −2 x3 + ............... + 
k k k k  k  k  k +1
a x + k x
 
0  
1  
2  − 1
k  
 k k   k k 
⇒ (a + x)k +1 =   a k +1 +    +    a k x1 +    +    a k −1 x 2 + ...............
k
0  1   0  2  1
 k k   k  k  k +1
+    +  a x +  k  x
  k   k − 1   
n + 1   n   n   n + 1 n + 1
Since   =  and   = 
n n
 ,  +  =  
 0   0   r   r − 1  r   n   n + 1
k + 1 k +1  k + 1 k 1  k + 1 k −1 2
⇒ (a + x)k +1 =   a +  1  a x +  2  a x + ...............
 0     
k + 1 k  k + 1 k +1
+   a x +  k + 1 x
 k   
Thus S (k + 1) is true when S (k ) is true so condition II is satisfied and S (n) is true for all
positive integral value of n.
FSc-I / Ex 8.2 - 2

Ð Question # 1 (i)
(a + 2b)5 = () ()
5 5 5 5−1
0
a +
1
a (2b )1 +
5 5− 2
2
a (2b) 2 +
5 5 −3
3
a (2b) 3 +
5 5−4
4 ()
a (2b) 4 +
5 5−5
5
a (2b) 5 () () ()
= (1) a 5 + (5) a 4 (2b) + (10) a3 (4b 2 ) + (10) a 2 (8b3 ) + (5) a1 (16b4 ) + (1) a 0 (32b5 )
= a5 + 10 a 4b + 40 a3b 2 + 80 a 2b3 + 80 ab4 + 32b5 Q a0 = 1
Ð Question # 1 (ii)
6
 x 2   6  x   6  x   2   6  x   2   6  x   2 
6 6 −1 1 6− 2 2 6 −3 3

 − 2  =    +     − 2  +     − 2  +     − 2 
 2 x   0  2   1  2   x   2  2   x   3  2   x 
 6 x   2   6 x   2   6 x   2 
6− 4 4 6−5 5 6− 6 6

+    − 2  +     − 2  +     − 2 
 4 2   x   5 2   x   6 2   x 
6 5 4 2 3 3
x x  2  x  2  x  2 
= (1)   − (6)    2  + (15)    2  − (20)    2 
2 2  x  2  x  2  x 
2 4 1 5 6
 x  2   x  2   2
+ (15)    2  − (6)    2  + (1)(1)  2 
2  x  2  x  x 
 x   x  2 
6 5
 x  4 
4
 x  8 
3
=   − 6    2  + 15    4  − 20    6 
 64   32   x   16   x   8  x 
 x 2   16   x  32   64 
+15   8  − 6   10  +  12 
 4   x   2  x   x 
x 6 3 x 3 15 20 60 96 64
= − + − 3 + 6 − 9 + 12
64 8 4 x x x x
Question # 1 (iii), (iv) and (v)
Do yourself

Ð Question # 1 (vi)
6 −1 6− 2
 a x   6 a   6 a   x   6  a   x
6 6 1 2

 − =
   +
     −  +    − 
 x a   0 x   1  x   a   2 x   a
6 −3 6− 4 6−5 6− 6
 6  a   x   6 a   x   6 a   x   6 a   x
3 4 5 6

+    −  +    −  +    −  +    − 
 3  x   a   4  x   a   5  x   a   6  x   a 
 a  a  x  a  x
6 5 1 4 2

= (1)   − (6)     + (15)    


 x  x  a  x  a
 a  x  a  x  a x  a  x
3 3 2 4 1 5 0 6

−(20)     + (15)     − (6)    + (1)    


 x  a  x  a  x a  x  a
−1 −2
 a  a  a  a  a
6 5 4

=  − 6   + 15    
 x  x  x  x  x
−3 −2 −1
 a  a  x  x  x  x  x
3 4 5 6

−20     + 15     − 6    + 1(1)  
 x  x  a  a  a  a  a
5 −1 4−2 3− 3 −2 + 4 −1+ 5
 a  a  a  a  x  x  x
6 6

=  − 6  + 15   − 20   + 15   − 6  + 1 
 x  x  x  x  a  a  a
 a  a  a  a  x  x  x
6 4 2 0 2 4 6

=  − 6  + 15   − 20   + 15   − 6  + 
 x  x  x  x  a  a  a
FSc-I / Ex 8.2 - 3

           
1 6 1 1 4 21 1 1 2 4 6

  a     a     a     x     x    x 
+   

2 2 2 2 2 2
=   −6   + 15   − 20 (1) + 15   −6  
 x    x    x    a    a    a  
           
3 2 2 3
a a a x x x
=   − 6   + 15   − 20 + 15   − 6   +  
x  x x a a a
3 2 2
a a a x x x3
= 3 − 6 2 + 15 − 20 + 15 − 6 2 + 3
x x x a a a
Ð Question # 2 (i)
(0.97)3 = (1 − 0.03)3
3  3 3  3
=   (1)3 +   (1)2 (−0.03) + +   (1)1 (−0.03)2 + +   (−0.03)3
0 1 2  3
= (1)(1) + 3(1)(−0.03) + 3(1)(0.0009) + + (1)(−0.000024)
= 1 − 0.09 + 0.0027 − 0.000027 = 0.912673

Ð Question # 2 (ii)
(2.02) 4 = (2 + 0.02)4 Now do yourself.

Ð Question # 2 (iii)
(9.98)4 = (10 − 0.02) 4
 4 4  4 4 4
=   (10) 4 +   (10)3 (−0.02) +   (10)2 (−0.02)2 +   (10)1 (−0.02)3 +   (10)0 (−0.02)4
0 1  2 3 4
= (1)(10000) + 4(1000)(−0.02) + 6(100)(0.0004) + 4(10)(−0.000008) + (1)(1)(0.00000016)
= 10000 − 80 + 0.24 − 0.00032 + 0.00000016 = 9920.23968

Ð Question # 2 (iv)
(2.1)5 = (2 + 0.1)5

Ð Question # 3(i)
( ) ( )
4 4
a + 2x + a − 2x
We take

(a + )
2 x =   a 4 +   a 3
4 4
( ) 4
( )  4
( ) 4
( )
4 1 2 3 4
2 x +   a2 2x +   a1 2 x +   a0 2x
0 1  2  3 4
= (1) a 4 + ( 4 ) a 3 ( ) ( ) ( )
2 x + ( 6 ) a 2 2 x 2 + ( 4 ) a 2 2 x 3 + (1)(1) 4 x 4 ( )
( )
4
⇒ a + 2x = a 4 + 4 2a 3 x + 12a 2 x 2 + 8 2 a x3 + 4 x 4 …………. (i)
Replacing 2 by − 2 in eq. (i)
( ) ( ) ( )
4
a − 2x = a 4 + 4 − 2 a 3 x + 12a 2 x 2 + 8 − 2 a x 3 + 4 x 4
= a 4 − 4 2a3 x + 12a 2 x 2 − 8 2 a x 3 + 4 x 4 ……………. (ii)
Adding (i) & (ii)
(a + ) + (a − )
4 4
2x 2x = 2 a 4 + 24 a 2 x 2 + 8 x 4

Ð Question # 3 (ii)
Do yourself.
FSc-I / Ex 8.2 - 4

Ð Question # 3 (iii)
Since
 5  5 5 5 5  5
(2 + i )5 =   25 +   25−1 i +   25−2 i 2 +   25−3 i 3 +   25−4 i 4 +   25−5 i 5
 0 1  2  3 4  5
= (1) 2 + ( 5) 2 i + (10 ) 2 i + (10 ) 2 i + ( 5) 2 i + (1) 2 i
5 4 3 2 2 3 1 4 0 5

= 32 + 80i + 80i 2 + 40i 3 + 10i 4 + i 5 ……….. (i)


Replacing i by −i in eq. (i)
(2 + i )5 = 32 + 80 ( −i ) + 80 ( −i ) + 40 ( −i ) + 10 ( −i ) + ( −i )
2 3 4 5

= 32 − 80i + 80i 2 − 40i 3 + 10i 4 − i 5


Subtracting (i) & (ii)
(2 + i)5 − (2 − i)5 = 160i + 80i 3 + 2i 5
= 160i + 80(−1) ⋅ i + 2(−1)2 ⋅ i
= 160i − 80i + 2i = 82i

Ð Question # 3 (iv)

(x + ) ( )
3 3
x −1 + x + x −1
2 2

Suppose t = x 2 − 1 then

(x + ) ( )
3 3
x −1 + x + x −1 = ( x + t ) + ( x + t )
2 2 3 3

= ( ( x )3 + 3( x )2 (t ) + 3( x)(t )2 + (t )3 ) + ( ( x )3 + 3( x )2 (−t ) + 3( x )(−t )2 + (−t )3 )


= x 3 + 3x 2t + 3xt 2 + t 3 + x 3 − 3x 2t + 3xt 2 − t 3
= 2 x 3 + 6 xt 2

( )
2
= 2 x3 + 6 x x2 − 1 Q t = x2 − 1

= 2 x 3 + 6 x ( x 2 − 1) = 2 x 3 + 6 x 3 − 6 x = 8x 3 − 6 x

Ð Question # 4 (i)
(2 + x − x 2 ) 4
Put t = 2 + x then
(2 + x − x 2 ) 4 = (t − x 2 )4
 4  4 4  4  4
=   (t ) 4 +   (t )3 (− x 2 ) +   (t )2 (− x 2 )2 +   (t )1 (− x 2 )3 +   (t )0 (− x 2 )4
0 1 2  3  4
= (1) (t )4 − ( 4 ) (t )3 ( x 2 ) + ( 6 ) (t )2 ( x 4 ) − ( 4 ) (t )( x 6 ) + (1) (1)( x8 )
= t 4 − 4t 3 x 2 + 6t 2 x 4 − 4t x 6 + x8 ………………… (i)
Now
4 4  4  4 4
t 4 = (2 + x) 4 =   (2)4 +   (2)3 ( x ) +   (2) 2 ( x) 2 +   (2)1 ( x)3 +   (2)0 ( x)4
0 1  2  3 4
= (1) (16) + ( 4 ) (8)( x) + ( 6 ) (4)( x 2 ) + ( 4 ) (2)( x 3 ) + (1) (1)( x 4 )
= 16 + 32 x + 24 x 2 + 8 x3 + x 4
Also
t 3 = (2 + x)3 = (2)3 + ( 3) (2) 2 ( x) + ( 3) (2)1 ( x )2 + ( x)3
= 8 + 12 x + 6 x 2 + x3
t 2 = (2 + x) 2 = 4 + 4 x + x 2
Putting values of t 4 , t 3 , t 2 and t in equation (i)
(2 + x − x 2 )4 = (16 + 32 x + 24 x 2 + 8 x 3 + x 4 ) − 4(8 + 12 x + 6 x 2 + x3 ) x 2
+6(4 + 4 x + x 2 ) x 4 − 4(2 + x ) x 6 + x8
FSc-I / Ex 8.2 - 5

= 16 + 32 x + 24 x 2 + 8 x 3 + x 4 − 32 x 2 − 48 x3 − 24 x 4 − 4 x 5
+24 x 4 + 24 x 5 + 6 x 6 − 8 x 6 + 4 x 7 + x8
= 16 + 32 − 8 x 2 − 40 x 3 + x 4 + 20 x 5 − 2 x 6 − 4 x 7 − x8
Ð Question # 4 (ii)
Suppose t = 1 − x Do yourself

Ð Question # 4 (iii)
Suppose t = 1 − x Do yourself

Ð Question # 5 (i)
Suppose t = x − 1 Do yourself

Ð Question # 5 (ii)
3
 1
 x −1− 
 x
Suppose t = x − 1 then
3 2 3
 1 2 1  1  1
 t −  = (t ) + 3(t )  −  + 3(t )  −  +  − 
3

 x  x  x  x
1 1 1
= t 3 − 3t 2 ⋅ + 3t ⋅ 2 − 3 ………… (i)
x x x
Now
t 3 = ( x − 1) = ( x )3 + 3( x )2 (−1) + 3( x)(−1)2 + (−1)3
3

= x3 − 3x 2 + 3x − 1
t 2 = ( x − 1) = x 2 − 2 x + 1
2

Putting values of t 3 , t 2 and t in equation (i)


3
 1 1 1 1
 x − 1 −  = ( x − 3 x + 3 x − 1) − 3( x − 2 x + 1) ⋅ + 3( x − 1) ⋅ 2 − 3
3 2 2

 x x x x
1 1 1 1
= x3 − 3x 2 + 3x − 1 − 3x + 6 − 3 + 3 − 3 2 − 3
x x x x
3 1
= x3 − 3x 2 + 5 − 2 − 3
x x
Ð Question # 6 (i)
Since
n
Tr +1 =   a n−r x r
r
Here a = 3 , x = −2 x , n = 7 so
7 7
Tr +1 =   (3)7 −r (−2 x )r =   (3)7−r (−2)r ( x) r
r r
For term involving x 4 we must have
xr = x4 ⇒ r = 4
So
7
T4+1 =   (3)7−4 (−2)4 ( x)4
4
⇒ T5 = ( 35) (3)3 (−2) 4 ( x) 4 = ( 35) (27)(16)( x )4
= 15120x 4

Made By: Atiq ur Rehman, (atiq@mathcity.org) http://www.mathcity.org


FSc-I / Ex 8.2 - 6

Ð Question # 6 (ii)
n
Since Tr +1 =   a n−r x r
r
2
Here a = x , x = − 2 , n = 13 so
x
r
 13  13−r  2  13  13−r
Tr +1 =   ( x)  − 2  =   ( x) ( −2 ) ( x)−2 r
r

r  x  r
13  13 
=   ( x)13−r −2 r ( −2 ) =   ( x )13−3r ( −2 )
r r

r r
For term involving x −2 we must have
x13−3r = x −2 ⇒ 13 − 3r = −2 ⇒ − 3r = −2 − 13
⇒ − 3r = −15 ⇒ r = 5
So
13 
T5+1 =   ( x )13−3(5) ( −2 )
5

5
⇒ T6 = (1287)( x)13−15 ( −32 ) = − 41184 x −2
Ð Question # 6 (iii)
Since
n
Tr +1 =   a n−r x r
r
2
Here a = , x = − a , n = 9 so
x
9− r 9− r
 9 2   9 2 
Tr +1 =     (− a ) =     (−1)r (a ) r
r

 r  x   r  x 
For term involving a 4 we must have
a r = a4 ⇒ r = 4
9− 4
9 2
So T4+1 =     (−1)4 (a ) 4
 4  x 
5
2  32  4 a4
⇒ T5 = (126)   (1) a = (126)  5  a = 4032 5
4

x x  x
Ð Question # 6 (iii)
Here a = x , x = − y , n = 11 so
Since
n
Tr +1 =   a n−r x r
r
11 11−r
( )
11 11−r
( )
r
Tr +1 =   ( x ) − y =  ( x)
r 1
− y2
r r
11 11−r
=   ( x ) (−1)r y 2 ( )
r

r
For term involving y 3 we must have
r r
y 2 = y3 ⇒ =3 ⇒ r =6
2
So
11  11−6
T6+1 =   ( x ) (−1)6 y 2
6
6

( )
( )
⇒ T7 = ( 462 )( x ) (1) y 3 = 462 x5 y 3
5
FSc-I / Ex 8.2 - 7

Ð Question # 7 (i)
3
Here a = x 2 , x = − , n = 10 so
2x
Since
n
Tr +1 =   a n−r x r
r
r
 10  2 10−r  3  10  2(10−r ) (3) r
Tr +1 =   ( x )  −  =   ( x ) (−1) r

r  2x   r  (2) r ( x)r


10  20−2 r 10  20−2 r −r
=  ( x) (−1)r (3)r (2) − r ( x)− r =   ( x ) (−1) r (3)r (2)− r
 r  
r
10  20−3r
=  ( x) (−1) r (3) r (2)− r
r
For term involving x 5 we must have
x 20 − 3r = x 5 ⇒ 20 − 3r = 5 ⇒ − 3r = 5 − 20
⇒ − 3r = −15 ⇒ r = 5
10  20−3(5)
So T5+1 =   ( x ) (−1)5 (3)5 (2)−5
5
1 1
⇒ T6 = 252 ( x ) (−1)5 (3)5 5 = − 252 ( x ) (243)
20−15 5

2 32
61236 5 15309 5
=− x =− x
32 8
15309
Hence coefficient of x 5 = −
8
Ð Question # 7 (ii)
1
Here a = x 2 , x = − , n = 2n so
x
Since
n
Tr +1 =   a n−r x r
r
r
 2n  2 2 n −r  1   2n  2(2 n−r )
Tr +1 =   ( x )  −  =   ( x )
r 1
( −1) r
 r   x  r  x
 2n  4 n− 2 r  2n  4 n− 2 r − r
=  ( x ) ( −1) x − r =   ( x ) ( −1)
r r

 r   r 
 2n  4 n −3 r
=  ( x) ( −1)
r

 r 
For term involving x n we must have
x 4 n−3 r = x n ⇒ 4n − 3r = n ⇒ − 3r = n − 4n
⇒ − 3r = − 3n ⇒ r = n
 2 n  4 n −3 n
Tn+1 =   ( x ) ( −1)
n
So
 n
(2n)! (2n)! n
= ( x ) ( −1) = ( x ) ( −1)
n n n

(2n − n)!⋅ n! n!⋅ n!


n (2n)! n
= ( −1) x
(n !)2
n (2n)!
Hence coefficient of x n = ( −1)
(n!)2
FSc-I / Ex 8.2 - 8

Ð Question # 8
3
Here a = x 2 , x = − , n = 10 and r + 1 = 6 ⇒ r = 5 so
2x
Since
n
Tr +1 =   a n−r x r
r
5
10  2 10−5  3 
T5+1 =   ( x )  − 
5  2x 
2 5 35   243 
⇒ T6 = 252 ( x )  − 5 
= 252 x10  − 5 
 (2 x)   32 x 
61236 10−5 15309 5
=− x =− x
32 8
Ð Question # 9 (i)
Do yourself as Q # 9 (ii)
Ð Question # 9 (ii)
1
Here a = x , x = , n = 10 so
2 x2
Since
n
Tr +1 =   a n−r x r
r

( )
10− r

( )
r
10− r  1   1 
10  10  12
Tr +1 =   x   =  x  r 2r 
r  2x   r  2 x 
2

10  1
(10− r ) 1 10  5 − 2 1 −2 r
r
=  ( x)2 x −2 r
=  r ( x) x
r 2r   2r
10  5 − 2r − 2 r 1 10  5 − 52r 1
=  ( x) =  ( x)
r 2r  r  2r
For term independent of x we must have
5r
5− 5r 5r
x 2 = x 0 ⇒ 5 − = 0 ⇒ − = −5
2 2
 2
⇒ r = (−5)  −  ⇒ r = 2
 5
10  5 − 5(2) 1
So T2+1 =   ( x ) 2 2
2 2
5 −5 1 1
⇒ T3 = 45 ( x ) = 45 x 0
4 4
1 45
= 45 (1) =
4 4
Ð Question # 9 (iii)
4
 x2 + 1 
4
 1 
(1 + x ) 1 + 2 
2 3
= (1 + x )  2 
2 3

 x   x 
( x + 1) (1 + x )
4 4
2 2

= (1 + x 2 )3 = (1 + x 2 )3
(x )2 4 x8
= x − 8 (1 + x 2 )3+4 = x − 8 (1 + x 2 )7
Now Tr +1 = x − 8   a n −r x r
n
r
FSc-I / Ex 8.2 - 9

Where n = 7 , a = 1 , x = x 2
7 7
Tr +1 = x − 8   (1)7−r ( x 2 ) = x − 8   (1) x 2 r
r

r r
7
=   x 2r −8
r
For term independent of x we must have
x 2 r − 8 = x 0 ⇒ 2r − 8 = 0 ⇒ 2r = 8 ⇒ r = 4
So
7
T4+1 =   x 2(4) − 8
4
⇒ T5 = 35 x8 − 8 = 35 x 0 = 35
Ð Question # 10
12
 1 x2 
 − 
x 2 
n + 2 12 + 2
Since n = 12 is an even so middle terms is = =7
2 2
Therefore r + 1 = 7 ⇒ r = 7 − 1 = 6
1 x2
And a = , x = − and n = 12
x 2
Now
n
Tr +1 =   a n−r x r
r
12 − 6 6
 x2 
12   1 
⇒ T6+1 =     − 
 6  x 
 2
1 x12 924 12−6
⇒ T7 = 924 6 = x
x 64 64
231 6
= x
16
231 6
Thus the middle terms of the given expansion is x .
16
Ð Question # 10 (ii)
n + 1 11 + 1
Since n = 11 is odd so the middle terms are = = 6 and
2 2
n + 3 11 + 3
= =7
2 2
So for first middle term
3 1
a = x , x = − , n = 11 and r + 1 = 6 ⇒ r = 5
2 3x
Now
11 −5 5
 n  n− r r  11  3   1 
Tr +1 =   a x ⇒ T5+1 =    x   − 
r  5  2   3 x 
Now simplify yourself.
Now for second middle term
r +1 = 7 ⇒ r = 6
11 −6 6
11  3   1 
so T6+1 =    x  −  Now simplify yourself.
 6  2   3x 
FSc-I / Ex 8.2 - 10

Ð Question # 10 (iii)
Since n = 2m + 1 is odd so there are two middle terms
n + 1 2m + 1 + 1 2m + 2
First middle term = = = = m +1
2 2 2
n + 3 2m + 1 + 3 2m + 4
Second middle terms = = = =m+2
2 2 2
1
Here a = 2 x , x = − and n = 2m + 1
2x
For first middle term r + 1 = m + 1 ⇒ r = m .
Since
n
Tr +1 =   a n−r x r
r
(2m + 1)!
m m
 2m + 1  2 m+1 − m  1  m +1  1 
⇒ Tm+1 =   ( 2x ) −  = (2x)  − 
 m   2x  (2m + 1 − m)!⋅ m !  2x 
(2m + 1)!
m m

( 2 ) ( x ) ( −1)    
m+1 m +1 m 1 1
=
(m + 1)!⋅ m! 2  x
(2m + 1)!
( 2 ) ( x ) ( −1) ( 2 ) ( x )
m+1 m +1 −m −m
=
m

(m + 1)!⋅ m!
(2m + 1)! (2m + 1)!
( 2 ) ( x ) ( −1) = ( 2 ) ( x ) ( −1)
m+1− m m+1−m
=
m 1 1 m

(m + 1)!⋅ m! (m + 1)!⋅ m!
(2m + 1)!
= 2 x ( −1)
m

(m + 1)!⋅ m!
For second middle term
r +1 = m + 2 ⇒ r = m + 2 −1 ⇒ r = m +1
n
As Tr +1 =   a n−r x r
r
m +1
 2m + 1 (2 m +1) − ( m +1)  1 
⇒ Tm+1+1 =  (2x) − 
 m +1   2x 
Now simplify yourself
Ð Question # 11 (Submitted by Waqas Ahmad - FAZMIC Sargodha – 2004-06)
1
Here a = x , x = − ,
2x
Number of term from the end = 2n + 1
1
To make it from beginning we take a = − , x = x and r + 1 = 2n + 1 ⇒ r = 2n
2x
n
As Tr +1 =   a n−r x r
r
3n− 2 n n
 3n   1  (3n)!  1  2n
⇒ T2 n+1 =    −  ( x) = −  x
2n

 2n  2 x  (3n − 2n)!⋅ (2n)!  2 x 


(3n)! 1 (3n)! n 1
= ( −1) n n x 2n = ( −1) n x 2n−n
n

(n)!⋅ (2n)! 2 ⋅x n!⋅ (2n)! 2


(−1) n (3n)! n
= x Answer
2 n n!⋅ (2n)!
Note: If there are p term in some expansion and qth term is from the end then the term
from the beginning will be = p − q + 1 .
So in above you can use term from the end = (3n + 1) − (2n + 1) + 1 = n + 1
FSc-I / Ex 8.2 - 11

Ð Question # 12
2n + 2
Since 2n is even so the middle term is = n + 1 and
2
a = 1 , x = x , n = 2n , r + 1 = n + 1 ⇒ r = n
Now Tr +1 =   a n−r x r
n
r
 2n 
⇒ Tn +1 =   (1) 2 n−n x n
n
(2n)! (2n)! n
⇒ Tn +1 = (1) n x n = x
(2n − n)!⋅ n! n!⋅ n !
2n (2n − 1)(2n − 2)(2n − 3)(2n − 4)................⋅ 5 ⋅ 4 ⋅ 3 ⋅ 2 ⋅1 n
= x
n!⋅ n!
=
[ 2n (2n − 2)(2n − 4)................⋅ 4 ⋅ 2][ (2n − 1)(2n − 3)................⋅ 5 ⋅ 3 ⋅ 1] x n
n!⋅ n!
2n [ n (n − 1)(n − 2)................⋅ 2 ⋅ 1][(2n − 1)(2n − 3)................⋅ 5 ⋅ 3 ⋅ 1] n
= x
n !⋅ n !
2n n! [(2n − 1)(2n − 3)................⋅ 5 ⋅ 3 ⋅1] n
= x
n!⋅ n!
2n [1 ⋅ 3 ⋅ 5 ⋅ ............... ⋅ (2n − 1)] n
= x
n!
1 ⋅ 3 ⋅ 5 ⋅ ............... ⋅ (2n − 1) n n
= 2 x
n!
Ð Question # 13
Consider
n n n
(1 + x) n =   +   x +   x 2
n n n  n  x n −1 +  n  x n
+   x 3 +   x 4 +   x 5 + .................. +   n
0 1   2 3 4 5  n − 1  
………. (i)
Put x = 1
n n n n n n
(1 + 1) n =   +   (1) +   (1) 2 +   (1) 3 +   (1) 4 +   (1) 5 + ............ + 
 n  (1) n−1 +  n  (1) n
  n
0 1 2 3 4 5  n − 1  
n n  n n n  n  n  n
⇒ 2 n =   +   +   +   +   +   + ............... +  + 
0 1   2  3 4  5  n − 1  n 
 n n   n n 
⇒ 2 n =   +   +   + ............... +    +   +   +   + ............... + 
n n n n

 0   2   4   n    1   3   5   n − 1 
………. (ii)
Now put x = −1 in equation (i)
n n n n n n
(1 − 1) n =   +   ( −1) +   ( −1) +   (−1) +   ( −1) +   ( −1) + ...........
2 3 4 5

0 1 2 3  4 5


 n  (−1) n−1 +  n  (−1)n
.......... +   n
 n − 1  
If we consider n is even then
n  n n n  n  n  n  n
⇒ (0)n =   −   +   −   +   −   + ................. −  + 
0 1  2  3  4  5  n − 1  n 
 n n   n n 
⇒ 0 =   +   +   + ................. +    −   +   +   + ................. + 
n n n n

 0   2   4   n    1   3   5   n − 1 
 n n    n   n   n  n 
⇒   +   +   + ................. +  =   +   +   + ................. +   
n n
 
 1   3   5   n − 1    0   2   4   n 
FSc-I / Ex 8.2 - 12
Using it in equation (ii)
2 n =   +   +   + .................... + 
n    n   n   n  n 
+   +   +   + .................... + 
n n n
 
 1   3   5   n − 1    1   3   5   n − 1  
 n n 
⇒ 2 n = 2   +   +   + .................... + 
n n

 1   3   5   n − 1  
2n   n   n   n  n 
⇒ =   +   +   + .................... +  
2  1   3   5   n − 1  
 n n 
⇒ 2 n−1 =   +   +   + .................... + 
n n

 1   3   5   n − 1 
n  n n  n 
⇒   +   +   + .................... +   = 2n −1
1  3 5  n − 1

Ð Question # 14
L.H.S =   +   +   +   + .................... +
n 1 n 1 n 1 n 1 n
 
0 2 1 3 2 4  3 n + 1 n 
 n  1  n!  1 n!  1 n!  1  n 
=   +   +   +   + .................... +  
 0  2  (n − 1)!⋅1!  3  (n − 2)!⋅ 2!  4  (n − 3)!⋅ 3!  n + 1  n 
n +1 1  n!  1  n!  1 n!  1 
= 1 +  +  +   + .................... + ⋅1
n + 1  2  (n − 1)!⋅1!  3  (n − 2)!⋅ 2!  4  (n − 3)!⋅3!  n +1 
1  1  ( n + 1) n !  1  ( n + 1) n !  1  ( n + 1) n !  n +1 
= ( n + 1) +  +  +   + .................... + ⋅ 1
n +1 2  ( n − 1)!⋅1!  3  ( n − 2)!⋅ 2!  4  ( n − 3)!⋅ 3!  n +1 
1   (n + 1)!   (n + 1)!   (n + 1)!  
= (n + 1) +  + +  + .................... + 1
n +1  (n − 1)!⋅ 2 ⋅1!   (n − 2)!⋅ 3 ⋅ 2!   (n − 3)!⋅ 4 ⋅ 3!  
1   ( n + 1)!   ( n + 1)!   ( n + 1)!  
=  ( n + 1) +   +  +  + .................... + 1
n +1  ( n + 1 − 2)!⋅ 2!   ( n + 1 − 3)!⋅ 3!   ( n + 1 − 4)!⋅ 4!  
1  n + 1   n + 1   n + 1   n + 1   n + 1 
=   1  +  2  +  3  +  4  + .................... +  n + 1 
n + 1          
1  n + 1   n + 1  n + 1   n + 1 n + 1 
=  −1 + 1 +   +  +  +  + .................... +  
n +1  1   2   3   4   n + 1 
1   n + 1 +  n + 1  +  n + 1 +  n + 1  +  n + 1 + .................... +  n + 1 
= − 1 +  0   1   2   3   4   n + 1 
n + 1             
1
=  −1 + 2n +1 
n +1
2n +1 − 1
= = R.H.S
n +1

Ð Remember
 n + 1  n + 1  n + 1
 0  = 1 ,  1  = n + 1 and  n + 1 = 1
     

Made By: Atiq ur Rehman, (atiq@mathcity.org) http://www.mathcity.org

J The End J
mathcity.org Exercise 8.3 (Solutions)
Textbook of Algebra and Trigonometry for Class XI
Merging man and maths Available online @ http://www.mathcity.org, Version: 1.0.5

Binomial Theorem when n is negative or fraction:


When n is negative or fraction and x < 1 then
n (n − 1) 2 n (n − 1)(n − 2) 3
(1 + x) n = 1 + n x + x + x + ......................
2! 3!
Where the general term of binomial expansion is
n (n − 1)(n − 2).............( n − (r − 1) ) r
Tr +1 = x
r!
Question # 1 (i)
1 1  1  1  1 
1  − 1  − 1 − 2 
(1 − x) = 1 + (− x) + 
2 2 
( − x) 2 + 
2
1 2 2  2  (− x )3 + ......................
2 2! 3!
1 1 1  1  3 
 −   −  − 
1 2  2  2 2  2  2 
=1− x + x + (− x 3 ) + ......................
2 2 3⋅ 2
1 1 1
= 1 − x − x 2 − x3 + ......................
2 8 16
Question # 1 (ii) and (iii) Do yourself as above

Question # 1 (iv)
1 1 1
  3x   2  3x  2  3x  2
1 1
(4 − 3 x) =  4 1 −   = (4) 2 1 −  = 2 1 − 
2

  4   4   4 

 1 1  1  1  1  
 1  3 x  2  2 − 1  3 x 2 2  2 − 1 2 − 2   3 x 3 
= 2 1 +  −  +   −
  +    −
  + ......................
 2  4  2!  4  3!  4  
 
 1 1 1  1  3  
 3x 2  − 2   9 x 2  2  − 2  − 2   27 x 3  
= 2 1 − +     
 + −  + ......................
 8 2  16  3⋅ 2  64  
 
 3 x 1  9 x 2  1  27 x 3  
= 2 1 − −   −   + ......................
 8 8  16  16  64  
 3 x 9 x 2 27 x3 
= 2 1 − − − + ......................
 8 128 1024 
3 x 9 x 2 27 x 3
=2− − − + ......................
4 64 512

Question # 1 (v)
−1 −1
 2x  1 x 
1
−1
(8 − 2 x ) = (8) 1 −  = 1 − 
2
Now do yourself
 8  8 4 

Question # 1 (vi)
Do yourself
FSc-I / Ex 8.3 - 2

Question # 1 (vii)
(1 − x)−1
= (1 − x)−1 (1 + x )−2
(1 + x) 2

 (−1)(−1 − 1) (−1)(−1 − 1)(−1 − 2) 


= 1 + (−1)(− x ) + ( − x) 2 + (− x)3 + ................... 
 2! 3! 
 (−2)(−2 − 1) 2 (−2)(−2 − 1)(−2 − 2) 3 
× 1 + (−2)( x ) + ( x) + ( x) + ................... 
 2! 3! 
 (−1)(−2) 2 (−1)(−2)(−3) 
= 1 + x + (x ) + (− x 3 ) + ................... 
 2 3⋅ 2 
 (−2)(−3) 2 (−2)(−3)(−4) 3 
× 1 − 2 x + ( x) + ( x) + ................... 
 2 3⋅ 2 
= (1 + x + x 2 + x 3 + ...................) × (1 − 2 x + 3 x 2 − 4 x 3 + ...................)
= 1 + ( x − 2 x) + ( x 2 − 2 x 2 + 3x 2 ) + ( x 3 − 2 x3 + 3x3 − 4 x3 )...................
= 1 − x + 2 x 2 − 2 x3 + ...................
Question # 1 (viii)
Do yourself as above
Question # 1 (ix)
1 1 −1
(4 + 2 x) 2 1
−1 2
1 2 x  2 −1  x
= (4 + 2 x) (2 − x) = (4) 1 +
2
 (2) 1 − 
2−x  4   2
1 −1 1 −1 1 −1
 1 x  2 −1  x  x 2 1 x  x 2  x
= (4) 1 +  (2) 1 −  = 2 1 + 
2
 1 −  =  1 +   1 − 
 2  2  2 2 2  2  2
1 −1
 x 2  x
= 1 +  1 − 
 2  2
 11  11  1 − 2  
 1 x  − 1  2  − 1   3

 x  x
= 1 +   + 2  2    + 2  2  2    + ...................... 
 2  2  2!  2  3! 2 
 
  x  (−1)(−1 − 1)  x  (−1)(−1 − 1)(−1 − 2)  x 
2 3

× 1 + (−1)  −  +  −  +  −  + ................... 
  2 2!  2 3!  2 
 
 1 1 1  1  3  
 x  −   −  −  
x 
2
x 3
= 1 + + 2  2    + 2  2  2    + ...................... 
 4 2  4 3⋅ 2 8 
 
 x (−1)(−2)  x 2  (−1)(−2)(−3)  x3  
× 1 + +  +  −  + ................... 
 2 2  4 3⋅ 2  8 
 x x 2 x3   x x 2 x3 
= 1 + − + + ......................  × 1 + + + + ................... 
 4 32 128   2 4 8 
 x x  x x 2 x 2   x3 x 3 x3 x 3 
2
=1+  +  + − + +  +  − + +  + ...................
 4 2   32 8 4   128 64 16 8 
3 x 11x 2 23 x 3
=1+ + + + ...................
4 32 128

Made by: Atiq ur Rehman, ( atiq@mathcity.org ), http://www.mathcity.org


FSc-I / Ex 8.3 - 3
Question # 1 (x)
1 1

(1 + x − 2 x2 ) 2 = (1 + ( x − 2x 2 ) ) 2
11  1 1  1 
 − 1  − 1  − 2 
= 1 + ( x − 2 x 2 ) + 2  2  ( x − 2 x 2 )2 + 2  2  2  ( x − 2 x 2 )3 + ................
1
2 2! 3!
1 1
− 
1 2 2 2
= 1 + (x − 2x ) +
2
( x − 4 x3 + 4 x 4 )
2 2
1  1  3 
 −  − 
+ 2  2  2  3
3⋅ 2
( x + 3( x )2 (−2 x 2 ) + 3( x )(−2 x 2 )2 − (2 x 2 )3 ) + ................

= 1 + ( x − 2 x 2 ) − ( x 2 − 4 x 3 + 4 x 4 ) + ( x3 − 6 x 4 + 12 x5 − 8 x 6 ) + ..................
1 1 1
2 8 16
1 2 1 4 4 1 6 12 8
= 1 + x − x 2 − x 2 − x 3 + x 4 + x 3 − x 4 + x 5 − x 6 + ..................
2 2 8 8 8 16 16 16 16
1 1 1 1 1 3 3 1
= 1 + x − x 2 − x 2 − x 3 + x 4 + x 3 − x 4 + x5 − x 6 + ..................
2 8 2 2 16 8 4 8
1 9 9
= 1 + x − x 2 − x 3 + ..................
2 8 16
Question # 1 (xi) Do yourself as above
Question # 2 (i)
1
 1 2
1 1 1
99 = ( 99 ) = (100 − 1)
2 2 = (100) 1 −
2

 100 
 11  
 1  1  2  2 − 1   1 2 
= 10 1 +  −  +  −  + .................... 
 2  100  2!  100  
 
 1 1 
  −  
 1 
+ 2 2
1
= 10 1 −  + ............... 
 200 2  10000  
 
 
= 10 1 − 0.005 − ( 0.0001) + ............... 
1
 8 
= 10 (1 − 0.005 − 0.0000125 + ...............)
≈ 10 ( 0.9949875) = 9.949875
≈ 9.950
Question # 2 (ii)
1 1
( 0.98) 2 = (1 − 0.02 ) 2 Now do yourself

Question # 2 (iii)
1 1
(1.03) 3 = (1 + 0.03) 3 Now do yourself
Question # 2 (iv)
1
 1 3
1 1 1
3
65 = ( 65 ) 3 = ( 64 − 1) = (64) 1 − 
3 3
Now do yourself
 64 
FSc-I / Ex 8.3 - 4

Question # 2 (v)
1
 1 4
1 1 1
4
17 = (17 ) = (16 − 1)
4 4 = (16) 1 −  4
Now do yourself
 16 
Question # 2 (vi)
1
 1 5
1 1 1
5
31 = ( 31) = ( 32 − 1) = (32) 1 − 
5 5 5
Now do yourself
 32 

Question # 2 (vii)
1

−  2 
1 1 1
1 1
( ) ( ) ( )
− − 3
= = 998 3 = 1000 − 2 3 = 1000 3 1−
 
998 ( 998 ) 3
1
3
 1000 
1

 1 
1
= (10 )
3 −3
3
 1 − 
 500 
 1 1  
−  − − 
 1    1  1  
1 2
3 3  1 
=   1 +  −  −  +  −  + .................... 
 10    3  500  2!  500  

 
 1 4 
 −  −  
 1  1  3 3  1 
=   1 +   +   + .................... 
 10    1500  2  250000  

 
 1  
=  1 + ( 0.0006667 ) + ( 0.000004 ) + .................... 
2
 10  9 
 1
=   (1 + 0.0006667 + 0.00000089 + ....................)
 10 
 1
≈   (1.00066759 ) = 0.100066759 ≈ 0.100 Answer
 10 
Question # 2 ( viii)
1

1 1 1 1 1
 9 
1 = ( 252 ) = ( 243 + 9 ) = ( 243) 1 +
− − − 5
= 5 5 5

5
252 ( 252 ) 5  243 
1

 1 
1
= (3 )
5 −5
5
1 +  Now do yourself as above
 27 
Question # 2 (ix)
1 1
7 7  7 2  1 2
= =   = 1 − 
8 8 8  8
11 
 − 1 2
1 1 2 2  1
=1+  −  +  −  + ....................
2 8 2!  8 
1 1

1 2  2   1 
=1− +   + ....................
16 2  64 
1 1 1 
= 1 − −   + ....................
16 8  64 
FSc-I / Ex 8.3 - 5
1 1
=1− − + ....................
16 512
= 1 − 0.0625 − 0.00195 + ....................
≈ 0.93555 ≈ 0.936 Answer
Question # 2 (x)
1 1
− −
(0.998) 3
= (1 − 0.002) 3
Now do yourself as above
Question # 2 (xi)
1

−  243 
1 1 1
1 1
( ) ( ) ( )
− − 6
= = 486 6 = 729 − 243 6 = 729 6 1−
 
486 ( 486 ) 6
1
6
 729 
1

 1
1
= (3 6 −6
)
6
1 −  Now do yourself
 3
Question # 2 (xii)
1 1
 16  4 4 4 1 4
1 1 1

( )
1
(1280) = (1296 − 16) = (1296 ) 1 −
4 4
 = 6 4
 1 − 
 1296   81 
Now do yourself
Question # 3 (i)
(1 + x 2 ) = 1 + x2 (1 + x )−2
( )
(1 + x )
2

 (−2)(−2 − 1) 2 (−2)(−2 − 1)(−2 − 2) 3 


( 
)
= 1 + x 2 1 + (−2)( x ) +
2!
( x) +
3!
( x) + ................... 

 (−2)(−3) 2 (−2)(−3)(−4) 3 
( )
= 1 + x 2 1 − 2 x +
 2
( x) +
3⋅ 2
( x) + ................... 

= (1 + x )(1 − 2 x + 3 x − 4 x + ...................)
2 2 3

= (1 + x )(1 + (−1)2 x + (−1) 3 x + (−1) 4 x + ...................)


2 2 2 3 3

Following in this way we can write


(1 + x2 ) = 1 + x 2 (1 + (−1)2 x + (−1)2 3x 2 + (−1)3 4 x3 + .............. + (−1)n−2 (n − 1) x n−2 +
( )
(1 + x )
2

(−1)n −1 (n) x n −1 + (−1) n (n + 1) x n + ................)


So taking only terms involving x n we get
(−1)n (n + 1) x n + (−1) n−2 (n − 1) x n
= (−1)n (n + 1) x n + (−1)n (−1) −2 (n − 1) x n
= (−1)n (n + 1) x n + (−1)n (n − 1) x n Q (−1) −2 = 1
= (n + 1 + n − 1)(−1)n x n = (2n)(−1)n x n
Thus the coefficient of term involving x n is (2n)(−1)n
Question # 3 (ii)
Hint:
After solving you will get
(1 + x2 ) = 1 + x 2 (1 + 2 x + 3x2 + 4 x3 + ............ + (n − 1) x n−2 + (n) x n−1 + (n + 1) x n + ..............)
( )
(1 − x )
2

Do yourself as above

Made by: Atiq ur Rehman ( atiq@mathcity.org ) , http://www.mathcity.org


FSc-I / Ex 8.3 - 6

Question # 3 (iii)
(1 + x )3
= (1 + x)3 (1 − x) −2
(1 − x) 2

3 (−2)(−2 − 1) (−2)(−2 − 1)(−2 − 2) 


= (1 + x ) 1 + (−2)(− x) + (− x )2 + (− x )3 + ................... 
 2! 3! 
3 (−2)(−3) 2 (−2)(−3)(−4) 
= (1 + x ) 1 + 2 x + ( x) + (− x 3 ) + ................... 
 2 3⋅2 
= (1 + 3 x + 3 x 2 + x 3 )(1 + 2 x + 3 x 2 + 4 x 3 + ...................)
Following in this way we can write
(1 + x)3
(1 − x) 2 ( )
= 1 + 3 x + 3 x 2 + x 3 (1 + 2 x + 3 x 2 + 4 x3 + ............ + (n − 2) x n−3 + (n − 1) x n−2

+(n) x n −1 + (n + 1) x n + ..............)
So taking only terms involving x n we have term
(n + 1) x n + 3(n) x n + 3(n − 1) x n + (n − 2) x n
= ( (n + 1) + 3(n) + 3(n − 1) + (n − 2) ) x n
= ( n + 1 + 3n + 3n − 3 + n − 2 ) x n
= ( 8n − 4 ) x n
Thus the coefficient of term involving x n is (8n − 4 ) .
Question # 3 (iv)
(1 + x ) = 1 + x 2 1 − x −3
2

( )( )
(1 − x )
3

2 (−3)(−3 − 1) (−3)(−3 − 1)(−3 − 2) 


= (1 + x ) 1 + (−3)(− x) + ( − x) 2 + (− x)3 + ................... 
 2! 3! 
2 (−3)(−4) (−3)(−4)(−5) 
= (1 + x ) 1 + (−3)(− x ) + ( − x) 2 + (− x)3 + ................... 
 2 3⋅ 2 
 
( 
)
= 1 + 2 x + x 2 1 + 3x +
(3)(4) 2
2
(x ) +
(4)(5) 3
2
( x ) + ................... 

 (2)(3) 
(
= 1 + 2 x + x 2 1 +

) 2
x+
(3)(4) 2 (4)(5) 3
2
x +
2
x + ................... 

Following in this way we can write
(1 + x ) = 1 + 2 x + x 2 1 + (2)(3) x + (3)(4) x2 + (4)(5) x3 + ..................
2

( ) 
(1 − x )
3
2 2 2
(n − 1)(n) n−2 (n)(n + 1) n−1 (n + 1)(n + 2) n 
+ x + x + x + ....... 
2 2 2 
n
So taking only terms involving x we have term
(n + 1)(n + 2) n (n)(n + 1) n (n − 1)(n) n
x +2 x + x
2 2 2
xn
= ( (n + 1)(n + 2) + 2(n)(n + 1) + (n − 1)(n) )
2
n
= ( n 2 + n + 2n + 2 + 2n 2 + 2n + n 2 − n )
x
2
n
xn
= ( 4n + 4n + 2 ) = 2 ( 2n + 2n + 1)
2 x 2

2 2
= ( 2n + 2n + 1) x
2 n

Thus the coefficient of term involving x n is ( 2n 2 + 2n + 1) .


FSc-I / Ex 8.3 - 7
Question # 3 (v)
Since we know that
(1 + x) −1 = 1 − x + x 2 − x 3 + ...............
Therefore
(1 − x + x ) ( ) = (1 + x )
−2
2 2
2
− x3 + ............. = (1 + x )−1
(−2)(−2 − 1) 2 (−2)(−2 − 1)(−2 − 2) 3
= 1 + (−2)( x) + ( x) + ( x ) + ...................
2! 3!
(−2)(−3) 2 (−2)(−3)(−4) 3
= 1 − 2x + ( x) + ( x) + ...................
2 3⋅ 2
= 1 − 2 x + 3 x 2 − 4 x 3 + ...................
= 1 + (−1)2 x + (−1)2 3x 2 (−1)3 4 x 3 + ...................

Following in this way we can write


= 1 + (−1)2 x + (−1)2 3x 2 (−1)3 4 x 3 + ................... + (−1)n (n + 1) x n + .........
So the term involving x n = (−1)n (n + 1) x n
And hence coefficient of term involving x n is (−1)n (n + 1)

Question # 4 (i)
1− x 1− x 1−
1 1
L.H.S = = 1 = (1 − x )
2
= (1 − x) 2
1 − x (1 − x ) 2
1
= 1 +   (− x ) + squares and higher power of x.
2
1
= 1 − x = R.H.S Proved
2
Question # 4 (ii)
1 + 2x 1 1
= (1 + 2 x ) (1 − x ) 2

Since 2
1− x
1
1
Now (1 + 2 x ) 2 = 1 +   (2 x) + squares and higher power of x.
2
≈1+ x
1
 1
Now (1 − x ) 2 = 1 +  −  (− x) + squares and higher power of x.

 2
1
≈1+ x
2
1 + 2x  1 
≈ (1 + x ) 1 + x 
1− x  2 
1
=1+ x + x ignoring term involving x 2 .
2
3
= 1 + x Proved.
2
Question # 4 (iii)

( )
1 1
(9 + 7 x) 2 − (16 + 3x ) 4
( 4 + 5x )
1 1 −1
= (9 + 7 x) 2 − (16 + 3 x) 4
4 + 5x
1
 7x 2
1 1
Now (9 + 7 x ) = 9 1 +
2

2

 9 
FSc-I / Ex 8.3 - 8
1   1  7 x  
= (32 ) 2 1 +    + squres and higher of x. 
  2  9  
 7x   7x  7x
≈ 3 1 +  = 3 + 3  = 3 +
 18   18  6
1
1
 3x  4
1
(16 + 3 x) 4 = (16) 1 + 
4

 16 
1  1  3 x  
= (2 4 ) 4 1 +    + square and higher power of x 
  4  16  
 3x   3x  3x
≈ (2) 1 +  = 2 + 2   = 2 +
 64   64  32
−1
 5 
( 4 + 5x )
−1 −1
= 4 1 + x 
 4 
1 5  
= 1 + ( −1)  x  + squares and higher power of x 
4 4  
1 5  1 5
≈ 1 − x  = − x
4 4  4 16
1 1
(9 + 7 x) 2 − (16 + 3 x) 4  7x   3x    1 5 
So ≈  3 +  −  2 +   − x 
4 + 5x  6   32    4 16 
 7x 3 x   1 5   103  1 5 
= 3 + − 2 −   − x  = 1+ x  − x 
 6 32   4 16   96  4 16 
1 103 5 1 17
= + x− x = − x Proved
4 384 16 4 384
Question # 4 (iv) Do yourself
Question # 4 (v)
(1 + x ) 2 ( 4 − 3x ) 2 = 1 + x 12 4 − 3x 32 8 + 5 x − 13
1 3

( ) ( ) ( )
(8 + 5 x )
1
3

1
Now (1 + x ) 2 = 1 +   ( x) + square and higher power of x
1

2
1
≈1+ x
2
3
 3 2
( 4 − 3x )
3 3
2 = 4 1 − x 
2

 4 
  3  3  
( )
3
= 2 2 2 1 +   − x  + square and higher power of x 
  2  4  
3 9   9 
≈ ( 2 ) 1 − x  = 8 1 − x 
 8   8 
−1
 5  3
(8 + 5 x ) = ( 8) 1 + x 
−1 −1
3 3

 8 
−1   1  5  
( )
= 23 3 1 +  −  x  + square and higher power of x 
  3  8  
 5  1 5 
≈ (2) −1 1 − x  = 1 − x
 24  2  24 
FSc-I / Ex 8.3 - 9

(1 + x ) 2 ( 4 − 3x ) 2
1 3
 1   9  1 5 
So ≈ 1 + x  8 1 − x  1 − x
(8 + 5x )
1
3  2   8  2  24 
8 1   9 5 
= 1 + x  1 − x − x
2 2   8 24 
 1   4   1 4   5 
= 4 1 + x  1 − x  = 4 1 + x − x  = 4 1 − x  Proved
 2   3   2 3   6 
Question # 4 (vi)
Do yourself as above
Question # 4 (vii)
Same as Question #4 (iii)
Question # 5 (i)
1 − x − 2 x 2 = (1 − ( x + 2 x 2 ) ) 2
1

1
1 1 −1
( )
( ) ( )
2
= 1 +   −( x + 2 x ) + 2 2
2
−( x + 2 x 2 ) + cube & higher power of x.
2 2!
1
1 −1
( )
≈ 1 −   ( x + 2 x ) + 2 2 ( x + 2 x2 )2
2

2 2
1 1 1 1 1
≈ 1 − x − (2 x 2 ) − x 2 = 1 − x − x 2 − x 2
2 2 8 2 8
1 9
= 1 − x − x2 Proved
2 8

Question # 5 (ii)
1
1 + x (1 + x ) 2 1

1
= 1 = (1 + x ) (1 − x )
2 2
1 − x (1 − x) 2
Now
1
1
1 1 −1
(1 + x) = 1 +   x + 2 2
2
( )
x 2 + cube & higher power of x.
2 2!
1
1 −1
( ) 1
≈ 1 + x + 2 2 x2 = 1 + x − x2
1
2 2 2 8

1
 1
(1 − x) = 1 +  −  (− x) +
2
( )(
− 12 − 12 − 1 )
(− x)2 + cube & higher power of x.
 2 2!
1
≈1+ x +
( )( )
− 12 − 32 2 1 3
x = 1 + x + x2
2 2 2 8
So
1+ x  1 1  1 3 
= 1 + x − x 2 1 + x + x 2 
1− x  2 8  2 8 
1 1 1 1 3 1
= 1 + x − x2 + x + x 2 + x 2 = 1 + x + x 2 Proved
2 8 2 4 8 2

Made by: Atiq ur Rehman, ( atiq@mathcity.org )


http://www.mathcity.org
FSc-I / Ex 8.3 - 10
Question # 6
Since x is nearly equal to 1 so suppose x = 1 + h ,
where h is so small that its square and higher powers be neglected
L.H.S = px p − qx q
= p(1 + h) p − q(1 + h) q
= p (1 + ph + square & higher power of x)
− q (1 + qh + square & higher power of h)
= p (1 + ph) − q (1 + qh)
= p + p 2 h − q − q 2 h …………….. (i)
Now R.H.S = ( p − q) x p + q
= ( p − q)(1 + h) p + q
= ( p − q) (1 + ( p + q)h + square & higher power of h )
= ( p − q) (1 + ( p + q)h ) = ( p − q) (1 + ph + qh )
= p + p 2 h + pqh − q − pqh − q 2 h
= p + p 2 h − q − q 2 h …………….. (ii)
From (i) and (ii)
L.H.S = R.H.S Proved
Question # 7
Since p − q is small when compare
Therefore let p − q = h ⇒ p = q + h
(2n + 1) p + (2n − 1)q (2n + 1)(q + h ) + (2n − 1)q
L.H.S = =
(2n − 1) p + (2n + 1)q (2n − 1)(q + h) + (2n + 1)q
2nq + q + 2nh + h + 2nq − q 4nq + 2nh + h
= =
2nq − q + 2nh − h + 2nq + q 4 nq + 2 nh − h
−1
4nq + 2nh + h 4nq + 2nh + h  2nh − h 
= = 1 + 4nq 
 2nh − h  4nq  
4nq 1 + 
 4nq 
4nq + 2nh + h   2nh − h  
= 1 + (−1)   + square & higher power of x 2 
4nq   4nq  
4nq + 2nh + h  2nh − h  4nq + 2nh + h  4nq − 2nh + h 
=  1−  =  
4nq  4nq  4nq  4nq 
16n 2 q 2 + 8n 2 hq + 4nhq − 8n 2 hq + 4nhq
= ignoring squares of h
16n 2 q 2
16n 2 q 2 + 8nhq 16n 2 q 2 8nhq
= 2 2
= +
16n q 16n q 16 n 2 q 2
2 2

h
=1+ …………….. (ii)
2nq
1 1
 p + q n  q + h + q n
Now R.H.S =   =  2q 
 2 q   
1 1 1
 2q + h  n  2 q h  n  h n
=  = +  = 1 + 2q 
 2q   2 q 2q   
 1  h 
= 1 +    + square & higher power of h .
 n   2q 
FSc-I / Ex 8.3 - 11
h
=1+ …………….. (ii)
2nq
Form (i) and (ii)
L.H.S = R.H.S Proved
Question # 8
Since n and N are nearly equal therefore consider N = n + h ,
where h is so small that its squares and higher power be neglected.
1 1
 n   2 n  2
L.H.S =   = 
 2(n + N )   2(n + n + h) 
1 1 1 1
− − −
 n  2  2(2n + h )  2  4n + 2h  2  2h  2
=  =  =  =4+ 
 2(2 n + h )   n   n   n 
1 1
− −
 2h   h 
1 1
− 2 − 2
= (4) 1 + 2
 = (2 ) 1 +
2

2

 4n   2n 
  1 h 
= (2) −1 1 +  −  + square & higher power of h 
  2  2n 
1 h  1 h
= 1 −  = − …………….. (i)
2  4n  2 8n
8n n+ N
Now R.H.S = −
9n − N 4n
8n n + ( n + h) 8n n+n+h
= − = −
9n − ( n + h) 4n 9n − n − h 4n
−1
8n 2n + h 8n 2n + h  h  2n + h
= − = − = 1 −  −
8n − h 4n 8n 1 − h
8n ( 4n )
 8n  4n

  h    2n h 
= 1 + (−1)  −  + square & higher power of h  −  + 
  8n    4n 4n 
 h  1 h  h 1 h
= 1 +  −  +  =1+ − −
 8n   2 4n  8n 2 4n
1 h
= − …………….. (ii)
2 8n
From (i) and (ii)
L.H.S = R.H.S Proved
Question # 9 (i)
1  1  1⋅ 3  1  1 ⋅ 3 ⋅ 5  1 
2 3

1−   +   −   + .......................
2  4  2! ⋅ 4  4  3! ⋅ 8  4 
Suppose the given series be identical with
n(n − 1) 2
(1 + x) n = 1 + nx + x + ...............................
2!
1 1 
This implies nx = −   ……………….…… (i)
2 4 
n(n − 1) 2 1 ⋅ 3  1 
2

x =   …………. (ii)
2! 2! ⋅ 4  4 
1 1
From (i) nx = − ⇒ x=− …………… (iii)
8 8n
FSc-I / Ex 8.3 - 12
Putting value of x in (ii)
n(n − 1)  1  1⋅ 3  1 
2 2

−  =  
2!  8n  2! ⋅ 4  4 
n(n − 1)  1  3 1
⇒  2 
=  
2  64n  2 ⋅ 4  16 
(n − 1) 3 3
⇒ = ⇒ (n − 1) = ⋅ 128n ⇒ n − 1 = 3 n
128n 128 128
1
⇒ n − 3n = 1 ⇒ − 2n = 1 ⇒ n = −
2
Putting value of n in equation (iii)
1 1
x=− ⇒ x=
8 −1
2 ( ) 4
So
1 1 1
− −
 1 2 5 2  4 2 4
(1 + x) n = 1 +  =  =  =
 4 4 5 5
Question # 9 (ii) Do yourself as above
Question # 9 (iii)
3 3⋅5 3⋅5⋅7
1+ + + + .......................
4 4 ⋅8 4 ⋅ 8 ⋅ 12
Suppose the given series be identical with
n(n − 1) 2
(1 + x) n = 1 + nx + x + ...............................
2!
3
This implies nx = ……………….…… (i)
4
n(n − 1) 2 3 ⋅ 5
x = …………. (ii)
2! 4 ⋅8
3 3
From (i) nx = ⇒ x= …………… (iii)
4 4n
Putting value of x in (ii)
n(n − 1)  3  3 ⋅ 5
2

  =
2!  4n  4 ⋅8
n (n − 1)  9  15
⇒  =
2  16 n 2  32
9(n − 1) 15 15
⇒ = ⇒ 9(n − 1) = ⋅ 32 n ⇒ 9n − 9 = 15 n
32 n 32 32
9 3
⇒ 9n − 15n = 9 ⇒ − 6n = 9 ⇒ n = − ⇒ n=−
6 2
Putting value of n in equation (iii)
3 1
x=− ⇒ x=−
4 −3
2 ( ) 2
3 3
− −
 1 1
( 2)
3
= ( 2) =
2 2 3
So (1 + x) = 1 − 
n
=  2 =2 2 Answer
 2 2
Question # 9 (iv) Do yourself as above
FSc-I / Ex 8.3 - 13
Question # 10
1 1⋅ 3 1 ⋅ 3 ⋅ 5
1+ + + + .......................
4 4 ⋅8 4 ⋅ 8 ⋅ 12
Suppose the given series be identical with
n(n − 1) 2
(1 + x) n = 1 + nx + x + ...............................
2!
This implies
1
nx = ……………….…… (i)
4
n(n − 1) 2 1 ⋅ 3
x = …………. (ii)
2! 4 ⋅8
1 1
From (i) nx = ⇒ x= …………… (iii)
4 4n
Putting value of x in (ii)
n(n − 1)  1  1 ⋅ 3
2

  =
2!  4n  4 ⋅8
n (n − 1)  1  3
⇒  =
2  16 n 2  32
(n − 1) 3 3
⇒ = ⇒ (n − 1) = ⋅ 32 n ⇒ n −1 = 3n
32 n 32 32
1
⇒ n − 3n = 1 ⇒ − 2n = 1 ⇒ n=−
2
Putting value of n in equation (iii)
1 1
x= ⇒ x=−
4 −1
2 ( ) 2
1 1
− −
 1 2 1 2 1
So (1 + x) n = 1 −  =   = ( 2 ) 2 = 2
 2 2
1 1⋅ 3 1 ⋅ 3 ⋅ 5
Hence 1+ + + + ....................... = 2 Proved
4 4 ⋅8 4 ⋅ 8 ⋅ 12
Question # 11
1 1 ⋅ 3  1  1⋅ 3 ⋅ 5  1 
2 3

y= +   +   + .......................
3 2!  3  3!  3 
Adding 1 on both sides
1 1⋅ 3  1  1⋅ 3 ⋅ 5  1 
2 3

1+ y =1+ +   +   + .......................
3 2!  3  3!  3 
Suppose the given series be identical with
n(n − 1) 2
(1 + x) n = 1 + nx + x + ...............................
2!
This implies
1
nx = ……………….…… (i)
3
n(n − 1) 2 1 ⋅ 3  1 
2

x =   …………. (ii)
2! 2!  3 
1 1
From (i) nx = ⇒ x= …………… (iii)
3 3n
FSc-I / Ex 8.3 - 14
Putting value of x in (ii)
n(n − 1)  1  1 ⋅ 3  1 
2 2

  =  
2!  3n  2!  3 
n(n − 1)  1  3 1
⇒  = ⋅
2  9 n2  2 9
(n − 1) 1 1
⇒ = ⇒ (n − 1) = ⋅ 18 n ⇒ n −1 = 3n
18 n 6 6
1
⇒ n − 3n = 1 ⇒ − 2n = 1 ⇒ n = −
2
Putting value of n in equation (iii)
1 2
x= ⇒ x=−
3 −1
2( ) 3
1 1
− −
 2 1 1
= ( 3)
2 2
So (1 + x) = 1 − 
n
=  2 = 3
 3 3
This implies
1+ y = 3
On squaring both sides
(1 + y ) = ( )
2 2
3
⇒ 1 + 2 y + y2 = 3 ⇒ 1 + 2 y + y2 − 3 = 0
⇒ y 2 + 2 y − 2 = 0 Proved
Question # 12
1 1⋅ 3 1 1 ⋅ 3 ⋅ 5 1
2y = 2 + ⋅ + ⋅ + .......................
2 2! 24 3! 26
Adding 1 on both sides
1 1⋅ 3 1 1⋅ 3 ⋅ 5 1
1+ 2y =1+ 2 + ⋅ + ⋅ + .......................
2 2! 24 3! 26
Comparing above series with
n(n − 1) 2
(1 + x) n = 1 + nx + x + ...............................
2!
1 1
After solving as above you will get n = − and x = − , so
2 2
1 1
− −
 1 1 1
= ( 2) 2 = 2
2 2
(1 + x) n = 1 −  =  
 2 2
This implies
1+ 2y = 2
On squaring both sides
(1 + 2 y ) ( 2)
2
=
2

⇒ 1 + 4 y + 4 y2 = 4 ⇒ 1 + 4 y + 4 y2 − 2 = 0
⇒ 4 y2 + 4 y −1 = 0 Proved

ُ
Made by: Atiq ur Rehman, ( atiq@mathcity.org ), http://www.mathcity.org

‫ﺧﺘﻢ ﺷﺪ‬
mathcity.org Exercise 9.1 (Solutions)
Textbook of Algebra and Trigonometry for Class XI
Merging man and maths Available online @ http://www.mathcity.org, Version: 1.0.0

Question # 1
π π
(i) 30o = 30 × radian = radian
180 6
π π
(ii) 45o = 45 × radian = radian
180 4
π
(x) 10o15′ = 10.25o = 10.25 × radian = 0.1789 radian
180
π
(xii) 75o 6′ 30′′ = 75.1083o = 75.1083 × radian = 1.3109 radian
180

Question # 2
o
π  180 
(i) rad. =   = 22.5o Q π rad = 180o
8  8 
o
13  13 
(xii) π rad. =  × 180  = 146.25o Q π rad = 180o
16  16 

Question # 3

º
Since total angle in watch = 2π rad.
2π π
Angel made by hands in 1 hour = = rad.
12 6
π 2π
Thus angle made by hand in 4 hours = 4 × = rad.
6 3
Question # 4 (i)
l = 1.5 cm , r = 2.5 cm
l 1.5
Since θ = ⇒ θ= = 0.6 rad.
r 2.5
Question # 4 (ii)
Do yourself as above
Question # 5 (i)
θ = π rad. r = 6 cm
Since l = rθ ⇒ l = 6π = 6(3.14159) = 18.85 cm

Question # 5 (ii)
π 3.14159
θ = 65o20′ = 65o.33 = 65.33 × = 65.33 × = 1.1403 rad. , r = 18 mm
180 180
Since l = rθ ⇒ l = 18 × 1.1403 = 20.5254 mm

Question # 6 (i)
1
l = 5 cm , θ= rad
2
1
Since l = rθ ⇒ 5=r× ⇒ r = 5 × 2 = 10 cm
2
FSc-I / 9.1 - 2
Question # 6 (ii)
π π
l = 56 cm , θ = 45o = 45 × = rad
180 4
π 4 224
Since l = rθ ⇒ 56 = r × ⇒ r = 56 × = = 71.30 cm
4 π 3.14159

Question # 7
π
l =? , r = 14 cm , rad θ=
4
π 3.14159
Since l = rθ ⇒ l = 14 × = 14 × = 10.9956 cm
4 4
Question # 8
r = ?, θ = 1rad. , l = 35 cm
Since l = rθ ⇒ 35 = r × 1 ⇒ r = 35 cm

Question # 9
r = 500 m
1000 25
Speed = 30 km/h = 30 × m/s = m/s
60 × 60 3
25 250
Distance = l = × 10 = m
3 3
250 250 250 1
Now l = rθ ⇒ = 500 × θ ⇒ θ = = = rad.
3 3 × 500 1500 6
Question # 10
π 7
r = 9 m , θ = 70o = 70 × = π , l =?
180 18 70o
l
Now l = rθ
7 7
⇒ l = 9× π = (3.14159) = 10.9956 m ≈ 11 m
18 2
Thus the horse will cover 11m distance.
Question # 11
π π
r = 20 cm , θ = 20o = 20 ×
= rad
180 9 20o
π 3.14159
Now l = rθ ⇒ l = 20 × = 20 × = 6.98
9 9
Thus pendulum will move 6.98cm. l

Question # 12
Here r = 148 × 106 km ,
θ = 9.3 × 10−3 rad
Since l = rθ l θ

⇒ l = (148 × 106 )( 9.3 × 10 −3 ) = 1376400 km Earth

= 1.3764 × 106 km
Thus diameter of sun = 1.3764 × 106 km
FSc-I / 9.1 - 3
Question # 13
Length of wire = circumference of circle = 2π r ′ = 2π (6) = 12π
θ
i.e. l = 12π cm , r = 24 cm
l 12π π
Now θ = ⇒ θ= = rad.
r 24 2

Question # 14
Area of Sector Central angle of Sector
Since =
Area of Circle Angel of Circle
Area of Sector θ θ
⇒ =
π r2 2π
θ
⇒ Area of Sector = ×π r2

1
= r 2θ
2
Question # 15 N
r = 6400 km
π 7
θ = 45o + 25o = 70o = 70 × = π rad
180 18
Now l = rθ W E
25 45
7 
⇒ l = (6400)  π  = (6400)(0.3889 × 3.14159)
 18 
= 7819.075
S
Thus distance between cities = 7819.075 km

Question # 16
π r
θ = 0.5o = 0.5 × = 0.5 × (0.01745)
180
= 0.008727 rad l 0.5ο
r = 3.844 × 10 km
5
Earth
Now l = rθ
⇒ l = ( 3.844 × 105 ) ( 0.0087266 ) = 3354.505
Thus diameter of the moon = 3354.505 km
Question # 17
π r
θ = 1o54′ = 1.9o = 1.9 × = 0.03316 rad
180
l = 2(6400) = 12800 km , r =? l θ

Now l = rθ ⇒ 12800 = r (0.03316) Moon

12800
⇒ r= = 386007.24 km
0.03316
Thus distance between earth and moon = 386007.24km

Made by: Atiq ur Rehman (mathcity@gmail.com), http://www.mathcity.org

-; END ;-
mathcity.org Exercise 9.2 (Solutions)
Textbook of Algebra and Trigonometry for Class XI
Merging man and maths Available online @ http://www.mathcity.org, Version: 1.0.0

Question # 1 & 2
Do yourself
Question # 3 (i)
Since sinθ < 0 so θ lies in IIIrd or IVth quadrant.
Also cosθ > 0 so θ lies in Ist or IVth quadrant.
⇒ θ lies in IVth quadrant
Question # 3 (ii)
Since cot θ > 0 so θ lies in Ist or IIIrd quadrant.
Also cscθ > 0 so θ lies in Ist or IInd quadrant
⇒ θ lies in Ist quadrant.
Question # 3 (iii), (iv) and ……
Do yourself as above
Question # 4
Since sin 2θ + cos 2θ = 1
⇒ cos 2θ = 1 − sin 2θ
⇒ cosθ = ± 1 − sin 2θ
As terminal ray lies in Ist quadrant so cosθ is +ive.
cosθ = 1 − sin 2θ
2
 12  12
⇒ cosθ = 1 −   Q sin θ =
 13  13
144 25 5
= 1− = ⇒ cosθ =
169 169 13
Now
sin θ 1213 12 13 12
tan θ = = = ⋅ ⇒ tan θ =
cosθ 513 13 5 5

1 1 13 13
cscθ = = = ⇒ cscθ =
sin θ 12 12 12
13
1 1 13 13
secθ = = = ⇒ secθ =
cosθ 5 13 5 5
1 1 5 5
cot θ = = = ⇒ cot θ =
tan θ 12 5 12 12

Question # 4 (ii)
Since sin 2θ + cos 2θ = 1
⇒ sin 2θ = 1 − cos 2θ
⇒ sin θ = ± 1 − cos 2θ
As terminal ray lies in IVth quadrant so sinθ is –ive .
sinθ = − 1 − cos 2θ
2
 9 
⇒ sin θ = − 1 −  
 41 
FSc-I / 9.2 - 2

81 1600 40 40
= − 1− =− =− ⇒ sin θ = −
1681 1681 41 41
Now
sin θ − 40
tan θ = = 41 = − 40 ⋅ 41 = − 40 ⇒ tan θ = −
40
cosθ 9 41 9 9 9
41
1 1 41 40
cscθ = = =− ⇒ cscθ = −
sin θ − 40 41 40 41
1 1 41 41
secθ = = = ⇒ secθ =
cosθ 9
11 9 9
1 1 9 9
cot θ = = =− ⇒ cot θ = −
tan θ − 409 40 40

Question # 4 (iii)
Do yourself as above.
Question # 4 (iv)
Since sec 2θ = 1 + tan 2θ
⇒ secθ = ± 1 + tan 2θ
As terminal ray is in IInd quadrant so secθ is –ive.
⇒ secθ = − 1 − tan 2θ
2
 1 1 10
⇒ secθ = − 1 +  −  = − 1 + = −
 3 9 9
10
⇒ secθ = −
3
1 1 3 3
Now cosθ = = =− ⇒ cosθ = −
secθ − 10 10 10
3
sin θ
Also = tan θ
cosθ
 1  3  1
⇒ sinθ = ( tanθ )( cosθ ) =  −   −  ⇒ sin θ =
 3   10  10
1 1
cscθ = = ⇒ cscθ = 10
sin θ 1
10
1 1
cot θ = = ⇒ cot θ = −3
tan θ − 13

Question # 4 (v)
Do yourself as above.
Question # 5
As cotθ is +ive and it is not in Ist quadrant so it is in IIIrd quadrant
( cotθ +ive in Ist and IIIrd quadrant)
Now csc θ = 1 + cot θ
2 2

⇒ cscθ = ± 1 + cot 2 θ
As terminal ray is in IIIrd quadrant so cscθ is –ive.
cscθ = − 1 + cot 2 θ
FSc-I / 9.2 - 3
2
 15  225 15
⇒ cscθ = − 1 +   = − 1 + Q cot θ =
8 64 8
289 17
=− ⇒ cscθ = −
64 8
1 1 8
sin θ = = ⇒ sin θ = −
cscθ −178 17
cosθ
Now = cot θ
sin θ
 15  8  15
⇒ cosθ = cot θ sin θ =   −  ⇒ cosθ = −
 8  17  17
Question # 6
π
Since 0 < θ < therefore terminal ray lies in Ist quadrant.
2
Now 1 + cot 2 θ = csc 2 θ
⇒ cot 2 θ = csc 2 θ − 1
⇒ cot θ = ± csc2 θ − 1
As terminal ray of θ is in Ist quadrant so cotθ is +ive.
cot θ = csc 2 θ − 1
2
 m2 + 1  (m 2 + 1) 2 m +1
2

⇒ cot θ =   −1 = −1 Q cscθ =
 2m  (2m)2 2m

m4 + 2m2 + 1 m 4 + 2m 2 + 1 − 4m 2 m 4 − 2m 2 + 1
= −1 = =
4m 2 4m 2 4m 2
(m 2 − 1)2 m 2 − 1 m2 − 1
= = ⇒ cot θ =
(2m)2 2m 2m
1 1 2m 2m
sin θ = = = ⇒ sin θ =
cscθ (
m2 +1
2m
( )
m 2 + 1) m2 + 1

cosθ
Now = cot θ ⇒ cosθ = ( cotθ )( sinθ )
sin θ
 m 2 − 1   2m   m2 − 1 
⇒ cosθ =   2  ⇒ cosθ =  2 
 2m   m + 1   m +1
1 1  m2 + 1 
secθ = = 2 ⇒ secθ =  2 
cosθ m2 −1  m −1
m +1
1 1  2m 
tan θ = = 2 ⇒ tan θ =  2 
cot θ m −1  m −1
2m
Question # 7
tanθ is +ive and terminal arm is not in the IIIrd quadrant, therefore terminal arm
lies in Ist quadrant.
Now sec 2 θ = 1 + tan 2 θ
⇒ secθ = ± 1 + tan 2 θ
as terminal arm is in the first quadrant so secθ is +ive.
FSc-I / 9.2 - 4

secθ = 1 + tan 2 θ
2
 1  1 8 2 2
secθ = 1 +   = 1+ 7 = 7 ⇒ secθ =
 7 7
1 1 7
Now cosθ = = ⇒ cosθ =
secθ 2 2 2 2
7
sin θ
Now = tan θ ⇒ sinθ = ( tanθ )( cosθ )
cosθ
 1  7  1
⇒ sin θ =    ⇒ sin θ =
 7  2 2  2 2
1 1
cscθ = = ⇒ cscθ = 2 2
sin θ 1
2 2

( )
2
2 2 −  2 2 
2
8
8−
csc θ − sec θ
2 2
 7  = 7
Now =
csc θ + sec θ
( )
2 2 2
8
2 2 +  2 2 
2
8+
 7 7
48
48 7 4
= 7 = ⋅ = Answer
64 7 64 3
7
Question # 8
Since csc 2 θ = 1 + cot 2 θ
⇒ cscθ = ± 1 + cot 2 θ
As terminal ray is in Ird quadrant so cscθ is +ive.
2
 5 25 29 29
cscθ = 1 + cot θ = 1 +   = 1 +
2
= =
 3 4 4 2
1 1 2
Now sin θ = = ⇒ sin θ =
cscθ 29 29
2
cosθ
Now = cot θ ⇒ cosθ = ( cot θ )( sin θ )
sin θ
 5  2  5
⇒ cosθ =     ⇒ cosθ =
 2   29  29

3  2  + 4 5
 
 6
 + 20
3sin θ + 4 cosθ
= 
29   29  29 29
Now =
cosθ − sin θ 5 − 2 5 − 2
29 29 29 29
6 + 20
26
29 29 = 26 ⋅ 2 = 26
= = Answer
5−2 3 29 3 3
29
29

Made by: Atiq ur Rehman (mathcity@gmail.com) http://www.mathcity.org

THE END
mathcity.org Exercise 9.3 (Solutions)
Textbook of Algebra and Trigonometry for Class XI
Merging man and maths Available online @ http://www.mathcity.org, Version: 1.0.0

Question # 1 (i)
L.H.S = sin 60o cos30o − cos 60o sin 30o
3 3 1 1 3 1 3 −1 2 1
= ⋅ − ⋅ = − = = = ………… (i)
2 2 2 2 4 4 4 4 2
1
R.H.S = sin 30 = ………… (ii)
2
From (i) and (ii)
L.H.S = R.H.S
Question # 1 (ii)
π π π
L.S.H = sin 2 + sin 2 + tan 2
6 3 4
2
1  3
2
1 3
=  +  + (1) = + + 1
2

2  2  4 4
1+ 3 + 4 8
= = = 2 = R.H.S *
4 4
Question # 1 (iii)
1 1 1
L.H.S = 2sin 45o + csc 45o = 2sin 45o +
2 2 sin 45o
2 ( 2)
2

 1  1 1 2 2
= 2 + = + = +
 2 2 1  ( )
2
2 2 2 2
 
 2
1 2 +1 3
= 2+ = = = R.H.S *
2 2 2
Question # 1 (v)
π π π π
L.H.S = sin 2 : sin 2 : sin 2 : sin 2
6 4 3 2
2
1  1   3
2 2
1 1 3
=   :  : (1) = : : :1
2
 :
2  2   2  4 2 4
= 1: 2 : 3: 4 = R.H.S * ×ing by 4
Question # 2 (i)
π π 3−
1
tan − tan
3 6 = 3
π π 1
1 + tan tan 1+ 3 ⋅
3 6 3
3 −1 2
= 3 = 3 = 2 = 1 *
1+1 2 2 3 3
Question # 2 (ii)
π
( 3)
2
1 − tan 2 1−
3 = 1 − 3 −2 1
= = =− *
π
( 3) 1+ 3
2
1 + tan 2 1+ 4 2
3
FSc-I / 9.3 - 2

Question # 3 (i)
When θ = 30o
3
L.H.S = sin 2θ = sin 2(30) = sin 60 = ………. (i)
2
 1  3  3
R.H.S = 2sinθ cosθ = 2sin 30o cos30o = 2     = ……….. (ii)
 2  2  2
From (i) and (ii)
L.H.S = R.H.S
When θ = 45o
L.H.S = sin 2θ = sin 2(45) = sin 90 = 1 ………. (i)
 1  1  2
R.H.S = 2sinθ cosθ = 2sin 45o cos 45o = 2    = 2 = 1 ……….. (ii)
 2  2 
From (i) and (ii)
L.H.S = R.H.S
Question # 3 (ii)
When θ = 30o
1
L.H.S = cos 2θ = cos 2(30) = cos 60 = ………. (i)
2
R.H.S = cos 2 θ − sin 2 θ = cos 2 30o − sin 2 30o
2
 3   1 2 3 1 1
=  −   = − = ……….. (ii)
 2  2 4 4 2
From (i) and (ii)
L.H.S = R.H.S
When θ = 45o
L.H.S = cos 2θ = cos 2(45) = cos90 = 0 ………. (i)
R.H.S = cos 2 θ − sin 2 θ = cos 2 45o − sin 2 45o
2 2
 1   1  1 1
=  −  = 2 − 2 = 0 ……….. (ii)
 2  2
From (i) and (ii)
L.H.S = R.H.S
Quesiton # 3 (iii)
Do yourself as above
Question # 3 (iv)
Do yourself as above
Question # 3 (v)
When θ = 30o
L.H.S = tan 2θ = tan 2(30) = tan 60 = 3 ………. (i)
 1 
o 2 
2 tan θ 2 tan 30  3
R.H.S = = =
1 − tan 2 θ 1 − tan 2 30o  1 
2

1−  
 3
( )
2
2 2
3 = 3 = 2 ⋅3 = 2 ⋅ 3
= = 3 ……….. (ii)
1− 1 2 3 2 3 2
3 3
From (i) and (ii)
FSc-I / 9.3 - 3
L.H.S = R.H.S
When θ = 45o
L.H.S = tan 2θ = tan 2(45) = tan 90 = ∞ ………. (i)
2 tan θ 2 tan 45o 2 (1)
R.H.S = = =
1 − tan 2 θ 1 − tan 2 45o 1 − (1) 2
2 2
= = = ∞ ……….. (ii)
1 −1 0
From (i) and (ii)
L.H.S = R.H.S
Question # 5
(i) Since −π = −2π + π
So sin ( −π ) = sin ( −2π + π ) = sin π = 0
cos ( −π ) = cos ( −2π + π ) = cos π = −1
tan ( −π ) = tan ( −2π + π ) = tan π = 0
csc ( −π ) = csc ( −2π + π ) = csc π = ∞ (undefined)
sec ( −π ) = sec ( −2π + π ) = secπ = −1
cot ( −π ) = cot ( −2π + π ) = cot π = ∞ (undefined)

(ii) Since −3π = −4π + π = −2(2π ) + π


So sin ( −3π ) = sin ( −2(2π ) + π ) = sin π = 0
And now find other values yourself.
5 1 π
(iii) Since π = 2 π = 2π +
2 2 2
5   π π 5   π π
So sin  π  = sin  2π +  = sin = 1 , cos  π  = cos  2π +  = cos = 0
2   2 2 2   2 2
5   π π 5   π π
tan  π  = tan  2π +  = tan = ∞ , csc  π  = csc  2π +  = csc = 1
2   2 2 2   2 2
5   π π 5   π π
sec  π  = sec  2π +  = sec = ∞ , cot  π  = cot  2π +  = cot = 0
2   2 2 2   2 2
9 1 3π
(iv) Since − π = − 4 π = − 6π +
2 2 2
 9   3π  3π
So sin  − π  = sin  − 6π +  = sin = −1
 2   2  2
And now find other values yourself.
(v) Since −15π = −16π + π
So sin ( −15π ) = sin ( −16π + π ) = sin π = 0
And now find other values yourself.
π 17 1 π
(vi) Since 1530o = 1530 × = π = 8 π = 8π +
180 2 2 2
 π π
So ( )
sin 1530o = sin  8π +  = sin = 1
 2 2
And now find other values yourself.
π 27 1 π
(vii) Since −2430o = −2430 × = − π = −13 π = −14π +
180 2 2 2
Now do yourself
FSc-I / 9.3 - 4
235 3 3π
(viii) Since π = 116 π = 116π +
2 2 2
Now do yourself
407 3 3π
(ix) Since π = 202 π = 202π +
2 2 2
Now do yourself
Question # 6
(i) Since 390o = 360 + 30

( )
So sin 390o = sin ( 360 + 30 ) = sin 30o =
1
2
cos ( 390o ) = cos ( 360 + 30 ) = cos30o =
3
2
tan ( 390o ) = tan ( 360 + 30 ) = tan 30o =
1
3
csc ( 390o ) = csc ( 360 + 30 ) = csc30o = 2

sec ( 390o ) = sec ( 360 + 30 ) = sec30o =


2
3
cot ( 390o ) = cot ( 360 + 30 ) = cot 30o = 3
(ii) Since − 330o = − 360 + 30

( )
So sin −330o = sin ( − 360 + 30 ) = sin 30o =
1
2
And now find other values yourself.
(iii) Since 765o = 720 + 45 = 2(360) + 45

So sin ( 760o ) = sin ( 2(360) + 45 ) = sin 45o =


1
2
And now find other values yourself.
(iv) Since − 675o = −720 + 45 = −2(360) + 45 Now do yourself
17 2 π
(v) Since − π = −5 π = −6π + Now do yourself
3 3 3
13 1 π
(vi) Since π = 4 π = 4π + Now do yourself.
3 3 3
25 1 π
(vii) Since π = 4 π = 4π +
6 6 6
 25   π π 1
So sin  π  = sin  4π +  = sin = Now do yourself
 6   6 6 2
71 5 π
(viii) Since − π = −11 π = −12π + Now do yourself.
6 6 6
π 23π 3π π
(ix) Since −1035o = −1035 ⋅ =− = −5 = − 6π +
180 4 4 4
 π π
( 
)
So sin −1035o = sin  − 6π +  = sin =
4 4
1
2
Now do yourself

Made by: Atiq ur Rehman (mathcity@gmail.com) , http://www.mathcity.org


Exercise 9.4 (Solutions)
MathCity.org Textbook of Algebra and
an d Trigonometry for Class XI
Merging man and maths Available online @ http://www.mathcity.org, Version: 1.1.0
FSc-I- Ex 9.4 - 2 www.MathCity.org
www.MathCity.org FSc-I- Ex 9.4 - 3
FSc-I- Ex 9.4 - 4 www.MathCity.org
www.MathCity.org FSc-I- Ex 9.4 - 5

See Alternative
Solution at the
end.
FSc-I- Ex 9.4 - 6 www.MathCity.org
www.MathCity.org FSc-I- Ex 9.4 - 7
FSc-I- Ex 9.4 - 8 www.MathCity.org

Question # 18 [by Hafiz Syed Rizwan (FSc, Session: 2007-2009) Punjab College of Science, Lahore.]

tan θ + secθ − 1
L.H.S =
tan θ − secθ + 1

× ing and ÷ ing by tan θ + secθ

tan θ + secθ − 1 tan θ + secθ


= ×
tan θ − secθ + 1 tan θ + secθ
=
( tan θ + secθ − 1)( tan θ + secθ )
tan θ − tan θ secθ + tan θ + tan θ secθ − sec 2 θ + secθ
2

=
( tan θ + secθ − 1)( tan θ + secθ ) ∵ 1 + tan 2 θ = sec 2 θ
tan 2 θ − sec2 θ + tan θ + secθ ∴ tan 2 θ − sec2 θ = −1

=
( tan θ + secθ − 1)( tan θ + secθ )
−1 + tan θ + secθ
= tan θ + secθ = R.H.S

******************************************
Available online at http://www.MathCity.org

If you have any question, ask it at


http://forum.mathcity.org
mathcity.org Exercise 10.1 (Solutions)
Textbook of Algebra and Trigonometry for Class XI
Merging man and maths Available online @ http://www.mathcity.org, Version: 1.0.0

Question # 1
3
(i) sin(−780) = − sin 780 = − sin ( 4(90) + 60 ) = − sin(60) = − *
2
(ii) cot(−855o ) = − cot855o = − cot ( 9(90) + 45)
= − ( − tan 45o ) = tan 45o = 1 * Q 855 is in the IInd quad.
(iii) csc(2040o ) = csc ( 22(90) + 60 ) = − csc(60) Q 2040o is in the Ist quad.
1 1 2
=− =− =− *
sin ( 60 ) 3 3
2
(iv) sec(−960) = sec(960) = sec (10(90) + 60 ) = − sec 60o Q 960o is in the IIIrd quad.
1 1
=− = − = −2 *
cos 60o 1
2
1
(v) tan(1110) = tan (12(90) + 30 ) = tan(30) = * Q 1110o is in the Ist quad
3
(vi) sin(−300) = − sin(300) = − sin ( 3(90) + 30 )
3
= −(− cos30o ) = cos30o = * Q 300o is in the IIIrd quad.
2
Question # 2
(i) sin196o = sin(180 + 16) = sin180 ocos16o + cos180o sin16o
= (0)cos16o + (−1)sin16o = − sin16o *

(ii) cos147o = cos(180 − 33) = cos180 ocos33o + sin180o sin 33o


= (−1)cos33o + (0)sin 33o = − cos33o *

(iii) sin 319o = sin(360 − 41) = sin 360o cos 41o − cos360o sin 41o Do yourself
(iv) cos 254o = cos(270 − 16) Do yourself
o
sin 294 sin(270 + 24)
(v) tan 294o = o
=
cos 294 cos(270 + 24)
sin 270o cos 24o + cos 270o sin 24o (−1)cos 24o + (0)sin 24o
= =
cos 270o cos 24o − sin 270o sin 24o (0)cos 24o − (−1)sin 24o
− cos 24o + 0 − cos 24o
= o
= o
= − cot 24o *
0 + sin 24 sin 24
Alternative
tan 270o + tan 24o
tan 294o = tan(270 + 24) =
1 − tan 270o tan 24o
o tan 24o   tan 24o 
tan 270 1 + o  1 + ∞ 
=  tan 270  =  
 1  1 
tan 270o  o
− tan 24o   − tan 24o 
 tan 270  ∞ 
=
(1 + 0 ) = − 1 = − cot 24o *
(
0 − tan 24o ) tan 24o
FSc-I / 10.1 - 2
(vi) cos 728o = cos(720 + 8) Do yourself

(vii) sin(−625o ) = − sin 625o = − sin(630 − 5) = − ( sin 630o cos5o − cos630o sin 5o )
= − ( (−1)cos5o − (0)sin 5o ) = − ( − cos5o − 0 ) = cos5o *
(viii) cos(−435o ) = cos 435o = cos(450 − 15) Do yourself

Question # 3
(i) L.H.S = sin(180 + α )sin(90 − α )
= ( sin180o cosα + cos180o sin α ) )( sin 90o cosα − cos90o sin α )
= ( (0)cosα + (−1)sin α ) )( (1)cosα − (0)sin α )
= ( 0 − sin α ) )( cosα − 0 ) = − sin α cosα = R.H.S *
(ii) First we calculate
3
sin 780o = sin(720 + 60) = sin(2 × 360 + 60) = sin 60o =
2
sin 480 = sin(450 + 30) = sin 450 cos30 + cos 450 sin 30o
o o o o

3
= (1)cos30 + (0)sin 30 = cos30 + 0 =
2
1 1
cos120 = − and sin 30 =
2 2
So L.H.S = sin 780 sin 480 + cos120 sin 30o
o o o

 3  3   1  1  3 1 1
=   +  −   = − = = R.H.S *
 2  2   2  2  4 4 2
(iii) First we calculate
cos306o = cos(270 + 36) = cos 270o cos36o − sin 270o sin 36o
= (0)cos36o − (−1)sin 36o = 0 + sin 36o = sin 36o
cos 234o = cos(270 − 36) = cos 270cos36 + sin 270cos36
= (0)cos36o + (−1)sin 36o = 0 − sin 36o = − sin 36o
cos162o = cos(180 − 18) = cos180o cos18o + sin180o sin18o
= (−1)cos18 + (0)sin18 = − cos18 + 0 = − cos18
So L.H.S = sin 306o + cos 234o + cos162o + cos18o
= sin 36o − sin 36o − cos18o + cos18o = 0 = R.H.S *
(iv) First we calculate (Alternative Method)
3
cos330o = cos(360 − 30) = cos(−30o ) = cos(30o ) =
2
3
sin 600 = sin(6 × 90 + 60) = − sin 60 = − Q 600o is in the IIIrd quad
2
1
cos120o = cos(90 + 30) = − sin 30 = − Q 120o is in the IInd quad
2
1
sin150o = sin(90 + 60) = cos60o = Q 150o is in the IInd quad
2
So L.H.S = cos330 sin 600 + cos120o sin150o
o o

 3  3   1  1  3 1 4
=  −  +  −   = − − = − = −1 = R.H.S *
 2  2   2  2  4 4 4
FSc-I / 10.1 - 3
Question # 4
First we calculate
sin(π + θ ) = sin π cosθ + cos π sin θ = (0)cosθ + (−1)sin θ
= 0 − sinθ = − sinθ
 3π   π  3π
tan  + θ  = tan  3 ⋅ + θ  = − cotθ Q + θ is in the IVth quad
 2   2  2
 3π   π  3π
cot  − θ  = cot  3 ⋅ − θ  = tanθ Q − θ is in the IIIrd quad
 2   2  2
cos(π − θ ) = cos π cosθ + sin π sin θ = (−1)cosθ + (0)sin θ
= − cosθ + 0 = − cosθ
csc(2π − θ ) = csc(−θ ) = − cscθ
Now
 3π 
sin 2 (π + θ ) tan  +θ 
L.H.S =  2 
 3π 
cot  − θ  cos 2 (π − θ ) csc(2π − θ )
 2 
(− sin θ ) 2 ( − cot θ ) sin 2 θ ( − cot θ )
= =
( θ ) − θ − θ tan 2 θ cos 2 θ (− cscθ )
2 2
tan ( cos ) ( csc )
cosθ
sin 2 θ
sin θ sin θ cosθ
= = = cosθ = R.H.S *
sin 2 θ 1 sin θ
cos 2 θ
cos θ
2
sin θ
Question 4 (ii)
First we calculate
cos(90 + θ ) = − sin θ Q 90 + θ is in the IInd quad.
sec(−θ ) = secθ
tan(180 − θ ) = tan ( 2(90) − θ ) = − tanθ Q 180 − θ is in the IInd quad.
sec(360 − θ ) = sec(−θ ) = secθ
sin(180 + θ ) = sin ( 2(90) + θ ) = − sinθ Q 180 + θ is in the IIIrd quad.
cot(90 − θ ) = tan θ Q 90 − θ is in the Ist quad.
Now
cos(90 + θ ) sec(−θ ) tan(180 − θ )
L.H.S =
sec(360 − θ ) sin(180 + θ ) cot(90 − θ )
(− sin θ )secθ (− tan θ )
= = 1 = R.H.S *
secθ (− sin θ ) (− tan θ )

Question # 5 (i)
Since α , β and γ are angels of triangle therefore
α + β + γ = 180 ⇒ α + β = 180 − γ
Now L.H.S = sin(α + β ) = sin(180 − γ )
= sin180 cos γ − cos180 sin γ
= (0)cos γ − (−1)sin γ = 0 + sinγ = sinγ = R.H.S *

Question # 5 (ii)
Since α , β and γ are angels of triangle therefore
α + β + γ = 180
α + β 180 − γ
⇒ α + β = 180 − γ ⇒ =
2 2
FSc-I / 10.1 - 4
α + β   180 − γ   180 γ 
Now L.H.S = cos   = cos   = cos  − 
 2   2   2 2
 γ γ γ
= cos  90 −  = cos90 cos + sin 90 sin
 2 2 2
γ γ γ γ
= (0) cos + (1) sin = 0 + sin = sin = R.H.S *
2 2 2 2
Question # 5 (iii)
Since α , β and γ are angels of triangle therefore
α + β + γ = 180 ⇒ α + β = 180 − γ
Now L.H.S = cos(α + β ) = cos(180 − γ )
= cos180 cos γ + sin180 sin γ
= (−1)cos γ + (0)sin γ = − cos γ + 0 = − cosγ = R.H.S *

Question # 5 (iv)
Since α , β and γ are angels of triangle therefore
α + β + γ = 180 ⇒ α + β = 180 − γ
Now L.H.S = tan(α + β ) + tan γ = tan(180 − γ ) + tan γ
tan180 − tan γ
= + tan γ
1 + tan180 tan γ
(0) − tan γ − tan γ
= + tan γ = + tan γ
1 + (0) tan γ 1+ 0
= − tan γ + tan γ = 0 = R.H.S *

Made by: Atiq ur Rehman (mathcity@gmail.com) http://www.mathcity.org

Remember:
● sin (α + β ) = sin α cos β + cosα sin β ● sin (α − β ) = sin α cos β − cosα sin β
● cos (α + β ) = cosα cos β − sin α sin β ● cos (α − β ) = cosα cos β + sin α sin β
tan α + tan β tan α − tan β
● tan (α + β ) = ● tan (α − β ) =
1 − tan α tan β 1 + tan α tan β
 π 
Three Steps to solve sin  n ⋅ ± θ 
 2 
Step I: First check that n is even or odd
Step II: If n is even then the answer will be in sin and if the n is odd then sin will be
converted to cos and vice virsa (i.e. cos will be converted to sin).
π
Step III: Now check in which quadrant n ⋅ ± θ is lying if it is in Ist or IInd quadrant
2
the answer will be positive as sin is positive in these quadrant and if it is in the IIIrd or
IVth quadrant the answer will be negative.
e.g. sin 667 o = sin ( 7(90) + 37 )
Since n = 7 is odd so answer will be in cos and 667 is in IVth quadrant and sin is
–ive in IVth quadrant therefore answer will be in negative. i.e sin 667o = − cos37
Similar technique is used for other trigonometric rations. i.e tan € cot and sec € csc .

The End
mathcity.org Exercise 10.2 (Solutions)
Textbook of Algebra and Trigonometry for Class XI
Merging man and maths Available online @ http://www.mathcity.org, Version: 1.0.2

Question # 1
(i) L.H.S = sin(180 + θ ) = sin180 cosθ + cos180 sin θ
= sin ( 0 ) cosθ + ( −1)sinθ = 0 − sinθ = − sinθ = R.H.S ¡
(ii) Do youself
tan 270o − tan θ
(iii) L.H.S = tan ( 270 − θ ) =
o

1 + tan 270o tan θ


 tan θ   tan θ 
tan 270o 1 − o  1 − 
 tan 270   ∞ 
= =
o 1  1 
tan 270  o
+ tan θ   + tan θ 
 tan 270  ∞ 
=
(1 − 0 ) = 1 = cot θ = R.H.S ¡
( 0 + tanθ ) tanθ
Remaining do yourself.
Question # 2
(i) Since 15 = 45 – 30
So sin15o = sin(45 − 30) = sin 45o cos30o − cos 45o sin30o
 1   3   1  1  3 1 3 −1
=  2  −   2  = − = ¡
 2    2    2 2 2 2 2 2
(ii) Since 15 = 45 – 30
So cos15o = cos(45 − 30) = cos 45o cos30o + sin 45o sin 30o
 1   3   1  1  3 1 3 +1
=    +     = + = ¡
 2  2   2  2  2 2 2 2 2 2
(iii) Since 15 = 45 – 30
o tan 45o − tan 30o
So tan15 = tan(45 − 30) =
1 + tan 45o tan 30o
1 1 3 −1
1− 1−
= 3 = 3 = 3 = 3 −1 ¡
 1  1+
1 3 +1 3 +1
1 + (1)  
 3 3 3
1 1
(iv) sec15o = = Now do yourself
cos15o cos(45 − 30)
(v), (vi) and (vii)
Hint: Use 105 = 60 + 45 in these questions
Question # 3
(i) L.H.S = sin(45 + α )
 1 1 
= sin 45o cosα + cos 45o sin α =  cosα + sin α ) 
 2 2 
1 1
= ( cosα + sin α ) = ( sin α + cosα ) = R.H.S ¡
2 2
FSc-I / 10.2 - 2

(ii) Do youself as above

Question # 4 (i)
L.H.S = tan(45 + A) tan(45 − A)
 tan 45o + tan A   tan 45o − tan A 
= o  o 
 1 − tan 45 tan A   1 + tan 45 tan A 
 1 + tan A   1 − tan A   1 + tan A   1 − tan A 
=    =   = 1 = R.H.S ¡
 1 − (1) tan A   1 + (1) tan A   1 − tan A  1 + tan A 

Question 4 (ii)
π   3π 
L.H.S = tan  − θ  + tan  +θ 
4   4 
 π   3π 
 tan 4 − tan θ   tan 4 + tan θ 
= + 
π 3π
 1 + tan tan θ   1 − tan tan θ 
 4   4 
 1 − tan θ   −1 + tan θ 
= + 
 1 + (1) tan θ   1 − (−1) tan θ 
 1 − tan θ   −1 + tan θ 
= + 
 1 + tan θ   1 + tan θ 
1 − tan θ − 1 + tan θ 0
= = = 0 = R.H.S ¡
1 + tan θ 1 + tanθ

Question # 4 (iii)
 π  π
L.H.S = sin θ +  + cos θ + 
 6  3
 π π  π π
=  sin θ cos + cosθ sin  +  cosθ cos − sin θ sin 
 6 6  3 3
 3 1  1 3
=  sin θ + cosθ  +  cosθ − sin θ 
 2 2   2 2 
3 1 1 3
= sin θ + cosθ + cosθ − sin θ = cosθ = R.H.S ¡
2 2 2 2
Question # 4 (iv)
θ
sin θ − cosθ tan
L.H.S = 2
θ
cosθ + sin θ tan
2
θ θ θ
sin sin θ cos− cosθ sin
sin θ − cosθ 2 2 2
θ θ
cos cos
= 2 = 2
θ θ θ
sin cosθ cos + sin θ sin
cosθ + sin θ 2 2 2
θ θ
cos cos
2 2
FSc-I / 10.2 - 3

θ θ  θ
sin θ cos
− cosθ sin sin θ − 
= 2 2 =  2
θ θ  θ
cosθ cos + sin θ sin cos θ − 
2 2  2

=
sin θ ( )
2 = tan θ = R.H.S ¡
cos θ ( )
2
2

Question # 4 (v)
1 − tan θ tan ϕ
L.H.S =
1 + tan θ tan ϕ
sin θ sin ϕ cosθ cos ϕ − sin θ sin ϕ
1−
cosθ cos ϕ cosθ cos ϕ
= =
sin θ sin ϕ cosθ cos ϕ + sin θ sin ϕ
1+
cosθ cosϕ cosθ cos ϕ
cosθ cos ϕ − sin θ sin ϕ cos (θ + ϕ )
= = = R.H.S ¡
cosθ cos ϕ + sin θ sin ϕ cos (θ − ϕ )

Question # 5
L.H.S = cos(α + β ) cos(α − β )
= ( cosα cos β − sin α sin β )( cosα cos β + sin α sin β )
(
= ( cosα cos β ) − ( sin α sin β )
2 2
) = cos α cos β − sin α sin β
2 2 2 2

= cos 2 α (1 − sin 2 β ) − (1 − cos 2 α ) sin 2 β


= cos 2 α − cos 2 α sin 2 β − sin 2 β + cos 2 α sin 2 β
= cos 2 α − sin 2 β …………….. (i)
= (1 − sin 2 α ) − (1 − cos 2 β )
= 1 − sin 2 α − 1 + cos 2 β
= cos 2 β − sin 2 α …………….. (ii) ¡

Question # 6
Hint: Just open the formulas

Question # 7 (i)
1 1
L.H.S = cot(α + β ) = =
tan(α + β ) tan α + tan β
1 − tan α tan β
 1 
tan α tan β  − 1
=
1 − tan α tan β
=  tan α tan β 
tan α + tan β  1 1 
tan α tan β  + 
 tan β tan α 
cot α cot β − 1 cot α cot β − 1
= = = R.H.S ¡
cot β + cot α cot α + cot β

Question # 7 (ii)
Do yourself as above
FSc-I / 10.2 - 4

Question # 7 (iii)
sin α sin β sin α cos β + cosα sin β
+
tan α + tan β cosα cos β cosα cos β
L.H.S = = =
tan α − tan β sin α sin β sin α cos β − cosα sin β

cosα cos β cosα cos β
sin α cos β + cosα sin β sin(α + β )
= = = R.H.S
sin α cos β − cosα sin β sin(α − β )

Question # 8
4 π
Since sin α = ; 0 <α <
5 2
40 π
cosα = ; 0< β <
41 2
Now
cos 2 α = 1 − sin 2 α
⇒ cosα = ± 1 − sin 2 α
Since terminal ray of α is in the first quadrant so value of cos is +ive
cosα = 1 − sin 2 α
2
4 16 9 3
⇒ cosα = 1 −   = 1− = ⇒ cosα =
5 25 25 5
Also
sin 2 β = 1 − cos2 β ⇒ sin β = ± 1 − cos 2 β
Since terminal ray of β is in the first quadrant so value of sin is +ive
sin β = 1 − cos 2 β
2
 40  1600 81 9
⇒ sin β = 1 −   = 1 − = ⇒ sin β =
 41  1681 1681 41
Now
sin(α − β ) = sin α cos β − cosα sin β
 4  40   3  9  160 27 133
=    −    = − =
 5  41   5  41  205 205 205
133
i.e. sin(α − β ) = Proved
205

Question # 9
4 π
Since sin α = ; <α <π
5 2
12 π
sin β = ; < β <π
13 2
Since cos 2 α = 1 − sin 2 α ⇒ cosα = ± 1 − sin 2 α
As terminal ray of α lies in the IInd quadrant so value of cos is –ive
cosα = − 1 − sin 2 α
2
4 16 9 3
⇒ cosα = − 1 −   = − 1 − =− ⇒ cosα = −
5 25 25 5
Now
cos 2 β = 1 − sin 2 β
FSc-I / 10.2 - 5

⇒ cos β = ± 1 − sin 2 β
As terminal ray of β lies in the IInd quadrant so value of cos is –ive
cos β = − 1 − sin 2 β
2
 12  144 25 5
⇒ cos β = − 1 −   = − 1 − =− ⇒ cos β = −
 13  169 169 13
(i) sin(α + β ) = sin α cos β + cosα sin β
 4  5   3  12  20 36 56
=   −  +  −   = − − =−
 5  13   5  13  65 65 65
(ii) cos(α + β ) = cosα cos β − sin α sin β
 3  5   4  12  15 48 33
=  −  −  −    = − =−
 5  13   5  13  65 65 65
sin(α + β ) − 65 56
56
(iii) tan(α + β ) = = =
cos(α + β ) − 33 33
65
(iv), (v) & (vi) Do yourself as above
Since sin(α + β ) is –ive so terminal are of α + β is in IIIrd or IVth quadrant
and cos(α + β ) is –ive so terminal are of α + β is in IInd or IIIrd quadrant
therefore terminal ray of α + β lies in the IIIrd quadrant.
Similarly after solving (iv), (v) & (vi) find quadrant for α − β yourself.

Question # 10 (i)
3
Since tan α =
4
tanα is +ive and terminal arm of α in not in the Ist quad. Therefore it lies in
IIIrd quad.
Now
sec 2 α = 1 + tan 2 α
⇒ secα = ± 1 + tan 2 α
Since terminal arm of α is in the IIIrd quad. therefore value of sec is –ive
secα = − 1 + tan 2 α
2
3 9 25 5
⇒ secα = − 1 +   = − 1 + =− ⇒ sec x = −
4 16 16 4
1 1 4
Now cosα = = ⇒ cosα = −
secα − 5 5
4
sin α
Now = tan α ⇒ sin α = tan α cosα
cosα
 3  4  3
⇒ sin α =   −  ⇒ sin α = −
 4  5  5
5
Since cos β =
13
cos β is +ive and terminal arm of β is not in the Ist quad. Therefore it lies in
IVth quad.
Now sin 2 β = 1 − cos2 β
⇒ sin β = ± 1 − cos 2 β
FSc-I / 10.2 - 6

Since terminal ray of β is in the fourth quadrant so value of sin is –ive


sin β = − 1 − cos 2 β
2
5 25 144 12
⇒ sin β = − 1 −   = − 1 − =− ⇒ sin β = −
 13  169 169 13

Now sin(α + β ) = sin α cos β + cosα sin β


 3   5   4   12  3 48 33
=−   +−  −  = − + ⇒ sin(α + β ) =
 5   13   5   13  13 65 65

And cos(α + β ) = cosα cos β − sin α sin β


 4  5   3  12  4 36 56
=  −   −  −  −  = − − ⇒ cos(α + β ) = −
 5  13   5  13  13 65 65

Question # 10 (ii)
Do yourself as above

Question # 11
R.H.S = tan 37o = tan(45 − 8) Q 37 = 45 − 8
o o o
tan 45 − tan 8 1 − tan 8
= o o
=
1 + tan 45 tan 8 1 + (1) tan 8o
sin 8o cos8o − sin 8o
1−
cos8 o
cos8 o cos8o − sin 8o
= = = = L.H.S ¡
sin 8o cos8o + sin 8o cos8o + sin8o
1+
cos8o cos8o
Question # 12
Since α , β and γ are angles of triangle therefore
α + β + γ = 180 ⇒ α + β = 180 − γ
α + β 180 − γ α β γ
⇒ = ⇒ + = 90 −
2 2 2 2 2
α β   γ
Now tan  +  = tan  90 − 
2 2  2
α β
tan + tan
⇒ 2 2 = cot γ  γ
Q tan  90 −  = cot
γ
α β 2  2 2
1 − tan tan
2 2
α β 1 1  β α
tan tan  +  +
2 2  tan β2 tan α2  γ
cot
2
cot
2 = cot γ
⇒ = cot ⇒
α β  2 α β 2
tan tan 
1
− 1  cot cot −1
2 2  tan α2 tan β2  2 2

β α γ α β 
⇒ cot + cot = cot  cot cot − 1 
2 2 2 2 2 
β α α β γ γ
⇒ cot + cot = cot cot cot − cot
2 2 2 2 2 2
β α γ α β γ
⇒ cot + cot + cot = cot cot cot ¡
2 2 2 2 2 2
FSc-I / 10.2 - 7

Question # 13
Since α , β and γ are angles of triangle therefore
α + β + γ = 180 ⇒ α + β = 180 − γ
Now tan (α + β ) = tan (180 − γ )
tan α + tan β tan α + tan β
⇒ = tan ( 2(90) − γ ) ⇒ = − tan γ
1 − tan α tan β 1 − tan α tan β
⇒ tan α + tan β = − tan γ (1 − tan α tan β )
⇒ tan α + tan β = − tan γ + tan α tan β tan γ
⇒ tan α + tan β + tan γ = tan α tan β tan γ
Dividing through out by tan α tan β tan γ
tan α tan β tan γ tan α tan β tan γ
+ + =
tan α tan β tan γ tan α tan β tan γ tan α tan β tan γ tan α tan β tan γ
⇒ cot β cot γ + cot α cot γ + cot α cot β = 1
⇒ cot α cot β + cot β cot γ + cot γ cot α = 1 ¡

Question # 14 (i)
12sin θ + 5cosθ
Let 12 = r cos ϕ and 5 = r sin ϕ
Squaring and adding 5 r sin ϕ
=
(12) 2 + (5) 2 = r 2 cos 2 ϕ + r 2 sin 2 ϕ 12 r cos ϕ
⇒ 144 + 25 = r 2 ( cos 2 ϕ + sin 2 ϕ ) 5
= tan ϕ
⇒ 169 = r 2 (1) 12
5
⇒ r = 169 = 13 ⇒ ϕ = tan −1
12
Now
12sin θ + 5cosθ = r cos ϕ sin θ + r sin ϕ cosθ
= r ( cos ϕ sin θ + sin ϕ cosθ )
5
= r sin (θ + ϕ ) where r = 13 and ϕ = tan −1
12
Question # 14 (ii)
3sinθ − 4cosθ
Let 3 = r cos ϕ and −4 = r sin ϕ
Squaring and adding − 4 r sin ϕ
=
(3)2 + (−4)2 = r 2 cos 2 ϕ + r 2 sin 2 ϕ 3 r cos ϕ
⇒ 9 + 16 = r 2 ( cos 2 ϕ + sin 2 ϕ ) 4
− = tan ϕ
⇒ 25 = r 2 (1) 3
 4
⇒ r = 25 = 5 ⇒ ϕ = tan −1  − 
 3
Now
3sin θ − 4cosθ = r cos ϕ sin θ + r sin ϕ cosθ
= r ( cos ϕ sin θ + sin ϕ cosθ )
 4
= r sin (θ + ϕ ) where r = 5 and ϕ = tan −1  − 
 3

Made by: Atiq ur Rehman (mathcity@gmail.com) http://www.mathcity.org


FSc-I / 10.2 - 8

Question # 14 (iii)
sinθ − cosθ
Let 1 = r cos ϕ and −1 = r sin ϕ
Squaring and adding −1 r sin ϕ
=
(1)2 + (−1)2 = r 2 cos 2 ϕ + r 2 sin 2 ϕ 1 r cosϕ
⇒ 1 + 1 = r 2 ( cos 2 ϕ + sin 2 ϕ ) −1 = tanϕ
⇒ 2 = r 2 (1) ⇒ ϕ = tan −1 ( −1)
⇒ r= 2
Now
sin θ − cosθ = r cosϕ sin θ + r sin ϕ cosθ
= r ( cos ϕ sin θ + sin ϕ cosθ )
= r sin (θ + ϕ ) where r = 2 and ϕ = tan −1 ( −1)

Question # 14 (iv)
5sinθ − 4cosθ
Let 5 = r cosϕ and −4 = r sin ϕ
Squaring and adding − 4 r sin ϕ
=
(5)2 + (−4)2 = r 2 cos 2 ϕ + r 2 sin 2 ϕ 5 r cos ϕ
⇒ 25 + 16 = r 2 ( cos 2 ϕ + sin 2 ϕ ) 4
− = tan ϕ
⇒ 41 = r 2 (1) 5
 4
⇒ r = 41 ⇒ ϕ = tan −1  − 
 5
Now
5sin θ − 4cosθ = r cos ϕ sin θ + r sin ϕ cosθ
= r ( cos ϕ sin θ + sin ϕ cosθ )
 4
= r sin (θ + ϕ ) where r = 41 and ϕ = tan −1  − 
 5
Question # 14 (v)
sinθ + cosθ
Let 1 = r cos ϕ and 1 = r sin ϕ
Squaring and adding 1 r sin ϕ
=
(1)2 + (1)2 = r 2 cos 2 ϕ + r 2 sin 2 ϕ 1 r cosϕ
⇒ 1 + 1 = r 2 ( cos 2 ϕ + sin 2 ϕ ) 1 = tanϕ
⇒ 2 = r 2 (1) ⇒ ϕ = tan −1 (1)
⇒ r= 2
Now
sin θ + cosθ = r cos ϕ sin θ + r sin ϕ cosθ
= r ( cos ϕ sin θ + sin ϕ cosθ )
= r sin (θ + ϕ ) where r = 2 and ϕ = tan −1 (1)

Question # 14 (vi)
Do yourself as above J

Made by: Atiq ur Rehman (mathcity@gmail.com) http://www.mathcity.org


mathcity.org Exercise 10.3 (Solutions)
Textbook of Algebra and Trigonometry for Class XI
Merging man and maths Available online @ http://www.mathcity.org, Version: 1.0.2

Question # 1
12 π
(i) sin α = ; 0 <α <
13 2
Since cosα = ± 1 − sin 2 α
As α is in the first quadrant so value of cos is +ive
cosα = 1 − sin 2 α
2
 12  144 25 5
= 1−   = 1− = ⇒ cosα =
 13  169 169 13
12
sin α 12
and tan α = = 13 ⇒ tan α =
cosα 5 5
13
Now sin 2α = 2sin α cosα
 12  5  120
= 2    ⇒ sin 2α =
 13  13  169
cos 2α = cos α − sin α
2 2

2 2
 5   12  25 144 119
=  −  = − ⇒ cos 2α =
 13   13  169 169 169
2α tan
tan 2α =
1 − tan 2 α

=
( )
2 12
5 =
24
5 =
24
5 = − 24 ⋅ 25
( ) 1 − 144 −119
2
1 − 12 5 119
5 25 25
120
⇒ tan 2α =
119
3 π
(ii) cosα = ; 0 <α <
5 2
Hint: First find sinα and tanα then solve as above
Question # 2
cosα sin α cos 2 α − sin 2 α
L.H.S = cot α − tan α = − =
sin α cosα sin α cosα
2(cos α − sin α ) 2 cos 2α
2 2
= = = 2cot 2α = R.H.S 8
2sin α cosα sin 2α
Question # 3
sin 2α 2sin α cosα 1 + cos 2α
L.H.S = = Q cos α =
2

1 + cos 2α 2cos 2 α 2
sin α Q 2 cos α = 1 + cos 2α
2

= = tanα = R.H.S 8
cosα
www.mathcity.org
FSc I - 10.3 - 2

Question # 4
α α 1 − cos α
2sin 2 Q sin =
2

1 − cosα 2
L.H.S = = 2 2
sin α α α 2 α
2sin cos Q 2sin = 1 − cos α
2 2 2
α α α
sin sin α = 2sin
2 = tan α = R.H.S
cos
= 8 2 2
α 2
cos
2
Question # 5
cosα − sin α
L.H.S =
cosα + sin α
cosα − sin α cosα − sin α
= × Rationalize
cosα + sin α cosα − sin α
(cosα − sin α )2 cos 2 α + sin 2 α − 2sin α cosα
= =
cos 2 α − sin 2 α cos 2α
1 − sin 2α 1 sin 2α
= = − = sec2α − tan 2α = R.H.S 8
cos 2α cos 2α cos 2α
Question 6
1 + sin α α α
L.H.S = Q sin
2
+ cos
2
=1
1 − sin α 2 2
α α α α α α
sin 2 + cos 2 + 2sin cos sin α = 2sin cos
= 2 2 2 2 2 2
α α α α
sin 2 + cos 2 − 2sin cos
2 2 2 2
2
 α α α α
 sin + cos  sin + cos
= 
2 2 2 2
= = R.H.S 8
 α α
2
α α
sin − cos
 sin − cos  2 2
 2 2

Question # 7
cosecθ + 2cosec 2θ
L.H.S =
secθ
1 2
+
 1   1 
= sin θ sin 2θ = cosθ 
2 1
+  = cosθ  + 
1  sin θ 2sin θ cosθ   sin θ sin θ cosθ 
cosθ
θ
θ + θ + 2cos 2
 cos 1  cos 1 2
= cosθ  = =
 sin θ cosθ  sin θ θ θ
2sin cos
2 2
θ
= cos = R.H.S 8
2
www.mathcity.org
FSc I - 10.3 - 3
Question # 8
 sin α  sin 2α 
L.H.S = 1 + tan α tan 2α = 1 +   
 cosα  cos 2α 
cosα cos 2α + sin α sin 2α cos(2α − α )
= =
cosα cos 2α cosα cos 2α
cosα 1
= = = sec2α = R.H.S 8
cosα cos 2α cos 2α
Question # 9
2sin θ sin 2θ Q cos3θ = 4cos 3 θ − 3cosθ
L.H.S =
cosθ + cos3θ 1 + cos 2θ
2sin θ sin 2θ Q cos 2 θ =
= 2
cosθ + 4cos3 θ − 3cosθ ∴ 2cos θ − 1 = cos 2θ
2

2sin θ sin 2θ 2sin θ sin 2θ


= =
4cos3 θ − 2cosθ 2cosθ (2cos 2 θ − 1)
sin θ sin 2θ
= = tanθ tan 2θ = R.H.S 8
cosθ cos 2θ
Question # 10
sin 3θ cos3θ sin 3θ cosθ − cos3θ sin θ
L.H.S = − =
sin θ cosθ sin θ cosθ
sin ( 3θ − θ ) sin 2θ 2sin θ cosθ
= = = = 2 = R.H.S 8
sin θ cosθ sin θ cosθ sin θ cosθ
Question # 11
cos3θ sin 3θ cos3θ sin θ + sin 3θ cosθ
L.H.S = + =
cosθ sin θ sin θ cosθ
sin(θ + 3θ ) sin 4θ 2sin 2θ cos 2θ
= = =
sin θ cosθ sin θ cosθ sin θ cosθ
2(2sin θ cosθ )cos 2θ
= = 4cos 2θ = R.H.S 8
sin θ cosθ
Question # 12
sin θ2 cos θ2 sin 2 θ2 + cos 2 θ2
+
tan θ2 + cot θ2 cos θ2 sin θ2 sin θ2 cos θ2
L.H.S = = =
cot θ2 − tan θ2 cos θ2 − sin θ2 cos 2 θ2 − sin 2 θ2
sin θ2 cos θ2 sin θ2 cos θ2
sin 2 θ2 + cos 2 θ2 1
= = = secθ = R.H.S 8
cos 2 θ2 − sin 2 θ2 cosθ

Question # 13
sin 3θ cos3θ sin 3θ sin θ + cos3θ cosθ
L.H.S = + =
cosθ sin θ sin θ cosθ
cos(3θ − θ ) cos 2θ 2cos 2θ
= = =
sin θ cosθ sin θ cosθ 2sin θ cosθ
2cos 2θ
= = 2cot 2θ = R.H.S 8
sin 2θ
www.mathcity.org
FSc I - 10.3 - 4
Question # 14
2
 1 − cos 2θ 
( )
2
sin θ = sin θ = 
4 2

 2 
1 − 2cos 2θ + cos 2θ
( )
2
1
= = 1 − 2cos 2θ + cos 2 2θ
4 4
1 1 + cos 4θ  1  2 − 4cos 2θ + 1 + cos 4θ 
= 1 − 2cos 2θ +  =  
4 2  4 2 
1
= ( 3 − 4cos 2θ + 2cos 4θ ) 8
8

Question # 15
(i) Let θ = 18o ⇒ 5θ = 90o ⇒ 3θ + 2θ = 90o ⇒ 2θ = 90o − 3θ
sin 2θ = sin(90 − 3θ )
⇒ sin 2θ = cos3θ Q cos3θ = 4cos 3 θ − 3cosθ
⇒ 2sinθ cosθ = 4cos θ − 3cosθ
3
∴ sin 2θ = 2sin θ cos θ
⇒ 2sinθ = 4cos 2 θ − 3 ÷ing by cosθ
⇒ 2sinθ = 4(1 − sin 2 θ ) − 3
⇒ 2sinθ = 4 − 4sin 2 θ − 3 ⇒ 2sinθ = 1 − 4sin 2 θ
⇒ 4sin 2 θ + 2sinθ − 1 = 0
This is quadratic in sinθ with a = 4 , b = 1 and c = −1
−2 ± (2)2 − 4(4)(−1) −2 ± 4 + 16 −2 ± 20
sin θ = = =
2(4) 8 8
−2 ± 2 5 −1 ± 5
= =
8 8
o
Since θ = 18 lies in the first quadrant so value of sin can not be negative therefore
−1 + 5 5 −1
sin θ = ⇒ sin18o = Q θ = 18o
4 4
Now
2
o o
 5 −1
o
cos 18 = 1 − sin 18
2 2
⇒ cos 18 = 1 − 
2

 4 
( 5)
2

o
− 2 5 +1
5 − 2 5 +1
⇒ cos 18 = 1 −
2
=1−
16 16
6−2 5 16 − 6 + 5 10 + 2 5
=1− = =
16 16 16
10 + 2 5 10 + 2 5
⇒ cos18o = ⇒ cos18o =
16 4

(ii)
1 + cos 2θ
Since cos 2 θ =
2
⇒ cos 2θ = 2cos 2 θ − 1
⇒ cos 2(18) = 2cos 2 (18) − 1
www.mathcity.org
FSc I - 10.3 - 5
2
 10 + 2 5 
⇒ cos36 = 2   −1
 4 
 
 10 + 2 5  10 + 2 5
= 2  −1 = −1
 16  8
10 + 2 5 − 8 2+2 5 1+ 5
= = ⇒ cos36o =
8 8 2
Now sin 36 = 1 − cos 36
2 2

( 5)
2
1+ 5 
2
1+ 2 5 +
=1−   = 1 −
 2  16
1+ 2 5 + 5 6+ 2 5
=1− =1−
16 16
16 − 6 − 2 5 10 − 2 5
= =
16 16
10 − 2 5 10 − 2 5
⇒ sin 36o = ⇒ sin 36o =
16 4
(iii)
Now sin(90 − 36) = cos36o Q sin(90 − θ ) = cosθ
1+ 5
⇒ sin54o = cos36o ⇒ sin 54o =
2
and cos(90 − 36) = sin36o
10 − 2 5
⇒ cos54o = sin36o ⇒ cos54o =
4
(iv)
Now sin(90 − 18) = cos18o Q sin(90 − θ ) = cosθ

10 + 2 5
⇒ sin 72o = cos18o ⇒ sin 72o =
4
and cos(90 − 18) = sin18o
5 −1
⇒ cos 72o = sin18o ⇒ cos 72o =
4

Alternative Method for Q # 15 (iii)


Let θ = 54o ⇒ 5θ = 270o ⇒ 3θ + 2θ = 270o ⇒ 2θ = 270o − 3θ
sin 2θ = sin(270 − 3θ )
⇒ sin 2θ = sin(3(90) − 3θ ) Q cos3θ = 4cos 3 θ − 3cosθ
⇒ sin 2θ = − cos3θ ∴ sin 2θ = 2sin θ cosθ
⇒ 2sinθ cosθ = −(4cos3 θ − 3cosθ )
⇒ 2sinθ cosθ = − 4cos3 θ + 3cosθ )
⇒ 2sinθ = − 4cos 2 θ + 3 ÷ing by cosθ
⇒ 2sinθ = − 4(1 − sin 2 θ ) + 3
⇒ 2sinθ = − 4 + 4sin 2 θ + 3 ⇒ 2sinθ = 4sin 2 θ − 1
⇒ 4sin 2 θ − 2sinθ − 1 = 0
This is quadratic in sinθ with a = 4 , b = 1 and c = −1
www.mathcity.org
FSc I - 10.3 - 6

−(−2) ± (−2) 2 − 4(4)(−1) 2 ± 4 + 16 2 ± 20


sin θ = = =
2(4) 8 8
2±2 5 1± 5
= =
8 4
o
Since θ = 54 lies in the first quadrant so value of sin can not be negative therefore
1+ 5 1+ 5
sin θ = ⇒ sin 54o = Q θ = 54o
4 4

Now
2
o o o
1+ 5 
cos 54 = 1 − sin 54
2 2
⇒ cos 54 = 1 − 
2

 4 
( )
2

o
5 + 2 5 +1
5 + 2 5 +1
⇒ cos 542
=1− =1−
16 16
6+ 2 5 16 − 6 − 5 10 − 2 5
=1− = =
16 16 16
10 − 2 5 10 − 2 5
⇒ cos54o = ⇒ cos54o =
16 4

Review:
● sin 2 θ + cos2 θ = 1 ● 1 + tan 2 θ = sec2 θ ● 1 + cot 2 θ = csc2 θ
● sin(−θ ) = − sin θ ● cos(−θ ) = cosθ ● tan(−θ ) = − tan θ
………………………………………………………………………………………………………………………………………
● sin (α + β ) = sin α cos β + cosα sin β ● sin (α − β ) = sin α cos β − cosα sin β
● cos (α + β ) = cosα cos β − sin α sin β ● cos (α − β ) = cosα cos β + sin α sin β
tan α + tan β tan α − tan β
● tan (α + β ) = ● tan (α − β ) =
1 − tan α tan β 1 + tan α tan β
………………………………………………………………………………………………………………………………………
● sin 2θ = 2sinθ cosθ 2 2
● cos 2θ = cos θ − sin θ
tan 2θ
● tan 2θ =
1 − tan 2 θ
………………………………………………………………………………………………………………………………………
θ 1 − cosθ θ 1 + cosθ θ 1 − cosθ
● sin 2 = ● cos 2 = ● tan 2 =
2 2 2 2 2 1 + cosθ
………………………………………………………………………………………………………………………………………
3tan θ − tan 3 θ
● sin 3θ = 3sin θ − 4sin θ
3
● cos3θ = 4cos θ − 3cosθ
3
● tan 3θ = 2
1 − 3tan θ
………………………………………………………………………………………………………………………………………
2tan θ 2
1 − tan θ
● sin 2θ = ● cos 2θ =
1 + tan 2 θ 1 + tan 2 θ

…………………………………………………………………..
For updates and news visit
http://www.mathcity.org

Error Analyst

Waiting for Someone


mathcity.org Exercise 10.4 (Solutions)
Textbook of Algebra and Trigonometry for Class XI
Page 336

Merging man and maths Available online @ http://www.mathcity.org, Version: 1.0.0

Question # 1
(i) Since 2sin α cos β = sin(α + β ) + sin(α − β )
Put α = 3θ and β = θ
2sin 3θ cosθ = sin(3θ + θ ) + sin(3θ − θ )
= sin 4θ + sin 2θ q
(ii) Since 2cosα cos β = sin(α + β ) − sin(α − β )
Put α = 5θ and β = 3θ
2cos5θ cos3θ = sin(5θ + 3θ ) − sin(5θ − 3θ )
= sin8θ − sin 2θ q
(iii) Since 2sin α cos β = sin(α + β ) + sin(α − β )
Put α = 5θ and β = 2θ
2sin 5θ cos 2θ = sin(5θ + 2θ ) + sin(5θ − 2θ )
= sin 7θ + sin 3θ q
1
⇒ sin 5θ cos 2θ = ( sin 7θ + sin 3θ )
2
(iv) Since − 2sin α sin β = cos(α + β ) − cos(α − β )
Put α = 7θ and β = 2θ
− 2sin α sin β = cos(α + β ) − cos(α − β )
−2sin 7θ sin 2θ = cos(7θ + 2θ ) − cos(7θ − 2θ )
= cos9θ − cos5θ
2sin 7θ sin 2θ = cos5θ − cos9θ q
(v) Since 2cosα sin β = sin(α + β ) − sin(α − β )
Put α = x + y , β = x − y
2cos( x + y )sin( x − y ) = sin( x + y + x − y ) − sin( x + y − x + y )
= sin 2 x − sin 2 y
1
⇒ cos( x + y )sin( x − y ) = ( sin 2 x − sin 2 y ) q
2
(vi) Since 2cosα cos β = cos (α + β ) + cos (α − β )
Put α = 2 x + 30o and β = 2 x − 30o
( ) ( ) ( ) (
2cos 2 x + 30o cos 2 x − 30o = cos 2 x + 30o + 2 x − 30o + cos 2 x + 30o − 2 x + 30o )
( )
= cos ( 4 x ) + cos 60o

cos ( 2 x + 30 ) cos ( 2 x − 30 ) = ( cos 4 x + cos60 )


o o 1 o
⇒ q
2
(vi) Since − 2sin α sin β = cos(α + β ) − cos(α − β )
Put α = 12o and β = 46o
− 2sin12o sin 46o = cos(12 + 46) − cos(12 − 46)
= cos58 − cos(−34)
= cos58 − cos34 Q cos(−θ ) = cosθ
1
⇒ sin12o sin 46o = − ( cos58 − cos34 )
2
1
= ( cos34 − cos58 ) q
2
FSc-I / Ex 10.4 - 2

(viii) Since − 2sin α sin β = cos(α + β ) − cos(α − β )


Put α = x + 45o and β = x − 45o
( ) ( ) { } {
− 2sin x + 45o sin x − 45o = cos ( x + 45o ) + ( x − 45o ) − cos ( x + 45o ) − ( x − 45o ) }
= cos 2 x − cos90o

( ) ( ) 1
⇒ sin x + 45o sin x − 45o = cos90o − cos 2 x
2
q

Question # 2
α + β  α − β 
(i) Since sin α + sin β = 2sin   cos  
 2   2 
Put α = 5θ , β = 3θ
 5θ + 3θ   5θ − 3θ 
sin 5θ + sin 3θ = 2sin   cos  
 2   2 
 8θ   2θ 
= 2sin   cos   = 2sin 4θ cosθ q
 2   2 
α + β  α − β 
(ii) Since sin α − sin β = 2cos   sin  
 2   2 
Put α = 8θ , β = 4θ
 8θ + 4θ   8θ − 4θ 
sin 8θ − sin 4θ = 2cos   sin  
 2   2 
= 2cos6θ sin 2θ
(iii) Do yourself J
α + β  α − β 
(iv) Since cosα − cos β = −2sin   sin  
 2   2 
Put α = 7θ , β = θ
 7θ + θ   7θ − θ 
cos 7θ − cosθ = −2sin   sin  
 2   2 
 8θ   6θ 
= −2sin   sin   = −2sin 4θ sin 3θ q
 2   2 
α + β  α − β 
(v) Since cosα + cos β = 2cos   cos  
 2   2 
Put α = 12 , β = 48
 12 + 48   12 − 48 
cos12 + cos 48 = 2cos   cos  
 2   2 
 60   −36 
= 2cos   cos   = 2cos30cos ( −18)
 2   2 
= 2cos30o cos18o Q cos(−θ ) = cosθ q
α + β  α − β 
(vi) Since sin α + sin β = 2sin   cos  
 2   2 
Put α = x + 30 , β = x − 30
 x + 30 + x − 30   x + 30 − x + 30 
sin( x + 30) + sin( x − 30) = 2sin   cos  
 2   2 
 2x   60 
= 2sin   cos   = 2sin x cos30
 2   2 
FSc-I / Ex 10.4 - 3

Question # 3
sin 3 x − sin x
(i) L.H.S =
cos x − cos3 x
 3x + x   3x − x   4x   2x 
2cos   sin   cos   sin  
=  2   2  =  2   2 
 x + 3x   x − 3x   4 x   −2 x 
−2sin   sin   − sin   sin  
 2   2   2   2 
cos ( 2 x ) sin ( x )
= = cot 2x = R.H.S q
+ sin ( 2 x ) sin ( x )
(ii) Do yourself J
sin α − sin β
(iii) L.H.S =
sin α + sin β
α + β  α −β 
2cos   sin  
=  2   2  = cot  α + β  tan  α − β  = R.H.S q
   
α + β  α −β   2   2 
2sin   cos  
 2   2 

Question # 4
(i) L.H.S = cos 20o + cos100o + cos140o
( )
= cos100o + cos 20o + cos140o
 100 + 20   100 − 20  o
= 2cos   cos   + cos140
 2   2 
1
= 2cos60o cos 40o + cos140o = 2   cos 40o + cos140o
2
 140 + 40   140 − 40 
= cos140o + cos 40o = 2cos   cos  
 2   2 
= 2cos90 cos50 = 2 ( 0 ) cos50 = 0 = R.H.S
o o o
q
π  π 
(ii) L.H.S = sin  − θ  sin  + θ 
4  4 
 π π  π π 
=  sin cosθ − cos sin θ  sin cosθ + cos sin θ 
 4 4  4 4 
 1 1  1 1 
= cosθ − sin θ  cosθ + sin θ 
 2 2  2 2 
2 2
 1   1  1 1
= cosθ  −  sin θ  = cos 2 θ − sin 2 θ
 2   2  2 2
1
( ) 1
= cos 2 θ − sin 2 θ = cos 2θ = R.H.S
2 2
q

sin θ + sin 3θ + sin 5θ + sin 7θ


(iii) L.H.S =
cosθ + cos3θ + cos5θ + cos7θ
=
( sin 7θ + sin θ ) + ( sin 5θ + sin 3θ )
( cos7θ + cosθ ) + ( cos5θ + cos3θ )
 7θ + θ   7θ − θ   5θ + 3θ   5θ − 3θ 
2sin   cos   + 2sin   cos  
=  2   2   2   2 
 7θ + θ   7θ − θ   5θ + 3θ   5θ − 3θ 
2cos   cos   + 2cos   cos  
 2   2   2   2 
FSc-I / Ex 10.4 - 4
2sin 4θ cos3θ + 2sin 4θ cosθ
=
2cos 4θ cos3θ + 2cos 4θ cosθ
2sin 4θ ( cos3θ + cosθ ) sin 4θ
= = = tan 4θ = R.H.S q
2cos 4θ ( cos3θ + cosθ ) cos 4θ

Question # 5
(i) L.H.S = cos 20o cos 40o cos 60o cos80o
1 1
= cos 20o cos 40o   cos80o = cos80o cos 40o cos 20o
2 2
1
4
( )
= 2cos80o cos 40o cos 20o = ( cos ( 80 + 40 ) + cos ( 80 − 40 ) ) cos 20o
1
4
1 1 
1
( )
= cos120o + cos 40o cos 20o =  − + cos 40o  cos 20o
4 4 2 
1
8
1
4
1 1
= − cos 20o + cos 40o cos 20o = − + 2cos 40o cos 20o
8 8
( )
1 1
= − cos 20o + ( cos(40 + 20) + cos(40 − 20) )
8 8
1 1 1 1 1 
= − cos 20o + ( cos 60 + cos 20 ) = − cos 20o +  + cos 20 
8 8 8 8 2 
1 1 1 1
= − cos 20o + + cos 20o = = R.H.S q
8 16 8 16
π 2π π 4π
(ii) L.H.S = sin sin sin sin
9 9 3 9
o o
180 2(180 ) (180o ) 4(180o )
= sin sin sin sin Q π = 180o
9 9 3 9
3
= sin 20o sin 40o sin 60o sin80o = sin 20o sin 40o sin80o
2
=
2
3
sin 80o sin 40o sin 20o = −
4
3
( −2sin 80o sin 40o ) sin 20o

3
=− ( cos(80 + 40) − cos(80 − 40) ) sin 20o
4
3 1 
=−
4
3
( cos120o − cos 40o ) sin 20o = − 
4  2
− − cos 40o  sin 20o

=
8
3
sin 20o +
4
3
cos 40o sin 20o =
8
3
sin 20o +
8
3
(
2cos 40o sin 20o )
3 3
= sin 20o + ( sin(40 + 20) − sin(40 − 20) )
8 8
31 
=
8
3
sin 20o +
8
3
(
sin 60o − sin 20o = )8
3
sin 20o + 
8 2
− sin 20o 

3 3 3 3
= sin 20o + − sin 20o = = R.H.S q
8 16 8 16
(iii) Do yourself as above J

Made by: Atiq ur Rehman (atiq@mathcity.org) http://www.mathcity.org

The End
Exercise 12.2 (Solutions)
mathcity.org Textbook of Algebra and
a nd Trigonometry for Class XI
Merging man and maths Available online @ http://www.mathcity.org, Version: 1.0.0
FSc-I- Ex 12.2 - 2 www.mathcity.org

******************************************
Available online at http://www.mathcity.org
******************************************
Exercise 12.3 (Solutions)
mathcity.org Textbook of Algebra and
a nd Trigonometry for Class XI
Merging man and maths Available online @ http://www.mathcity.org, Version: 1.0.0
FSc-I- Ex 12.3 - 2 www.mathcity.org
www.mathcity.org FSc-I- Ex 12.3 - 3
FSc-I- Ex 12.3 - 4 www.mathcity.org
www.mathcity.org FSc-I- Ex 12.3 - 5
FSc-I- Ex 12.3 - 6 www.mathcity.org

******************************************
Available online at http://www.mathcity.org
******************************************
mathcity.org Exercise 12.5 (Solutions)
Textbook of Algebra and Trigonometry for Class XI
Page 371

Merging man and maths Available online @ http://www.mathcity.org, Version: 1.0.0

Question # 1
Do yourself as Question # 2

Question # 2
b = 12.5 , c = 23 , α = 38o 20′
By law of cosine
a 2 = b 2 + c 2 − 2bc cosα
= (12.5)2 + (23)2 − 2(12.5)(23)cos38o 20′
= 156.25 + 529 − 575(0.7844) = 234.21
⇒ a = 234.21 ⇒ a = 15.304
Again by law of cosine
c 2 + a 2 − b2 (23) 2 + (15.304)2 − (12.5)2
cos β = =
2ca 2(23)(15.304)
529 + 234.21 − 156.25 606.96
= = = 0.8622
703.984 703.984
β = cos −1 (0.8622) ⇒ β = 30o26′
Now
a 2 + b 2 − c2 (15.304)2 + (12.5)2 − (23)2
cos γ = =
2ab 2(15.304)(12.5)
234.212 + 156.25 − 529 138.538
= =− = −0.3621
382.6 382.6
o
β = cos −1 (− 0.3621) ⇒ β = 11114 ′

Question # 3
a = 3 − 1 = 0.732 , b = 3 + 1 = 2.732 , γ = 60o
By law of cosine
c 2 = a 2 + b 2 − ab cos γ
= (0.732)2 + (2.732)2 − 2(0.732)(2.732)cos 60o
= 0.5358 + 7.4638 − 1.9998 = 5.9998 ≈ 6
⇒ c = 6 = 2.449
Again by law of cosines
b2 + c2 − a 2 (2.732) 2 + (2.449)2 − (0.732)
cosα = =
2bc 2(2.732)(2.449)
7.4638 + 5.9976 − 0.5358 12.9256
= = = 0.9659
13.3813 13.3813
⇒ α = cos −1 (0.9659) ⇒ α = 15o
Since in any triangle
α + β + γ = 180
⇒ β = 180 − α − γ = 180 − 15 − 60 ⇒ β = 105o

Question # 4 & 5
Do yourself as above
FSc-I - 12.5 - 2

Question # 6
a = 36.21 , b = 42.09 , γ = 44o 29′
Since α + β + γ = 180
⇒ α + β = 180 − γ
= 180 − 44o 29′
⇒ α + β = 135o31′ ………….. (i)
By law of tangent
α − β 
tan  
a −b  2 
=
a+b α + β 
tan  
 2 
α − β 
tan  
36.21 − 42.09  2 
⇒ =
36.21 + 42.09  135o31′ 
tan  
 2 
α − β  α − β 
tan   tan  
−5.88  2   2 
⇒ = ⇒ − 0.0751 =
78.3 (
tan 67o 45′ ) 2.4443
α − β 
⇒ tan   = −0.0751( 2.4443)
 2 
= −0.1836
α−β
⇒ = tan −1 ( −0.1836 )
2
α−β
⇒ = −10o 24′ ⇒ α − β = −20o 48′ ………….. (ii)
2
Adding (i) & (ii)
α + β = 135o31′
α − β = − 20o48′

2α = 114o 43′ ⇒ α = 57o 22′


Putting value of α in eq. (i)
57o 22′ + β = 135o 22′
⇒ β = 135o 22′ − 57o22′ ⇒ β = 78o9′
Now by law of sine
c a
=
sin γ sin α
c 36.21
⇒ =
sin 44 29′ sin 57 o 22′
o

36.21
⇒ c= o
⋅ sin 44o29′
sin 57 22′
36.21
= ⋅ 0.7007 ⇒ c = 30.13
0.8421
Question # 7, 8 & 9
Do yourself as above
Made by: Atiq ur Rehman (atiq@mathcity.org), http://www.mathcity.org
FSc-I - 12.5 - 3

Question # 10
b = 61 , c = 32 , α = 59o30′ *Correction
Since α + β + γ = 180
⇒ β + γ = 180 − α
= 180 − 59o30′
⇒ β + γ = 120o30′ ………….. (i)
By law of tangent
 β −γ   β −γ 
tan   tan  
b−c  2  61 − 32  2 
= ⇒ =
b+c  β +γ  61 + 32  120o30′ 
tan   tan  
 2   2 
 β −γ   β −γ 
tan   tan  

29
=  2 
⇒ 0.3118 =  2 
(
93 tan 60o15′ ) 1.7496
 β −γ 
⇒ tan   = 0.3118 (1.7496 )
 2 
= 0.5455
β −γ
⇒ = tan −1 ( 0.5455 )
2
β −γ
⇒ = 28o37′ ⇒ β − γ = 57o14′ ………….. (ii)
2
Adding (i) & (ii)
β + γ = 120o30′
β − γ = 57o14′

2β = 177o 44′ ⇒ β = 88o52′


Putting value of α in eq. (i)
88o52′ + γ = 120o30′
⇒ γ = 120o30′ − 88o52′ ⇒ γ = 31o38′
Now by law of sine
c a
=
sin γ sin α
c 36.21
⇒ =
sin 31o38′ sin 59o30′
36.21
⇒ c= o
⋅ sin 31o38′
sin 59 30′
36.21
= ⋅ 0.5244 ⇒ c = 22.04
0.8616

Question # 11
Let a : b = 3 : 2
a 3 3
i.e. = ⇒ a= b
b 2 2
o
and γ = 57
Since α + β + γ = 180
⇒ α + β = 180 − γ
FSc-I - 12.5 - 4
= 180 − 57 ⇒ α + β = 123o ……….. (i)
By law of tangent
α − β  3 α − β 
tan   b − b tan  
a −b  2  2  2 
= ⇒ =
a+b α + β  3 o
b + b tan  123 
tan  
 2  2  2 
1 α − β  α − β 
b tan   tan  
⇒ 2 =  2  1
⇒ =  2 
5
b tan 61 30′(o
) 5 1.8418
2
α − β  1
⇒ tan   = (1.8418 )
 2  5
= 0.3684
α−β
⇒ = tan −1 ( 0.3684 ) = 20o13′
2
⇒ α − β = 40o26′ ……….. (ii)
Adding (i) & (ii)
α + β = 123o
α − β = 40o 27′

2α = 163o 27′ ⇒ α = 81o 44′


Putting value of α in eq. (i)
81o 44′ + β = 123o
⇒ β = 123o − 81o 44′ o
⇒ β = 4116′

Question # 12
uuur C
r = c = 40 N
Since uuu
AB
BC = a = 30 N
m∠B = β = 147o 25′ a = 30
uuur
AC = b = ?
By law of cosine β
b2 = c 2 + a 2 − 2ca cos β A c = 40 B
o
= (40) + (30) − 2(40)(30)cos147 25′
2 2

= 1600 + 900 − 2400(−0.8426)


= 4522.26
⇒ b = 4522.26 = 67.248
uuur
i.e. AC = 67.248 N

If you found any error, submit at


http://www.mathcity.org/error

Error Analyst

Waiting for someone

Tuesday, March 22, 2005 *


Updated: March 4, 2006 *
Exercise 12.6 (Solutions)
mathcity.org Textbook of Algebra and
a nd Trigonometry for Class XI
Merging man and maths Available online @ http://www.mathcity.org, Version: 1.0.0
FSc-I- Ex 12.6 - 2 www.mathcity.org
www.mathcity.org FSc-I- Ex 12.6 - 3
FSc-I- Ex 12.6 - 4 www.mathcity.org

******************************************
Available online at http://www.mathcity.org
******************************************
Exercise 12.7 (Solutions)
mathcity.org Textbook of Algebra and
a nd Trigonometry for Class XI
Merging man and maths Available online @ http://www.mathcity.org, Version: 1.0.0
FSc-I- Ex 12.7 - 2 www.mathcity.org
www.mathcity.org FSc-I- Ex 12.7 - 3
FSc-I- Ex 12.7 - 4 www.mathcity.org

******************************************
Available online at http://www.mathcity.org
******************************************
mathcity.org Exercise 12.8 (Solutions)
Textbook of Algebra and Trigonometry for Class XI
Page 385

Merging man and maths Available online @ http://www.mathcity.org, Version: 1.0.0

IMPORTANT FORMULAS
Ð The Law of Cosine
Ÿ a 2 = b 2 + c 2 − 2bc cosα Ÿ b2 = c 2 + a 2 − 2ca cos β Ÿ c 2 = a 2 + b 2 − 2ab cos γ
b2 + c2 − a 2 c 2 + a 2 − b2 a 2 + b 2 − c2
Ÿ cosα = Ÿ cos β = Ÿ cos γ =
2bc 2ca 2ab
…………………………………………………………………………………………………………………………………..

Ð The Law of Sine


a b c
Ÿ = =
sin α sin β sin γ
…………………………………………………………………………………………………………………………………..

Ð The Law of Tangent


α−β β −γ γ −α
tan tan tan
a −b 2 b−c 2 c−a 2
Ÿ = Ÿ = Ÿ =
a + b tan α + β b + c tan β + γ c + a tan γ + α
2 2 2
…………………………………………………………………………………………………………………………………..

Ð Half Angle Formulas


α ( s − b)( s − c ) β ( s − c )( s − a ) γ ( s − a )( s − b)
Ÿ sin = Ÿ sin = Ÿ sin =
2 bc 2 ca 2 ab
α s (s − a) β s ( s − b) γ s ( s − c)
Ÿ cos = Ÿ cos = Ÿ cos =
2 bc 2 ac 2 ab
α ( s − b)(s − c) β ( s − c)( s − a ) γ ( s − a )( s − b)
Ÿ tan = Ÿ tan = Ÿ tan =
2 s ( s − a) 2 s ( s − b) 2 s ( s − c)
a+b+c
where s = ⇒ 2s = a + b + c
2
…………………………………………………………………………………………………………………………………..

Ð Area of the Triangle ( = ∆ )


1 1 1
Ÿ ∆ = bc sin α = ca sin β = ab sin γ
2 2 2
a sin β sin γ b sin γ sin α c 2 sin α sin β
2 2
Ÿ ∆= = =
2sin α 2sin β 2sin γ
Ÿ ∆ = s (s − a )( s − b)( s − c ) (Hero’s Formula)
…………………………………………………………………………………………………………………………………..

Ð Circum Radius ( = R )
a b c abc
Ÿ R= = = Ÿ R=
2sin α 2sin β 2sin γ 4∆
…………………………………………………………………………………………………………………………………..

Ð In-Radius ( = r )

Ÿ r=
s
…………………………………………………………………………………………………………………………………..

Ð Escribed Circle
∆ ∆ ∆
Ÿ r1 = Ÿ r2 = Ÿ r3 =
s−a s −b s−c
…………………………………………………………………………………………………………………………………..
FSc-I - 12.8 - 2

Question # 1
α β γ
R.H.S = 4 R sin sin sin
2 2 2
(s − b)( s − c) (s − a )(s − c) ( s − a )( s − b)
= 4R
bc ac ab
(s − b)( s − c)( s − a )( s − c )( s − a )( s − b)
= 4R
(bc)(ac)(ab)
( s − a ) 2 ( s − b) 2 ( s − c ) 2
= 4R
a 2b 2 c 2
( s − a )( s − b)( s − c )
= 4R
abc
 abc  ( s − a )(s − b)( s − c) abc
= 4  Q R=
 4∆  abc 4∆
( s − a )( s − b)( s − c ) s( s − a )( s − b)(s − c)
= =
∆ s∆
∆ 2
= Q ∆ = s ( s − a )( s − b)( s − c )
s∆

= = r = L.H.S
s
Question # 1 (ii)
α β γ
R.H.S = 4 R cos cos cos
2 2 2
s ( s − a ) s ( s − b) s ( s − c )
= 4R
bc ac ab
s 2 ⋅ s ( s − a )( s − b)( s − c ) s2 ∆2
= 4R = 4R 2 2 2
(bc)(ac )(ab) abc
s∆  abc  ∆ abc
= 4R = 4 s Q R=
abc  4 ∆  abc 4∆
=s = L.H.S

Question # 2
We take
β γ α β γ 1
a sin sin sec = a sin sin
2 2 2 2 2 cos α
2
( s − a )( s − c) ( s − a )( s − b) 1
=a
ac ab s( s − a)
bc
( s − a)( s − c) ( s − a )( s − b) bc
=a
ac ab s( s − a)
( s − a )( s − c)( s − a )( s − b)(bc)
=a
(ac)(ab) s ( s − a )
( s − a )( s − b)( s − c) s ( s − a )( s − b)( s − c)
=a =a
a2s a2 s2
FSc-I - 12.8 - 3

s( s − a )( s − b)(s − c) ∆
=a = =r
as s
β γ α
⇒ a sin sin sec = r ……………. (i)
2 2 2
Similarly prove yourself
γ α β
b sin sin sec = r …………. (ii) *Correction
2 2 2
α β γ
c sin sin sec = r …………. (iii) *Correction
2 2 2
From (i), (ii) and (iii)
β γ α γ α β α β γ
r = a sin sin sec = b sin sin sec = c sin sin sec
2 2 2 2 2 2 2 2 2
Question # 3 (i)
α β γ
R.H.S = 4 R sin cos cos
2 2 2
(s − b)( s − c) s ( s − b) s (s − c)
= 4R
bc ac ab
(s − b)( s − c) s ( s − b) s ( s − c ) s 2 ( s − b) 2 ( s − c) 2
= 4R = 4R
(bc)(ac )(ab) a 2b 2c 2
s ( s − b )( s − c) abc s (s − b)( s − c) ( s − a ) abc
= 4R =4 ⋅ Q R=
abc 4∆ abc (s − a) 4∆
s ( s − a )( s − b)( s − c )
=
∆ ( s − a)
∆2 ∆
= = = r1 = R.H.S
∆ (s − a) ( s − a)

Question # 3 (ii) & (iii)


Do yourself
Question # 4 (i)
α
R.H.S = s tan
2
( s − b)( s − c) ( s − b)( s − c) s ( s − a )
=s =s ⋅
s ( s − a) s (s − a) s ( s − a)
s ( s − a )( s − b)( s − c )
=s
s 2 ( s − a) 2
∆2 ∆ ∆
=s 2 =s = = r1 = L.H.S
s (s − a) 2
s ( s − a) ( s − a)

Question # 4 (ii) & (iii)


Do yourself

Question # 5 (i)
L.H.S = r1 r2 + r2 r3 + r3 r1
 ∆  ∆   ∆  ∆   ∆  ∆ 
=  +  +  
 s − a  s − b   s − b  s − c   s − c  s − a 
FSc-I - 12.8 - 4

∆2 ∆2 ∆2
= + +
( s − a )( s − b) ( s − b)( s − c) ( s − c)( s − a )
 1 1 1 
= ∆2  + + 
 ( s − a )( s − b) ( s − b )( s − c) ( s − c)( s − a ) 
 s −c+ s − a + s −b  2 3s − ( a + b + c ) 
= ∆2   =∆  
 ( s − a )( s − b)( s − c )   ( s − a )( s − b)( s − c ) 
 3s − 2 s  a+b+c
= ∆2   Q s=
 ( s − a )( s − b)( s − c )  2
 s s
= ∆2  ⋅ 
 ( s − a )( s − b)(s − c) s 
 s2  2 s 
2
=∆  2
 = ∆  2  = s = R.H.S
2

 s ( s − a )( s − b )( s − c)  ∆ 

Quesiton # 5 (ii)
L.H.S = r r1 r2 r3
 ∆  ∆  ∆  ∆ 
=     
 s  s − a  s − b  s − c 
∆4 ∆4
= = 2 = ∆ 2 = R.H.S
s ( s − a )( s − b)( s − c ) ∆

Question # 5 (iii)
L.H.S = r1 + r2 + r3 − r
∆ ∆ ∆ ∆  1 1 1 1
= + + − = ∆ + + − 
s −a s −b s −c s  s −a s −b s −c s 
 ( s − b) + ( s − a ) s − (s − c)   2s − b − a s−s+c
= ∆ ++  = ∆ + 
 ( s − a )( s − b) s ( s − c)   (s − a )(s − b) s ( s − c ) 
a +b +c −b −a c 
= ∆ +  Q 2s = a + b + c
 ( s − a )(s − b) s( s − c ) 
 c c   1 1 
= ∆ +  = c∆  + 
 ( s − a )( s − b) s ( s − c)   (s − a )(s − b) s ( s − c ) 
 s ( s − c) − ( s − a )( s − b)   s 2 − sc + s 2 − as − bs + ab 
= c∆   = c∆  
 s ( s − a )( s − b )( s − c )   ∆2 
 2 s − s (a + b + c ) + ab 
2
 2 s − s (2 s) + ab 
2
= c  = c 
 ∆   ∆ 
 2 s 2 − 2 s 2 + ab  abc abc
= c  = = 4 ⋅ = 4R = R.H.S
 ∆  ∆ 4∆

Question # 5 (iv)
 ∆  ∆  ∆ 
L.H.S =    
 s − a  s − b  s − c 
∆3 s ∆3
= =
( s − a )( s − b)( s − c ) s ( s − a )( s − b)( s − c )
FSc-I - 12.8 - 5

s ∆3 ∆
= = s ∆ = s2 = s2 r = r s 2 = R.H.S
∆ 2
s

Question # 6
(i) a = 13 , b = 14 , c = 15
a + b + c 13 + 14 + 15
s= = = 21
2 2
s − a = 21 − 13 = 8
s − b = 21 − 14 = 7
s − c = 21 − 15 = 6
So ∆ = s (s − a )( s − b)( s − c )
= 21(8)(7)(6) = 7056 = 84
Now
abc (13)(14)(15)
R= = = 8.125
4∆ 4(84)
∆ 84
r= = =4
s 21
∆ 84
r1 = = = 10.5
s−a 8
∆ 84
r2 = = = 12
s −b 7
∆ 84
r3 = = = 12
s−c 6
(ii) Do yourself as (i)

Question # 7
(i) Do yourself as (ii)
(ii) In equilateral triangle all the sides are equal so a = b = c
a+b+c a+a+a 3a
s = = =
2 2 2
3a 3  1
s−a= − a =  − 1 a = a
2 2  2
Now
∆ = s (s − a )( s − b)( s − c )
3
3a  1 
= s ( s − a )( s − a )( s − a ) = s ( s − a ) 3
=  a
2 2 
3a  a 3  3a 4 3 a2
=   = =
2 8  8 4
3 a2
∆ 4 3 a2 2 3a
Now r= = = ⋅ =
s 3a 4 3a 6
2
abc a ⋅ a ⋅a a a 3 3a
R= = = = ⋅ =
4∆ 4 3a
2
3 3 3 3
4
FSc-I - 12.8 - 6

3a 2
∆ 4 3 a2 2 3a
r1 = = = ⋅ =
s−a 1a 4 a 2
2
∆ ∆ 3a
r2 = = =
s −b s−a 2
∆ ∆ 3a
r3 = = =
s−c s−a 2
Now
3a 3a 3a 3a 3a
r : R : r1 : r 2 : r 3 = : : : :
6 3 2 2 2
1 1 1 1 1
= : : : : ÷ ing by 3a
6 3 2 2 2
= 1 : 2 : 3 : 3 : 3 × ing by 6

Question # 8
α β γ
(i) R.H.S = r 2 cot cot cot
2 2 2
1 1 1
= r2 ⋅ ⋅
α β γ
tan tan tan
2 2 2
1 1 1
= r2 ⋅ ⋅
( s − b)( s − c ) ( s − a )( s − c) ( s − a )(s − b)
s ( s − a) s ( s − b) s (s − c)
s ( s − a) s ( s − b) s ( s − c)
= r2 ⋅ ⋅
( s − b)(s − c) ( s − a )( s − c ) ( s − a )( s − b)
s 3 ( s − a )( s − b)( s − c) s3
=r 2
=r 2

( s − a ) 2 ( s − b) 2 ( s − c ) 2 ( s − a )( s − b )( s − c)
s3 s s4
=r 2
⋅ = r2
( s − a )( s − b)(s − c) s s ( s − a )( s − b)( s − c )

2
s4 ∆ s
2 2
2 s
=r 2
=r =  Q r=
∆2 ∆ s ∆ s
∆ s
2 2
= 2 = ∆ = L.H.S
s ∆
(ii) Do yourself
α β γ
(iii) R.H.S = 4 Rr cos cos cos
2 2 2
s ( s − a ) s ( s − b) s ( s − c)
= 4 Rr ⋅ ⋅
bc ca ab
s ( s − a ) ⋅ s ( s − b) ⋅ s ( s − c)
= 4 Rr
(bc)(ac)(ab)
s 2 ⋅ s ( s − a )( s − b)( s − c ) s 2 ⋅ ∆2
= 4 Rr = 4 Rr
a 2b 2 c 2 a 2b 2c 2
s∆  a b c  ∆  s ∆
= 4 Rr = 4   = ∆ = L.H.S
abc  4∆  s  a b c
FSc-I - 12.8 - 7

Question # 9
1
(i) L.H.S =
2r R
1 4s∆ 2s a+b+c
= = = = Q 2s = a + b + c
 ∆  abc  2 ∆ abc abc abc
2   
 s  4∆ 
a b c
= + +
abc abc abc
1 1 1 1 1 1
= + + = + + = R.H.S
bc ac ab ab bc ca
1 1 1
(ii) R.H.S = + +
r1 r2 r3
1 1 1 s −a s −b s −c
= + + = + +
∆ ∆ ∆ ∆ ∆ ∆
s−a s−b s−c
s − a + s − b + s − c 3s − (a + b + c)
= =
∆ ∆
3s − 2 s
= Q 2s = a + b + c

s 1 1
= = = = L.H.S
∆ ∆ r
s

Question # 10
β γ
a sin sin
We take 2 2 = a sin β sin γ 1
α 2 2 cos α
cos
2 2
Now see Question # 2

Question # 11
L.H.S = abc ( sin α + sin β + sin γ )
1 1 1
Since ∆ = ab sin γ = bc sin α = ca sin β
2 2 2
2∆ 2∆ 2∆
∴ sin γ = , sin α = , sin β =
ab bc ca
 2∆ 2∆ 2∆ 
Thus L.H.S = abc  + + 
 bc ac ab 
 2∆a + 2∆b + 2∆c 
= abc   = 2∆a + 2∆b + 2∆c
 abc 
= 2∆ ( a + b + c ) = 2∆ (2 s) Q 2s = a + b + c
= 4∆s = R.H.S

Question # 12 (i)
γ
L.H.S = (r1 + r2 ) tan
2
 ∆ ∆  (s − a )(s − b)
= + 
 s−a s−b s (s − c)
FSc-I - 12.8 - 8

 ∆ ( s − b) + ∆( s − a )  ( s − a )( s − b) s (s − c)
=  ⋅
 ( s − a )( s − b)  s ( s − c) s ( s − c)
 s − b + s − a  s ( s − a )( s − b)( s − c)
= ∆ 
 (s − a )(s − b)  s 2 ( s − c )2
 2s − a − b  ∆2
= ∆  2
 ( s − a )( s − b)  s ( s − c)
2

a+b+c−a−b ∆
= ∆  Q 2s = a + b + c
 ( s − a )(s − b)  s ( s − c)
∆ 2c ∆ 2c
= = 2 = c = R.H.S
s( s − a )( s − b)(s − c) ∆
γ
(ii) L.H.S = ( r3 − r ) cot *Correction
2
 ∆ ∆ 1  1 1 1
= −  γ = ∆  − 
 s − c s  tan 2  s − c s  (s − a )(s − b)
s (s − c)
 s − ( s − c)  s ( s − c)  c  s ( s − c) s (s − c)
= ∆  = ∆  ⋅
 s (s − c)  ( s − a )( s − b)  s ( s − c)  ( s − a )( s − b) s ( s − c )
 c  s 2 ( s − c) 2  c  s 2 ( s − c) 2
= ∆  = ∆ 
 s ( s − c)  s ( s − a )( s − b)( s − c )  s ( s − c)  ∆2
 c  s(s − c)
= ∆  = c = R.H.S
 s ( s − c )  ∆

If you found any error, submit at


http://www.mathcity.org/error

Error Analyst

Waiting for someone

Tuesday, March 22, 2005 *


Updated: September 1, 2007 *
mathcity.org Exercise 13.1 (Solutions)
Textbook of Algebra and Trigonometry for Class XI
Page 398

Merging man and maths Available online @ http://www.mathcity.org, Version: 1.0.0

Question # 1
 π π
(i) Suppose y = sin −1 (1) where y ∈  − , 
 2 2
⇒ sin y = 1
π π 
⇒ y= Q sin   = 1 *
2 2
 π π
(ii) Suppose y = sin −1 (−1) where y ∈  − , 
 2 2
⇒ sin y = −1
π  π
⇒ y=− Q sin  −  = −1 *
2  2
 3
(iii) Suppose y = cos −1   where y ∈ [0,π ]
 2 
3
⇒ cos y =
2
π π  3
⇒ y= Q cos   = *
6 6 2
 1   π π
(iv) Suppose y = tan −1  −  where y ∈  − , 
 3  2 2
1
⇒ tan y = −
3
π  π 1
⇒ y=− Q tan  −  = − *
6  6 3
(v) & (vi) Do yourself *
(vii) Suppose y = cot −1 ( −1) where y ∈ ] 0,π [
⇒ cot y = −1
1 1
⇒ =
cot y −1
⇒ tan y = −1
3π  3π 
⇒ y= Q tan   = −1 *
4  4 
 2   π π
(viii) Suppose y = cosec −1  −  where y ∈  − ,  , y ≠ 0
 3  2 2
2
⇒ cosec y = −
3
1 1
⇒ =
cosec y − 2
3
3
⇒ sin y = −
2
FSc-I / 13.1 - 2

π  π 3
⇒ y =− Q sin  −  = − *
3  3 2
(x) Do yourself *

Question # 2(i)
5  π π
Suppose α = sin −1 ………….. (i) where α ∈  − , 
13  2 2
5
⇒ sin α =
13
Now cosα = ± 1 − sin 2 α
 π π
Since α ∈  − ,  and value of sin is +ive therefore cos is +ive.
 2 2
cosα = + 1 − sin 2 α
2
5 25 144 12
= 1−   = 1− = =
 13  169 169 13
5
sin α 5
Now tan α = = 13 =
cosα 12 12
13
5
⇒ α = tan −1 ………….. (ii)
12
From (i) and (ii)
5 5
tan −1 = sin −1 Proved *
12 13
Question # 2(ii)
4 α
Suppose α = 2cos −1 ………….. (i) where ∈ [0,π ]
5 2
α 4 α 4
⇒ = cos −1 ⇒ cos =
2 5 2 5
α α
Now sin = ± 1 − cos 2
2 2
α
Since ∈ [0,π ] and value of cos is +ive therefore sin is +ive.
2
α α
sin = + 1 − cos 2
2 2
16 16 9 3
= 1− = 1− = =
25 25 25 5
α α
Now sin α = 2sin cos
2 2
3  4 24
⇒ sin α = 2     ⇒ sin α =
5  5 25
24
⇒ α = sin −1 ………….. (ii)
25
From (i) and (ii)
4 24
2cos −1 = sin −1 Proved *
5 25
FSc-I / 13.1 - 3
Question # 2(iii)
where α ∈ [0,π ]
4
Suppose α = cos −1 ………….. (i)
5
4
⇒ cosα =
5
Now sin α = ± 1 − cos 2 α
Since α ∈ [0,π ] and value of cos is +ive therefore sin is +ive.
sin α = + 1 − cos 2 α
2
4 16 9 3
= 1−   = 1− = =
5 25 25 5
4
cosα 4
Now cot α = = 5 =
sin α 3 3
5
4
⇒ α = cot −1 ………….. (ii)
3
From (i) and (ii)
4 4
cos −1 = cot −1 Proved *
5 3

Question 3
1  π π
(i) Suppose y = sin −1 where y ∈  − , 
2  2 2
1
⇒ sin y =
2
π π  1
⇒ y= Q sin   =
4 4 2

 1  π 1
Now cos  sin −1  = cos y = cos = Answer *
 2 4 2

where y ∈ [0,π ]
1
(ii) Suppose y = cos −1
2
1
⇒ cos y =
2
π π 1
⇒ y= Q cos =
3 3 2
 1 1 1 1
Now sec  cos −1  = sec y = = = =2 Answer *
 2 cos y cos π 1
3 2
(iii) Do yourself *
 π π
(iv) Suppose y = tan −1 (−1) where y ∈  − , 
 2 2
⇒ tan y = −1
π  π
⇒ y=− Q tan  −  = −1
4  4
cosec ( tan −1 (−1) ) = cosec y =
1 1 1
= = =− 2 *
( )
Now Answer
sin y sin − π − 1
4 2
FSc-I / 13.1 - 4
(v) Do yourself *
 π π
(vi) Suppose y = tan −1 (−1) where y ∈  − , 
 2 2
⇒ tan y = −1
π  π
⇒ y=− Q tan  −  = −1
4  4
 π
Now tan ( tan −1 (−1) ) = tan y = tan  −  = −1 Answer *
 4
1  π π
(vii) Suppose y = sin −1 where y ∈  − , 
2  2 2
1
⇒ sin y =
2
π π  1
⇒ y= Q sin   =
6 6 2
 1 π 1
Now sin  sin −1  = sin y = sin = Answer *
 2 6 2

 1  π π
(viii) Suppose y = sin −1  −  where y ∈  − , 
 2  2 2
1
⇒ sin y = −
2
π  π 1
⇒ y=− Q sin  −  =
6  6 2
  1   π 1
Now tan  sin −1  −   = tan y = tan  −  = − Answer *
  2   6  2
(ix) Do yourself

For latest news and updates visit


http://www.mathcity.org

If you found any error, submit at


http://www.mathcity.org/error

Error Analyst

Waiting for someone

Tuesday, March 22, 2005 *


Updated: September 1, 2007 *
mathcity.org Exercise 13.2 (Solutions)
Textbook of Algebra and Trigonometry for Class XI
Merging man and maths Available online @ http://www.mathcity.org, Version: 1.0.0

Question # 1
5 7
L.H.S = sin −1 + sin −1
13 25
(
Q sin −1 A + sin −1 B = sin −1 A 1 − B 2 + B 1 − A2 )
5 2
5 
2
 7  7 
= sin 
−1
1−   + 1−   
 13  
25 25  13  
 
5 49 7 25   5 576 7 144 
= sin −1  1− + 1−  = sin −1
 + 
 13 625 25 169   13 625 25 169 
 5  24  7  12    120 84  −1  204 
= sin −1    +    = sin −1  +  = sin  
 13  25  25  13    325 325   325 
π  204  π
= − cos −1   Q sin −1 θ = − cos −1 θ
2  325  2
 204  π
= cos −1 ( 0 ) − cos   Q = cos −1 (0)
 325  2

(
Q cos−1 A − cos−1 B = cos −1  AB + 1 − A2 1 − B 2 

)( ) 
 2 
2  

= cos  ( 0 ) 
−1 204
  325 

 + 1 (
− ( 0 )  

1)−


204
325

  
 
 
  41616     64009  
= cos −1  0 + (1 − 0 ) 1 −  = cos −1
 ( 1)   
  105625    105625 
  
 64009  −1 253
= cos −1   = cos = R.H.S 
 105625  325

Question # 2
1 1
L.H.S = tan −1 + tan −1
4 5
 1 +1 
−1    A+ B 
= tan 4 5 Q tan −1 A + tan −1 B = tan −1  
1− 1
 4( )( )
1 
5 
 1 − AB 

 9   9 
  9
= tan −1 20 = tan  20  = tan −1   = R.H.S
−1

1− 1   19   19 
 20   20 
Question # 3
12
Suppose α = sin −1 ……………… (i)
13
12
⇒ sin α =
13
2
 12  144 25 5
Now cosα = 1 − sin α = 1 −   = 1 −
2
= =
 13  169 169 13
FSc I / Ex 13.2 - 2

α 1 − cosα 1− 5 8
Now tan = = 13 = 13 = 8 = 4 = 2
2 1 + cosα 1+ 5 18 18 9 3
13 13
α 2 2
⇒ = tan −1   ⇒ α = 2 tan −1 ……………… (ii)
2 3 3
from (i) and (ii)
2 12
2 tan −1 = sin −1 proved. 
3 13
Question # 4
120
Suppose α = tan −1 ………………… (i)
119
120
⇒ tan α =
119
Now secα = 1 + tan 2 α
2
 120  14400 28561 169
= 1+   = 1+ = =
 119  14161 14161 119
1 1 119
So cosα = = =
secα 169 169
119
α 1 + cosα 1 + 119 288
288 144 12
Now cos = = 169 = 169 = = =
2 2 2 2 2 × 169 169 13
α 12 12
⇒ = cos −1 ⇒ α = 2cos −1 ………………… (ii)
2 13 13
From (i) and (ii)
120 12
tan −1 = 2cos −1 proved. 
119 13
Question # 5
1
Suppose α = sin −1 ………………… (i) *Correction
5
1
⇒ sin α =
5
2
Now cosα = 1 − sin α = 1 −  1  = 1 − 1 = 4 = 2
2

 5 5 5 5
1
sin α 5 =1
So tan α = =
cosα 2 2
5
⇒ α = tan −1 1 ………………… (ii)
2
From (i) and (ii)
1 1
sin −1 = tan −1
5 2
1 1
Now cot −1 3 = tan −1 Q cot −1 x = tan −1
3 x
1
And L.H.S = sin −1 + cot −1 3
5
1 1
= tan −1 + tan −1
2 3
FSc I / Ex 13.2 - 3

 1 +1   5 
= tan −1  2 3  = tan  6 
−1

 ( )( )
1− 1
2
1 
3 
1− 1 
 6
5  π
= tan  6  = tan −1 (1) =
−1
= R.H.S proved. 
5  4
 6
Question # 6
3 8
L.H.S = sin −1 + sin −1
5 17
3
( ) 8
( ) 
2 2
= sin −1  1 − 8 + 1− 3 
17 5
5 17 
3 64 8 9  
−1 3 225 8 16 
= sin −1  1 − + 1−  = sin  + 
 5 289 17 25   5 289 17 25 
 3  15  8  4    45 32 
= sin −1    +    = sin −1  + 
 5  17  17  5    85 85 
 77 
= sin −1   = R.H.S proved. 
 85 
Question # 7
77 3
L.H.S = sin −1 − sin −1
85 5
π 77   π 3 π
=  − cos −1  −  − cos −1  Q sin −1 x = − cos −1 x
2 85   2 5 2
π 77 π 3 3 77
= − cos −1 − + cos −1 = cos −1 − cos −1
2 85 2 5 5 85
  3  77  2 
= cos −1     + 1 − 3 ( )
1 − 77 
( )
2

 5  85   5  85  
 
 231
= cos −1 
 425
+ (1 − 9 25)(1 − 5929 7225) 
 20736 
 231
= cos −1 
 425
+ 16(25
1296 )(
7225 ) 


= cos −1 231

 425
+
180625


 231 144  −1  375   15 
= cos −1  +  = cos   = cos −1   = L.H.S proved 
 425 425   425   17 
Question # 8
63
Suppose α = cos −1
………………… (i)
65
63
⇒ cosα =
65

( 65 )
3969 256 16
2
Now sin α = 1 − cos 2 α = 1 − 63 = 1− = =
4225 4225 65
 16 
⇒ α = sin −1   ………………… (ii)
 65 
So from equation (i) and (ii)
 63   16 
cos −1   = sin −1  
 65   65 
FSc I / Ex 13.2 - 4

1
Now suppose β = tan −1 ………………… (iii)
5
⇒ tan β = 1
5
2
1 1 26 26
So secα = 1 + tan α = 1 +   = 1 +
2
= =
5 25 25 5
1 1 5
So cos β = = =
sec β 26 26
5
sin β
As = tan β ⇒ sin β = tan β ⋅ cos β
cos β
 1  5  1
⇒ sin β =     =
 5   26  26
1
⇒ β = sin −1 ………………… (iv)
26
From (iii) and (iv)
1 1
tan −1 = sin −1
5 26
63 1
Now L.H.S = cos −1 + 2 tan −1
65 5
16 1 16  1 1 
= sin −1 + 2sin −1 = sin −1 +  sin −1 + sin −1 
65 26 65  26 26 
 2 2 
16 1  1  1  1  
= sin −1
+ sin −1  1−   + 1−  
65  26  26  26  26  
 
16  1 1 1 1 
= sin −1 + sin −1  1− + 1− 
65  26 26 26 26 
16  1 25 1 25 
= sin −1 + sin −1  + 
65  26 26 26 26 
16  1 5 1 5 
= sin −1 + sin −1  ⋅ + ⋅ 
65  26 26 26 26 
16  5 5  16 5
= sin −1 + sin −1  +  = sin −1 + sin −1  
65  26 26  65  13 
  
2
 
2 
−1 16 5 5 16
= sin  1−   + 1−   
 65  
13 13  65  

 16 25 5 256 
= sin −1  1− + 1− 
 65 169 13 4225 
 16 25 5 256  
−1 16 144 5 3969 
= sin −1  1− + 1−  = sin  + 
 65 169 13 4225   65 169 13 4225 
 16 144 5 3969  −1  16  12  5  63  
= sin −1  +  = sin    +   
 65 169 13 4225   65  13  13  65  
 192 315  −1  3 
= sin −1  +  = sin   = R.H.S proved 
 845 845  5
FSc I / Ex 13.2 - 5

Question # 9
3 3 8
L.H.S = tan −1 + tan −1 − tan −1 *Correction
4 5 19
 3 +3 
= tan −1 4 5  − tan −1 8
1− 3
 ( )( )
4
3
5 
 19
 27   27 
  −1 8 8  27  8
= tan −1 20 − tan = tan  20  − tan −1
−1
= tan −1   − tan −1  
1− 9  19  11  19  11   19 
 20   20 
 27 − 8   425   425 
= tan 
−1 11 19  = tan 
−1 209  = tan 
−1 209 
 (
 1 + 27
11)( )8
19 
  1 + 216


209 
 1 + 216


209 
 425 
209  = tan −1 1 = π = R.H.S
= tan −1
() proved. 
 425  4
 209 

Question # 10 Do Yourself 

Question # 11
1 5
L.H.S = tan −1 + tan −1
11 6
= Solve this ……………….. (i)
1 1
R.H.S = tan −1 + tan −1
3 2
= Solve this ………………. (ii)
From (i) and (ii)
L.H.S = R.H.S 
Question # 12
1 1
L.H.S = 2 tan −1 + tan −1
3 7
1 1 1
= tan −1 + tan −1 + tan −1
3 3 7
Now do yourself as Question # 9. 

Question # 13
Suppose y = sin −1 x
⇒ sin y = x
Since cos y = 1 − sin 2 y = 1 − x2
⇒ cos ( sin −1 x ) = 1 − x 2 Proved 

Question # 14
Suppose y = cos−1 x
Then cos y = x
Also sin y = 1 − cos 2 y = 1 − x 2
Now sin ( 2cos −1 x ) = sin ( 2 y )
= 2sin y ⋅ cos y
= 2 1 − x2 ⋅ x
= 2 x 1 − x2 Proved 
FSc I / Ex 13.2 - 6

Question # 15
Suppose y = sin −1 x ⇒ sin y = x
& cos y = 1 − sin 2 y = 1 − x 2
Now cos ( 2sin −1 x ) = cos 2 y
= cos 2 y − sin 2 y

( ) − x =1− x − x
2
= 1− x 2 2 2 2

= 1 − 2x 2 Proved 

Question # 16
Suppose y = tan −1 (− x) …………….. (i)
⇒ tan y = − x ⇒ − tan y = x
⇒ tan(− y ) = x Q − tan θ = tan(−θ )
⇒ − y = tan −1 x
⇒ y = − tan −1 x ………………. (ii)
From equation (i) and (ii)
tan −1 (− x ) = − tan −1 x Proved 

Question # 17 Do yourself as above

Question # 18
Suppose y = π − cos −1 x ………………… (i)
−1
⇒ π − y = cos x ⇒ cos(π − y ) = x
⇒ cosπ cos y + sin π sin y ) = x ⇒ (−1)cos y + (0)sin y ) = x
⇒ − cos y + 0 = x ⇒ − cos y = x
⇒ cos y = − x ⇒ y = cos −1 (− x ) ………… (ii)
From (i) and (ii)
cos −1 (− x ) = π − cos −1 x Proved 
Question # 19
Suppose y = sin −1 x ⇒ sin y = x
Now cos y = 1 − sin 2 y = 1 − x 2
sin y x
& tan y = =
cos y 1 − x2
tan ( sin −1 x ) = tan y =
x
Now proved. 
1 − x2
Question # 20
1 1
Since x = sin −1 ⇒ sin x =
2 2

( 2)
2
Now cos x = 1 − sin 2 x = 1 − 1 = 1− 1 = 3 = 3
4 4 2
sin x 1 1 1 1
tan x = = 3
2
= , cot x = = = 3
cos x 2 3 tan x 1 3
1 1 2 1 1
sec x = = 3 = , cosec x = = =2
cos x 3 sin x 1
2 2

Made By: Atiq ur Rehman (mathcity@gmail.com) http://www.mathcity.org)


The End
mathcity.org Exercise 14 (Solutions)
Textbook of Algebra and Trigonometry for Class XI
Merging man and maths Available online @ http://www.mathcity.org, Version: 1.1.3

Question # 1
3  3
(i) Since sin x = − ⇒ x = sin −1  − 
2  2 
5π 4π
⇒ x= , where x ∈ [0, 2π ]
3 3
(ii) Since cosecθ = 2
1 1
⇒ = 2 ⇒ sin θ =
sinθ 2
1 π 5π
⇒ θ = sin −1   = , where θ ∈ [0, 2π ]
2 6 6
(iii) Do yourself
1
(iv) Since cot θ =
3
1 1
⇒ = ⇒ tanθ = 3
tan θ 3
π 4π
⇒ θ = tan −1 3 ( )
⇒ θ=
3
,
3
where θ ∈ [0, 2π ]

Question # 2 (i)
1 1
Since tan 2 θ = ⇒ tan θ = ±
3 3
1 1
⇒ tan θ = or tan θ = −
3 3
 1   1 
⇒ θ = tan −1   or θ = tan −1  − 
 3  3
π 5π
⇒ θ= or θ=
6 6
Since period of tanθ is π
π 5π
Therefore general value of θ = + nπ , + nπ
6 6
π   5π 
So Solution Set =  + nπ  U  + nπ  where n ∈ ¢
6   6 

Question # 2 (ii)
4 2
Since cosec2 θ = ⇒ cosecθ = ±
3 3
2 2
⇒ cosecθ = or cosecθ = −
3 3
3 3
⇒ sin θ = or sin θ = −
2 2
 3  3
⇒ θ = sin −1   or θ = sin −1  − 
 2   2 
π 2π 4π 5π
⇒ θ= , or θ= ,
3 3 3 3
FSc I / Ex. 14 - 2
Since period of cosecθ is 2π
π 2π 4π 5π
Therefore general value of θ = + 2nπ , + 2nπ , + 2nπ , + 2nπ
3 3 3 3
π   2π   4π   5π 
Solution set =  + 2nπ  U  + 2nπ  U  + 2nπ  U  + 2nπ  where n ∈ ¢ .
3   3   3   3 

Question # 2 (iii)
4 2
Since sec 2 θ = ⇒ secθ = ±
3 3
2 2
⇒ secθ = or secθ = −
3 3
3 3
⇒ cosθ = or cosθ = −
2 2
 3  3
⇒ θ = cos −1   or θ = cos −1  − 
 2   2 
π 11π 5π 7π
⇒ θ= , or θ= ,
6 6 6 6
Q period of secθ is 2π
π 5π 7π 11π
∴ general values of θ = + 2nπ , + 2nπ , + 2nπ , + 2nπ
6 6 6 6
π   5π   7π   11π 
S.Set =  + 2nπ  U  + 2 nπ  U  + 2nπ  U  + 2nπ  where n ∈ ¢ .
6  6   6   6 

Question # 2 (iv)
Do yourself

Question # 3
3tan 2 θ + 2 3 tan θ + 1 = 0
( ) ( )
3 tan θ (1) + (1) = 0
2
⇒ 3 tan θ +2
2

( ) ⇒ ( )
2
⇒ 3 tan θ + 1 = 0 3 tan θ + 1 = 0 ⇒ 3 tanθ = −1
1  1  5π
⇒ tan θ = − ⇒ θ = tan −1  −  ⇒θ= 6
3  3
Q period of tanθ is π

∴ general value of θ = + nπ , n ∈ ¢
6
Question # 4
tan 2 θ − secθ − 1 = 0
⇒ ( sec 2 θ − 1) − secθ − 1 = 0
⇒ sec 2 θ − 1 − secθ − 1 = 0 ⇒ sec 2 θ − secθ − 2 = 0
⇒ sec 2 θ − 2secθ + secθ − 2 = 0 ⇒ secθ ( secθ − 2 ) + 1( secθ − 2 ) = 0
⇒ ( secθ + 1)( secθ − 2 ) = 0
⇒ ( secθ + 1) = 0 or ( secθ − 2 ) = 0
⇒ secθ = −1 or secθ = +2
1
⇒ cosθ = −1 or cosθ =
2
FSc I / Ex. 14 - 3

1
⇒ θ = cos −1 ( −1) or θ = cos −1  
2
3π π 5π
⇒ θ= or θ= ,
2 6 6
Q period of cosθ is 2π
3π π 5π
∴ general value of θ = + 2nπ , + 2nπ , + 2nπ where n ∈ ¢
2 6 6
Question # 5
2sin θ + cos 2 θ − 1 = 0
⇒ 2sinθ + 1 − sin 2 θ − 1 = 0 ⇒ − sin 2 θ + 2sinθ = 0
⇒ − sin θ ( sinθ − 2 ) = 0
⇒ − sin θ = 0 or sinθ − 2 = 0
⇒ sin θ = 0 or sinθ = 2
⇒ θ = sin −1 ( 0 ) Which does not hold as sin θ ∈ [−1,1]
⇒ θ =0, π
Q period of sinθ is 2π
∴ general value of θ = 0 + 2nπ , π + 2nπ
= 2 nπ , π + 2nπ where n ∈ ¢

Question # 6
3cos2 θ − 2 3 sin θ cosθ − 3sin 2 θ = 0
Dividing throughout by cos 2 θ
3cos 2 θ 2 3 sin θ cosθ 3sin 2 θ
− − =0
cos 2 θ cos 2 θ cos 2 θ
⇒ 3 − 2 3 tanθ − 3tan 2 θ = 0
⇒ − 3tan 2 θ − 2 3 tanθ + 3 = 0
⇒ 3tan 2 θ + 2 3 tan θ − 3 = 0 ×ing by -1

( ) − 4 ( 3)( −3 )
2
−2 3 ± 2 3
⇒ tan θ =
2 ( 3)
−2 3 ± 12 + 36 −2 3 ± 48
⇒ tan θ = =
6 6
−2 3 ± 16 × 3 −2 3 ± 4 3
= =
6 6
−2 3 + 4 3 2 3 −2 3 − 4 3 6 3
⇒ tan θ = = or tan θ = =−
6 6 6 6
3 3 1
⇒ tan θ = = = or ⇒ tanθ = − 3
( )
2
3 3 3

 1 
⇒ θ = tan −1 
 3
 or (
θ = tan −1 − 3 )
π 11π
⇒ θ= or θ=
6 6
Q period of tanθ is π
π 11π
∴ general value of θ = + nπ , + nπ where n ∈ ¢ .
6 6
FSc I / Ex. 14 - 4
Question # 8
4sin 2 θ − 8cosθ + 1 = 0
⇒ 4 (1 − cos 2 θ ) − 8cosθ + 1 = 0 ⇒ 4 − 4cos 2 θ − 8cosθ + 1 = 0
⇒ − 4cos 2 θ − 8cosθ + 5 = 0 ⇒ 4cos 2 θ + 8cosθ − 5 = 0
⇒ 4cos 2 θ + 10cosθ − 2cosθ − 5 = 0
⇒ 2cosθ ( 2cosθ + 5 ) − 1( 2cosθ + 5 ) = 0 ⇒ ( 2cosθ + 5)( 2cosθ − 1) = 0
⇒ 2cosθ + 5 = 0 or 2cosθ − 1 = 0
⇒ 2cosθ = −5 or 2cosθ = 1
−5 1
⇒ cosθ = or cosθ =
2 2
 −5  1
⇒ θ = cos −1   or θ = cos −1  
 2  2
π 5π
Which is not possible as cosθ ∈ [−1,1] or θ = ,
3 3
Q period of cosθ is 2π
π 5π
∴ general value of θ = + 2π n, + 2nπ where n ∈ ¢ .
3 3
Question # 9
3 tan x − sec x − 1 = 0 ………… (i )
sin x 1
⇒ 3 − −1 = 0
cos x cos x
⇒ 3 sin x − 1 − cos x = 0 × ing by cosθ .
⇒ 3 sin x − 1 = cos x
On squaring both sides.
( )
3 sin x − 1 = ( cos x )
2 2

⇒ 3sin 2 x − 2 3sin x + 1 = cos 2 x


⇒ 3sin 2 x − 2 3 sin x + 1 = 1 − sin 2 x
⇒ 3sin 2 x − 2 3sin x + 1 − 1 + sin 2 x = 0 ⇒ 4sin 2 x − 2 3 sin x = 0
(
⇒ 2sin x 2sin x − 3 = 0 )
⇒ 2sin x = 0 or 2sin x = 3
3
⇒ sin x = 0 or sin x =
2
 3
⇒ x = sin −1 ( 0 ) or x = sin −1  
 2 
π 2π
⇒ x=0 , π or x= ,
3 3
Now to check extraneous roots put x = 0 in (i )
L.H.S = 3 tan(0) − sec(0) − 1 = 0 − 1 − 1 = −2 ≠ 0 = R.H.S
Implies that x = 0 is an extraneous root of given equation.
Now put x = π in (i)
L.H.S = 3 tan(π ) − sec(π ) − 1 = 0 − (−1) − 1 = 0 = R.H.S
Implies that x = π is a root of the equation.
π
Now put x = in (i )
3
FSc I / Ex. 14 - 5

L.H.S =
π  π 
( )
3 tan   − sec   − 1 = 3 3 − 2 − 1 = 1 − 2 − 1 = 0 = R.H.S
3 3
π
Implies that x = is a root of given equation.. Since period of tan is π .
3

Now put x = in (i )
3
 2π   2π 
L.H.S = 3 tan   − sec   −1
 3   3 
( )
= 3 − 3 − ( −2 ) − 1 = −3 + 2 − 1 = − 2 = R.H.S

Implies x = is an extraneous root of given equation.
3
Q period of sin x is 2π
π
∴ general values of x = π + 2nπ , + 2nπ
3
π 
Solution Set = {π + 2nπ } U  + 2nπ  where n ∈ ¢ .
3 
Question # 10
cos 2 x = sin 3x
⇒ cos 2 x − sin 2 x = 3sin x − 4sin 3 x Q cos 2 x = cos 2 x − sin 2 x
⇒ cos 2 x − sin 2 x − 3sin x + 4sin 3 x = 0 sin 3 x = 3sin x − 4sin 3 x
⇒ (1 − sin 2 x ) − sin 2 x − 3sin x + 4sin 3 x = 0
⇒ 1 − 2sin 2 x − 3sin x + 4sin 3 x = 0
⇒ 4sin 3 x − 2sin 2 x − 3sin x + 1 = 0
Take sin x = 1 as a root then by synthetic division

1 4 -2 -3 1
↓ 4 2 -1
4 2 -1 0

⇒ ( sin x − 1) ( 4sin 2 x + 2sin x − 1) = 0


⇒ sin x − 1 = 0 or 4sin 2 x + 2sin x − 1 = 0
−2 ± (2) − 4 ( 4 )( −1)
2

⇒ sin x = 1 or sin x =
2 ( 4)
−2 ± 4 + 16 −2 ± 20
⇒ x = sin −1 (1) or sin x = =
8 8
π −2 + 20 −2 − 20
⇒ x= or sin x = or sin x =
2 8 8
sin x = 0.309 or sin x = −0.809
⇒ x = sin ( 0.309 ) or
−1
x = sin −1 ( −0.809 )
≈ 18, 162 or ≈ 234 , 306
π π π π
⇒ x = 18 ⋅ , 162 ⋅ or x = 234 ⋅ , 306 ⋅
180 180 180 180
π 9π 13π 17π
= , or = ,
10 10 10 10
FSc I / Ex. 14 - 6
Q period of sin x is 2π
π 9π 13π 17π π
∴ general value of x = + 2nπ , + 2nπ , + 2nπ , + 2nπ , + 2nπ
10 10 10 10 2
π   9π  13π  17π  π 
S.Set =  + 2nπ  U  + 2nπ  U  + 2nπ  U  + 2nπ  U  + 2nπ 
10   10   10   10  2 

Question No. 11
sec3θ = secθ
1 1
⇒ =
cos3θ cosθ
⇒ cos3θ = cosθ
⇒ 4cos3 θ − 3cosθ = cosθ Q cos3θ = 4cos3 θ − 3cosθ
⇒ 4cos3 θ − 3cosθ − cosθ = 0
⇒ 4cos3 θ − 4cosθ = 0
( )
⇒ 4cosθ cos 2 θ − 1 = 0
⇒ 4cosθ = 0 or cos 2 θ − 1 = 0
⇒ cosθ = 0 or cos 2 θ = 1
⇒ θ = cos −1 (0) or cosθ = ±1 ⇒ θ = cos −1 (1) , θ = cos −1 ( −1)
π 3π
⇒ θ= , or θ =0 ,π
2 2
Q period of cosθ is 2π
π 3π
∴ general values of θ = + 2nπ , + 2nπ , 0 + 2nπ , π + 2nπ
2 2
π 3π
= + 2nπ , + 2nπ , nπ
2 2
π   3π 
S. Set =  + 2nπ  U  + 2nπ  U {nπ } where n ∈ ¢ .
2   2 

Question # 12
tan 2θ + cotθ = 0
⇒ tan 2θ = − cot θ
sin 2θ cosθ 2sin θ cosθ cosθ
⇒ =− ⇒ =−
cos 2θ sin θ cos θ − sin θ
2 2
sin θ
⇒ ( 2sin cosθ )( sin θ ) = ( − cosθ ) ( cos θ − sin θ )
2 2

⇒ 2sin 2 θ cosθ = − cos3 θ + sin 2 θ cosθ


⇒ 2sin 2 θ cosθ + cos3 θ − sin 2 θ cosθ = 0
⇒ sin 2 θ cosθ + cos3 θ = 0
( )
⇒ cosθ sin 2 θ + cos 2 θ = 0 ⇒ cosθ (1) = 0
⇒ θ = cos −1 (0)
π 3π
= ,
2 2
Q period of cosθ is 2π
π 3π
∴ general values of θ = + 2nπ , + 2nπ
2 2
π   3π 
S. Set =  + 2nπ  U  + 2nπ  where n ∈ ¢ .
2   2 
FSc I / Ex. 14 - 7
Question # 13
sin 2 x + sin x = 0
⇒ 2sin x cos x + sin x = 0 Q sin 2θ = 2sin θ cosθ
⇒ sin x ( 2cos x + 1) = 0
⇒ sin x = 0 or 2cos x + 1 = 0
1  1
⇒ x = sin −1 (0) or cos x = − ⇒ x = cos −1  − 
2  2
2π 4π
⇒ x=0,π or x= ,
3 3
Q period of sin x and cos x is 2π
2π 4π
∴ general values of x = 0 + 2nπ , π + 2 nπ , + 2nπ , + 2nπ
3 3
2π 4π
= nπ , + 2nπ , + 2nπ
3 3
 2π   4π 
So solution set = { nπ } U  + 2 nπ  U  + 2nπ  where n ∈ ¢ .
 3   3 

Question # 14
sin 4 x − sin 2 x = cos3x
 4 x + 2x   4x − 2 x 
⇒ 2cos   sin   = cos3 x
 2   2 
⇒ 2cos3 x sin x − cos3 x = 0
⇒ cos3x ( 2sin x − 1) = 0
⇒ cos3 x = 0 or 2sin x − 1 = 0
1
⇒ 3x = cos −1 (0) , sin x =
2
π 3π 1
⇒ 3x = , , x = sin −1  
2 2 2
π π π 5π
⇒ x= , , x= ,
6 2 6 6

Since period of cos3x is and period of sin x is 2π
3
π 2nπ π 2nπ π 5π
∴ general values of x = + , + , + 2nπ , + 2nπ
6 3 2 3 6 6
 π 2nπ   π 2 nπ  π   5π 
So solution set =  + U +  U  + 2nπ  U  + 2nπ  where n ∈ ¢ .
6 3  2 3  6   6 

Question # 15
sin x + cos3x = cos5 x
⇒ sin x = cos5 x − cos3 x
 5 x + 3x   5 x − 3 x 
⇒ sin x = −2sin   sin  
 2   2 
⇒ sin x = −2sin 4 x sin x
⇒ sin x + 2sin 4 x sin x = 0
⇒ sin x (1 + 2sin 4 x ) = 0
⇒ sin x = 0 or 1 + 2sin 4 x = 0
1  1
⇒ x = sin −1 (0) or sin 4 x = − ⇒ 4 x = sin −1  − 
2  2
FSc I / Ex. 14 - 8
7π 11π 7π 11π
⇒ x=0 , π or 4x = , ⇒ x= ,
6 6 24 24
2π π
Since period of sin x is 2π and period of sin 4x is =
4 2
7π nπ 11π nπ
∴ general values of x = 0 + 2nπ , π + 2nπ , + , +
24 2 24 2
 7π nπ  11π nπ 
So solution set = {2 nπ } U {π + 2nπ } U  + U  +  where n ∈ ¢ .
 24 2   24 2 

Question # 16
sin 3x + sin 2 x + sin x = 0
⇒ ( sin3x + sin x ) + sin 2 x = 0
 3x + x   3x − x 
⇒ 2sin   cos   + sin 2 x = 0 ⇒ 2sin 2 x cos x + sin 2 x = 0
 2   2 
⇒ sin 2 x ( 2cos x + 1) = 0
⇒ sin 2 x = 0 or 2cos x + 1 = 0
1
⇒ 2 x = sin −1 (0) or cos x = −
2
 1
⇒ 2x = 0 , π or x = cos −1  − 
 2
π 2π 4π
⇒ x=0 , or x= ,
2 3 3

Since period of sin 2x is = π and period of cos x is 2π
2
π 2π 4π
∴ general values of x = 0 + nπ , + nπ , + 2nπ , + 2nπ
2 3 3
π   2π   4π 
S. Set = {nπ } U  + nπ  U  + 2nπ  U  + 2nπ  where n ∈ ¢ .
2   3   3 

Question # 17
sin 7 x − sin x = sin3x
 7x + x   7x − x 
⇒ 2cos   sin   = sin 3 x
 2   2 
⇒ 2cos 4 x sin 3 x − sin 3x = 0
⇒ sin 3x ( 2cos 4 x − 1) = 0
⇒ sin 3 x = 0 or 2cos 4 x − 1 = 0
1
⇒ 3x = sin −1 ( 0 ) or cos 4 x =
2
 1  π 5π
⇒ 3x = 0 , π or 4 x = cos −1   = ,
 
2 3 3
π π 5π
⇒ x=0 , or x= ,
3 12 12
2π 2π π
Since period of sin 3x is and period of cos 4x is =
3 4 2
2nπ π 2nπ π nπ 5π nπ
∴ general values of x = 0 + , + , + , +
3 3 3 12 2 12 2
 2nπ   π 2 nπ   π nπ   5π nπ 
So S. set =  U + U + U +  where n ∈ ¢ .
 3  3 3  12 2   12 2 
FSc I / Ex. 14 - 9

Question # 18
sin x + sin 3x + sin 5 x = 0
⇒ ( sin5 x + sin x ) + sin 3x = 0
 5x + x   5x − x 
⇒ 2sin   ⋅ cos   + sin 3 x = 0
 2   2 
⇒ 2sin 3 x cos 2 x + sin 3 x = 0
⇒ sin 3x ( 2cos 2 x + 1) = 0
⇒ sin 3 x = 0 or 2cos 2 x + 1 = 0
 1
⇒ 3x = sin −1 ( 0 ) 2cos 2 x = −1 ⇒ 2 x = cos −1  − 
or
 2
2π 4π
⇒ 3x = 0 , π or 2x = ,
3 3
π π 2π
⇒ x=0 , or x= ,
3 3 3
2π 2π
Since period of sin 3x is and period of cos 2x is =π
3 2
2nπ π 2nπ π 2π
∴ general values of x = 0 + , + , + nπ , + nπ
3 3 3 3 3
 2nπ  π 2nπ  π   2π 
S.Set =  U +  U  + nπ  U  + nπ  where n ∈ Z
 3  3 3  3   3 

Question # 19
sinθ + sin 3θ + sin 5θ + sin 7θ = 0
⇒ ( sin 7θ + sinθ ) + ( sin5θ + sin 3θ ) = 0
 7θ + θ   7θ − θ   5θ + 3θ   5θ − 3θ 
⇒ 2sin   cos   + 2sin   cos  =0
 2   2   2   2 
⇒ 2sin 4θ cos3θ + 2sin 4θ cosθ = 0
⇒ 2sin 4θ ( cos3θ + cosθ ) = 0
  3θ + θ   3θ − θ 
⇒ 2sin 4θ  2cos   cos   = 0
  2   2 
⇒ 4sin 4θ cos 2θ cosθ = 0
⇒ sin 4θ = 0 or cos 2θ = 0 or cosθ = 0
⇒ 4θ = sin −1 ( 0 ) , 2θ = cos −1 ( 0 ) , θ = cos −1 ( 0 )
π 3π π 3π
⇒ 4θ = 0 , π or 2θ = , or θ = ,
2 2 2 2
π π 3π
⇒ θ =0 , or θ= ,
4 4 4
2π π 2π
Since period of sin 4θ is = , cos 2θ is = π and cosθ is 2π
4 2 2
nπ π nπ π 3π π 3π
∴ general values of θ = 0 + , + , + nπ , + nπ , + 2nπ , + 2nπ
2 4 2 4 2 2 2
 nπ   π nπ  π   3π  π   3π 
S. Set =   U  +  U  + nπ  U  + nπ  U  + 2nπ  U  + 2nπ 
 2  4 2  4   2  2   2 
FSc I / Ex. 14 - 10
Question # 20
cosθ + cos3θ + cos5θ + cos 7θ = 0
⇒ ( cos7θ + cosθ ) + ( cos5θ + cos3θ ) = 0
 7θ + θ   7θ − θ   5θ + 3θ   5θ − 3θ 
⇒ 2cos   cos   + 2cos   cos  =0
 2   2   2   2 
⇒ 2cos 4θ cos3θ + 2cos 4θ cosθ = 0
⇒ 2cos 4θ ( cos3θ + cosθ ) = 0
  3θ + θ   3θ − θ  
⇒ 2cos 4θ  2cos   cos   = 0
  2   2 
⇒ 4cos 4θ cos 2θ cosθ = 0
Now do yourself as above question.

For latest news and updates visit


http://www.mathcity.org

If you found any error, submit at


http://www.mathcity.org/error

Error Analyst

Waiting for someone

Tuesday, March 22, 2005 *


Updated: September 1, 2007 *

S-ar putea să vă placă și